You are on page 1of 302

6044

BMJ

1. An 18-month-old infant male is referred to the ophthalmology clinic with


progressive visual disturbance.

He has recently begun to display abnormal posturing and protracted contractures of limb and
neck muscles. His GP has performed fundoscopy and remarked that the appearances are
"very abnormal".

On the basis of this information, which of the following is the least likely diagnosis?

(Please select 1 option)

GM2 gangliosidosis

Homocystinuria

Neuronal ceroid-lipofuscinosis

Spatz syndrome

Spinocerebellar atrophy type 3

This boy demonstrates dystonic posturing, in association with retinal abnormalities.

Spinocerebellar atrophy type 3 (SCA3) is characterised by progressive cerebellar ataxia and


variable findings including

A dystonic-rigid syndrome
A parkinsonian syndrome, or
A combined syndrome of dystonia and peripheral neuropathy.

There are no associated ocular abnormalities. The age of onset of SCA3 is variable but is
usually in the second to the fourth decade.

The causes of secondary dystonia with occular abnormalities include:

Spatz syndrome disorders (including Leber's), ceroid-lipofuscinosis and gangliosidosis.


Spatz syndrome is a very rare disease beginning in childhood, with degeneration of the
globus pallidus, red nucleus, and reticular part of the substantia nigra of the brain. It is
characterised by progressive Parkinson-like rigidity, athetotic movements, and
progressive mental and emotional retardation.
Neuronal ceroid-lipofuscinosis most commonly presents at age 6 to 18 months and runs
a subacute course. It is characterised by rapid deterioration with psychomotor
retardation, loss of speech, seizures, ataxia, blindness, hypotonia, microcephaly and
occasional convulsions.
The classic infantile subtype of GM2 gangliosidosis combines the features of a
neurolipidosis (that is, neurodegeneration, macular cherry-red spots) with those of a
mucopolysaccharidosis (that is, visceromegaly, dysostosis multiplex, coarsened facial
features). This form of gangliosidosis most frequently presents in early infancy and may
be evident at birth.
Homocystinuria may be associated with lens dislocation, cataracts, glaucoma and retinal
detachment. It is also associated with mental retardation and the pro-coagulant
features may result in ischaemic cerebral events and complex neurological signs.

Next question Go to summary

6186

2. Which of the following patients requires the highest level of anticoagulation?

(Please select 1 option)

Bioprosthetic valve implantation in aortic position with chronic atrial fibrillation

Non-valvar atrial fibrillation with previous embolic stroke

Severe mitral stenosis with atrial fibrillation

Starr-Edwards valve in the mitral position in sinus rhythm

St Jude (bi-leaflet) valve in mitral position in sinus rhythm

The old generation of ball and cage type mechanical prostheses are most thrombogenic and
need high levels of anticoagulation (INR 3.5 to 4.5).

The newer bileaflet prosthetic valves (St. Jude) require less anticoagulation (INR 2.5 to 3.5).

As a general rule, mitral prostheses are more likely to thrombose than aortic prostheses and
so need greater levels of anticoagulation.

Non-valvar AF needs moderate levels of anticoagulation (INR 2 to 2.5).

Next question Go to summary

Related Articles (BMJ)


ABC of antithrombotic therapy: Valvar heart disease and prosthetic heart valves -- Goldsmith et
al. 325 (7374): 1228 -- BMJ
Read article

Choice of heart valve prosthesis -- Bloomfield 87 (6): 583 -- Heart


Read article
Congenital heart disease in pregnancy -- Head and Thorne 81 (955): 292 -- Postgraduate
Medical Journal
Read article

3. A 25-year-old man presents to the Emergency department with shortness of


breath.

One week ago he developed influenza and has become more short of breath and
fatigued in the last 24 hours. His temperature is 38.5°C, his SaO2 is 90% on 2L of
oxygen, a blood pressure 100/60 mmHg and heart rate 120/min. The CXR shows
patchy consolidation.

Which antibiotic therapy should you select for this man?

(Please select 1 option)

Amoxicillin

Amoxicillin and flucloxacillin

Co-amoxiclav and clarithromycin

Co-amoxiclav

Flucloxacillin Incorrect answer selected

This gentleman has community acquired pneumonia (CAP). The recent history of influenza
may lead you to consider staphylococcus aureus as the possible underlying organism,
although this is an uncommon cause of CAP in the UK. It is more common in the winter
months, and coincident influenza-type symptoms in approximately 40%. Pneumonia
complications approximately 3% of cases of influenza, 10% of those admitted have been
confirmed to be due to Staphyloccus aureus.

In the majority of patients CAP should be confirmed by chest radiography before the
commencement of antibiotics. However, if patients are critically unwell they should be
treated for the presumptive diagnosis. Antibiotic treatment should always be initiated within 4
hours of presentation.

CAP caused by Staphyloccus aureus is more likely to present with multilobar shadowing,
cavitation, pneumatoceles and spontaneous pneumothorax than other organisms. However,
there are no characteristic features of chest radiographs that allow a confident prediction of
the likely pathogen. Therefore, the general guidelines for treatment of CAP should be
followed until an organism is identified. Staphylococcus aureus carries a high mortality, and
therefore if suspected treatment should initially be for a severe CAP (see below for details).

Low severity CAP (CURB 0-1) can be treated with amoxicillin 500mg TDS PO. CURB 2 CAP
should be treated with amoxicillin 500 mg-1 g TDS and clarithromycin 500 mg BD.
Alternatives are available if patients are allergic to any of the above combinations. High
severity CAP (CURB 3-5) should be treated as soon as possible with co-amoxiclav 1.2 g
TDS and clarithromycin 500 mg BD.
The oral route is recommended in those with low and moderate severity CAP. Patients
treated with parenteral antibiotics initially should be switched to an oral regimen once clinical
improvement is seen and the patients has been afebrile for at least 24 hours. For most
patients with uncomplicated CAP 7 days of antibiotic treatment is recommended. For those
with high severity pneumonia where an organism has not been identified, 7-10 days
treatment is indicated and extended to 14-21 days where clinically needed.

If Staphylococcus aureus is identified, treatment should be altered. Non-MRSA organisms


should be treatment with flucloxacillin and/or rifampicin; an alternative for penicillin-allergic
patients is teicoplanin and rifampicin. MRSA should be treated with vancomycin. A
prolonged antibiotic course is indicated.

References & Further Reading:

1. Lim WS, et al. BTS guidelines for the management of community acquired pneumonia in
adults: update 2009. Thorax2009;64:iii1-iii55
2. Macfarlane J. BTS Guidelines for the Management of Community Acquired Pneumonia in
Adults. Thorax2001;56:iv1-iv64

Next question

4. Which of the following statements is of aetiological significance in a patient with


cardiomyopathy?

(Please select 1 option)

AST of 50 IU/l with a bilirubin of 12 mmol/l in a 50-year-old lady

The presence of a soft pan-systolic murmur in the mitral area

The presence of diabetes mellitus in a tanned patient

The presence of sinus tachycardia with BP 140/80 mmHg

T-wave flattening in the inferior ECG leads in a 60-year-old man

The most common form of cardiomyopathy is dilated cardiomyopathy, with dilatation of the
left ventricle and reduction in the ejection fraction.

Often no cause is found, although rarely, it can be familial. A cause should be looked for in
most cases.

The commonest causes are

Ischaemic heart disease (Q-waves on the ECG, history of myocardial infarction [MI]) and
Hypertension.
Non-specific ECG changes and atrial fibrillation are common and do not point to a specific
cause.

Minor abnormalities of liver function result from liver congestion.

More marked abnormalities and diabetes in a pigmented patient would suggest


haemochromatosis.

Mitral regurgitation and arrhythmias are common in dilated cardiomyopathy of any cause.

Next question Go to summary

Related Articles (BMJ)


Fatal thyrotoxic cardiomyopathy in a young man -- Soh and Croxson 337: a531 -- BMJ
Read article

Enteroviral infection causing fatal myocarditis and subclinical myopathy -- Arbustini et al. 83 (1):
86 -- Heart
Read article

Diastolic dysfunction is associated with poor survival in patients with cirrhosis with transjugular
intrahepatic portosystemic shunt -- Cazzaniga et al. 56 (6): 869 -- Gut
Read article

5. A 60-year-old patient with diabetes presents with gradual deterioration of vision


and dazzling of view in well-lit places.

What is the most likely diagnosis?

(Please select 1 option)

Acute iritis

Cataracts

Subconjunctival haemorrhage

Thyroid-associated ophthalmopathy

Viral conjunctivitis

Diabetes mellitus is a common cause of pre-senile cataracts. The pointers in this case are
the gradual deterioration in vision and the dazzling effect of light (caused by the opacities in
the lens).

Common causes of cataracts in over 40s include diabetes, glaucoma and macular
degeneration.
Acute iritis may cause a degree of photophobia, however it is a cause of red eye and is
usually painful.

6. A 48-year-old HGV driver refuses to inform the DVLA of severe diabetic


retinopathy. You wish to inform the DVLA.

What consent should be obtained?

(Please select 1 option)

Consent from court of law

Consent from next of kin if possible

No consent required

Verbal consent required

Written consent required

It is the responsibility of the patient to inform the DVLA of their medical condition. It is the
responsibility of the doctor to advise the patient. However there are exceptional
cirumstances where the vehicle licence holder has breached their legal duty and continues
to drive despite medical advise.

The GMC has issued clear guidelines applicable to such circumstances, which state:

The driver is legally responsible for informing the DVLA about such a condition or treatment.
However, if a patient has such a condition, you should explain to the patient:

that the condition may affect their ability to drive (if the patient is incapable of
understanding this advice, for example, because of dementia, you should inform the
DVLA immediately), and
that they have a legal duty to inform the DVLA about the condition.

If a patient refuses to accept the diagnosis, or the effect of the condition on their ability to
drive, you can suggest that they seek a second opinion, and help arrange for them to do so.
You should advise the patient not to drive in the meantime.

If a patient continues to drive when they may not be fit to do so, you should make every
reasonable effort to persuade them to stop. As long as the patient agrees, you may discuss
your concerns with their relatives, friends or carers.

If you do not manage to persuade the patient to stop driving, or you discover that they are
continuing to drive against your advice, you should contact the DVLA immediately and
disclose any relevant medical information, in confidence, to the medical adviser.
Before contacting the DVLA you should try to inform the patient of your decision to disclose
personal information. You should then also inform the patient in writing once you have done
so.

A good knowledge of these guidelines is essential today.

Reference: General Medical Council (GMC). Consent guidance: patients and doctors
making decisions together.

7. A 70-year-old woman who requests a bone density scan as there is a maternal


history of hip fracture. She is fit and well herself and there is no history of
fracture. Total hip T score is −3.2 and T score at the lumbar spine is−2.9.

What is the most appropriate treatment in this case?

(Please select 1 option)

Alendronate

Calcium and vitamin D

Hormone replacement therapy

Raloxifene

Strontium ranelate

Alendronate is appropriate for this woman as studies have shown a reduction in


incidence of hip fracture and vertebral fractures.
A well-documented side effect of alendronic acid is peptic ulceration and so this
should be kept in mind when commencing this treatment.

8. A 79-year-old man presents with increased micturition and backache.

On examination, he has a palpable bladder and an enlarged prostate. His serum acid
phosphatase and alkaline phosphatase are both elevated.

What is the most likely diagnosis?

(Please select 1 option)

Carcinoma of the kidney

Carcinoma of the prostate

Hodgkin's lymphoma

Non-Hodgkin's lymphoma
Seminoma

This patient presents with prostate cancer metastases to bone, in particular the spine,
producing osteosclerotic lesions. Prostate specific antigen (PSA) is a prostate-specific
marker but is not cancer-specific.

Alkaline phosphatase and acid phosphatase may also be elevated.

With bony metastases hypercalcaemia should be excluded.

9. Which of the following statements concerning the causation and dynamics of


schizophrenia is correct?

(Please select 1 option)

Heavy cannabis use does not increase the risk of developing schizophrenia

In monozygotic twins the risk of the second twin developing schizophrenia if the first is affected is of the
order of 10%

Schizophrenia is commoner in higher socio-economic groups

Schizophrenia is commoner in individuals not in stable relationships

The lifetime risk of developing schizophrenia if one parent is affected is of the order of 50%

The famous Swedish conscript study suggested a sevenfold increase in schizophrenia risk in
heavy cannabis users1.

Schizophrenia occurs twice as often in unmarried and divorced people as in married or


widowed individuals. Furthermore, people with schizophrenia are eight times more likely to
be in the lowest socioeconomic groups.

These statistics are likely to reflect the alienating effects of this disease rather than any
causal relationship or risk factor associated with poverty or a single life.

Nevertheless, low income and poverty may increase the risk for exposure to biological
factors (for example, infections or toxins) or social stressors that could trigger the illness in
susceptible people.

Monozygotic twins may have a 50% concordance and 10% of offspring may be affected
suggesting strong inheritance.

10. Which of the following correctly indicates the relative strengths of these opioid
analgesics from least potent to most potent?

Of these diamorphine is usually given parenterally whereas the others are more often given
orally.

(Please select 1 option)


Codeine, morphine, oxycodone, diamorphine

Codeine, oxycodone, diamorphine, morphine

Codeine, oxycodone, morphine, diamorphine

Morphine, codeine, diamorphine, oxycodone

Morphine, codeine, oxycodone, diamorphine

It is important to have a good grasp of the relative potencies of opioid analgesics as patients
often have to be converted from one opioid to another and also from one route of
administration to another.

Codeine is the weakest opioid in the list and can be prescribed on its own or more commonly
in combination with paracetamol in co-codamol preparations.

Oxycodone is twice as potent as oral morphine salts for the same dose and when converting
between the two the dose of oral morphine needs to be halved to provide the equivalent
dose of oxycodone.

Diamorphine is typically administered subcutaneously to palliate terminal symptoms as it is


the most potent of the opioids listed. To convert from oral morphine to subcutaneous
diamorphine, the 24 hour oral morphine dose should be divided by 3 to give an approximate
equivalent 24 hour dose of diamorphine. To convert from oral oxycodone to subcutaneous
diamorphine the 24 hour oxycodone dose should be divided by 1.5. Given parenrally
oxycodone 7.5 mg equals 5 mg diamorphine.

11. A patient on clinical examination is found to have a third heart sound.

Which of the following statements is true?

(Please select 1 option)

Is lost in AF

It is accentuated by expiration

It is lost in constrictive pericarditis

It may be a normal finding in women up to the age of 50

It only arises from the left ventricle

The third heart sound is caused by early diastolic filling due to ventricular relaxation, 0.14-
0.16 seconds after closure of the aortic valve (corresponds to Y descent in JVP).
It may arise from either ventricle and is a low-pitched sound accentuated by inspiration.

It is most commonly heard in cardiac failure but is also prominent in constrictive pericarditis
('knock') and with atrial myxomas ('tumour plop').

It is a normal finding in children and young adults and may persist in women up to the age of
50.

S4 corresponds to ventricular filling in atrial systole (a wave in JVP), is never normal and is
lost in AF.

12. A 52-year-old engineer has required repeated admissions to hospital after


suffering a myocardial infarction 12 months previously.

He is on maximum doses of ACE inhibitors, diuretics including spironolactone and


betablockers. He is unable to carry out simple tasks, such as showering and dressing,
without stopping. His blood pressure is 112/60 mmhg and he is in sinus rhythm. His
left venrtricular ejection fraction is less than 20% and there is no major valvular
dysfunction.

What is the definitive management?

(Please select 1 option)

Add eplerenone

Angiotensin receptor antagonist

Digoxin

Dual chamber pacemaker insertion

Heart transplant

Patients with advanced heart failure like this fairly young man, should be considered for
heart transplant.

This is on account of

The history of repeated hospitalisations for heart failure


Maximal medical therapy
End stage heart failure
Poor left venrtricular function, and also,
If his average peak oxygen uptake (VO2max) is less than 14 mL/kg per min.

Absolute contraindications are systemic illnesses which will limit survival despite
transplantation (neoplasia, HIV, multisystem systemic lupus erythematosus [SLE], chronic
obstructive pulmonary disease [COPD] and fixed pulmonary hypertension)
13. A 25-year-old schizophrenic man has required readmission to hospital after
stabbing his mother's hand with a screwdriver. Now he is symptom free.

From the list below choose the most suitable form of psychotherapy you should offer
in this situation.

(Please select 1 option)

Brief psychodynamic psychotherapy

Cognitive behavioural therapy

Counselling

Interpersonal therapy

Systemic desenisitisation

There is evidence that family interventions can reduce relapse rates in schizophrenia.

Brief psychodynamic psychotherapy addresses the patient's difficulties by primarily


relying on insight, bringing unconscious or unclear material into awareness, and linking past
and present experiences.

Cognitive behavioural therapy combines principles of both behavioural and cognitive


therapy, focusing simultaneously on the environment, behaviour, and cognition. It is also:

Structured
Goal directed
Problem focused, and
Time limited.

Interpersonal therapy is derived partially from a psychodynamic perspective and focuses


primarily on the patient's interpersonal relationships. Interpersonal therapy is fairly non-
directive and addresses issues such as:

Grief
Role transitions
Interpersonal role disputes, and
Interpersonal deficits

as they relate to the patient's current symptoms.

Family therapy helps family members learn about the disorder, solve problems, and cope
more constructively with the patient's illness. It may also be useful in this situation.
Systemic desensitisation is exposure-based behavioural treatment which utilises gradual,
systematic, repeated exposure to the feared object or situation to allow patients with anxiety
disorders to become desensitised to the feared stimulus.

14. A 32-year-old woman treated with hydrocortisone 10 mg in the morning and 10 mg


in the evening for Addison's disease, presents to the clinic with poor compliance.
She feels that the hydrocortisone upsets her stomach and wants to switch to
enteric coated prednisolone.

What would be the appropriate corresponding daily dose of prednisolone?

(Please select 1 option)

4 mg daily

5 mg daily

7 mg daily

10 mg daily

15 mg daily

The approximate equivalent glucocorticoid action of prednisolone to hydrocortisone is 4:1.

Hence the equivalent dose for 20 mg of hydrocortisone is roughly 5 mg per day of


prednisolone.

For other glucocorticoid dose conversions try this glucocorticoid dose calculator

15. A 28-year-old man presented with recurrent nose bleeds and iron deficiency
anaemia.

A chest x ray found a shadow over the right lung base and auscultation in this area
revealed a bruit.

Which of the following is the most likely diagnosis?

(Please select 1 option)

Ehlers-Danlos syndrome

Hereditary haemorrhagic telangiectasia

Idiopathic thrombocytopenic purpura

von Willebrand's disease


Wegener's granulomatosis

This is hereditary haemorrhagic telangiectasia (Osler-Weber-Rendu syndrome)


characterised by bleeding from telangiectasia on mucous membranes such as the nose,
mouth and gastrointestinal tract.

Clinical examination reveals telangiectasia on the skin.

Arteriovenous malformation may be seen in the lung (as in this case) or brain.

16. Which of the following statements regarding coarctation of the aorta is correct?

(Please select 1 option)

Is associated with berry aneurysms

Is associated with rib notching all 12 ribs on the left

Is commonly associated with pulmonary stenosis

Is more common in females

Rarely causes problems in paediatric life

Coarctation can cause heart failure in the neonate and hypertension in the adult.

It is associated with

Turner's syndrome
Neurofibromatosis
Bicuspid aortic valve

and more weakly with

Ventricular septal defect (VSD) and


Atrial septal defect (ASD).

Stroke may result from hypertension or from associated berry aneurysms.

As with all left heart obstructions, it is more common in males. Notching of ribs 3-8 is seen.

Notching of ribs 1 and 2 suggests superior vena cava obstruction, a Blalock shunt or
hypertrophied nerves.

17. What is the mechanism of action of glycopeptides (for example, vancomycin)?

(Please select 1 option)


Cell wall inhibition

Inhibition of folic acid metabolism

Inhibition of protein synthesis (transcription)

Inhibition of protein synthesis (translation)

Interference with DNA replication

Glycopeptides inhibit cell wall synthesis through steric hindrance of peptidoglycans,


components of the bacterial cell wall.

Next question

18. A 29-year-old male attends the Emergency department after stubbing his toe on
the end of his bed. He complains of severe pain in his left second toe.

Examination confirms some mild swelling and pain localised over the distal phalanx
of the left second toe. There is no obvious deformity.

What is the most appropriate course of action?

(Please select 1 option)

Arrange a follow-up appointment in the fracture clinic

Reassure the patient and discharge him

Refer the patient to the on-call team for emergency admission

Request an x ray of the patient's foot

Suggest analgesia, padded buddy strapping and elevation

Suspected simple, undisplaced fractures of the four small toes do not routinely require an x
ray or routine admission.

They are best managed with analgesia, padded buddy strapping and elevation.

Patients do not usually require follow up and should be advised to return to full activities as
soon as possible

19. A 64-year-old arteriopath undergoes a PTFE femoro-popliteal bypass graft.The


graft fails and the patient requires a below knee amputation.
Recovery is protracted and on the 20th postoperative day a purulent discharging
sinus develops in the groin at the site of his original surgery.

What is most likely causative organism?

(Please select 1 option)

Bacteroides fragilis

Clostridium tetani

Clostridium welchii/perfringens

Methicillin resistant Staphyloccocus aureus (MRSA)

Staphylococcus aureus

Methicillin resistant Staphylococcal aureus (MRSA) is a common hospital-acquired


pathogen. It affects the skin and soft tissues and may result in sinus tract formation. This
gentleman is an arteriopath and therefore has poor perfusion of his tissues and is at
increased risk of developing infection.

He has had two operations including a failed graft followed by an amputation. The graft is
likely source of colonisation. The postoperative wound discharge he develops 20 days
postoperatively is now a chronic problem. He has unfortunately developed an MRSA
infection of the graft which probably contributed to the initial failure and is now the cause of
the purulent discharging sinus.

When a discharging sinus forms, incision and drainage is required.

MRSA is not susceptible to the beta lactam antibiotics; vancomycin is often used as a first
line, until further sensitivities are confirmed.

Next question

20. A cochlear implant is a device designed to create an alternative hearing pathway


for people of all ages with bilateral, severe to profound sensorineural hearing loss.

Which of the following normal structures of the ear are directly electrically stimulated
by the electrodes in a cochlear implant?

(Please select 1 option)

Auditory nerve ganglion cells

Intracochlear hair cells


Ossicles

Oval window

Round window

A cochlear implant bypasses the mechanical structures of the normal hearing pathway and
provides a direct electrical stimulus to the spiral ganglion cells of the auditory nerve.

Sensorineural hearing loss results from damage to or death of the intracochlear hair cells.
These normally are the transducers of the mechanical energy transmitted to the cochlear
into the electrical impulse required to initiate an action potential in the auditory nerve.

21. A 66-year-old man with insulin-dependent diabetes given ibuprofen for a knee
injury is admitted with palpitations.

His electrocardiogram (ECG) shows a rate of 105 beats per minute, with absent P
waves and tall T waves.

His urea and electrolytes show:

Sodium 132 mmol/L (137-144)

Potassium 6.4 mmol/L (3.5-4.9)

Urea 11 mmol/L (2.5-7.5)

Creatinine 180 µmol/L (60-110)

In this scenario, which of the following is the most appropriate immediate


management?

(Please select 1 option)

Calcium chloride 10 mmol IV

Calcium resonium orally

Dextrose 50 mls 50% with 10 units insulin

Dialysis

Furosemide 1 mg/kg IV
The ECG suggests cardiotoxicity related to hyperkalaemia and the history of palpitations is
suggestive of arrhythmias.

Therefore cardio protection with calcium chloride or gluconate should be first priority and
lowering potassium levels immediately thereafter.

Calcium antagonises the effects of hyperkalaemia on a cellular level by a number of


mechanisms. These all return myocyte excitability to normal thereby reducing the risk of
arrhythmias. Calcium gluconate is the preferred preparation, but calcium chloride can be
used. Caution is needed if the patient is taking digoxin, as hypercalcaemia can potentiate its
toxicity. The effects of intravenous calcium occur within one to three minutes but last for only
30-60 minutes, and therefore more definitive treatment is needed to lower potassium levels.

After calcium is given, treatment is required to shift potassium intracellularly. Insulin is most
commonly used, which stimulates the Na-K ATPase pump. The effect is seen within 10-20
minutes and usually decreases potassium levels by 0.6-1mEq/L. Salbutamol can also
increase the action of the Na-K ATPase pump.

Sodium bicarbonate infusion can shift potassium intracellularly by increasing blood pH, but
its use is controversial. It is therefore reserved for cases of severe acidosis, or where there is
another indication for its use (for example, TCA overdose).

If the above treatments fail, and the cause of hyperkalaemia cannot be treated, then
potassium may need to be removed from the body. The most efficient way to do this is with
haemodialysis. This is only done in resistant cases, or in patients who are already on
haemodialysis. For most patients, treatment with an exchange resin such as sodium
polystyrene sulphonate is more appropriate.

22. A 19-year-old girl presents at the antenatal clinic.


She is approximately six weeks pregnant and the pregnancy was unplanned. She
has a two year history of grand mal epilepsy for which she takes carbamazepine.
She has had no fits for approximately six months. She wants to continue with her
pregnancy if it is safe to do so.
She is worried about the anticonvulsant therapy and its effects on the baby. She
asks how she should be managed.

Which of the following management plans is the most appropriate in this case?

(Please select 1 option)

Advise termination due to drug teratogenicity

Continue with carbamazepine

Stop carbamazepine until the second trimester

Switch therapy to phenytoin

Switch therapy to sodium valproate


The patient and fetus are at far more risk from uncontrolled seizures than from any potential
teratogenic effect of the therapy.

In pregnancy total plasma concentrations of anticonvulsants fall, so the dose may need to be
increased.

The potential teratogenic effects (particularly neural tube defects) of carbamazepine do need
to be explained and in an effort to reduce this risk she should receive folate supplements.

Screening with alpha fetoprotein (AFP) and second trimester ultrasound are required.
Vitamin K should be given to the mother prior to delivery.

There is no point in switching therapies as this could precipitate seizures in an otherwise


stable patient.

Similarly both phenytoin and valproate are associated with teratogenic effects.

23.

What does this ECG show on a 62-year-old male who attends for a BUPA health
check?

(Please select 1 option)

1st degree heart block

Left bundle branch block

Left ventricular hypertrophy

Right bundle branch block

Sinus arrhythmia

This ECG shows a prolonged PR interval of six small squares 0.24 s. The normal PR interval
is 0.12 - 0.2 s.
Causes of first degree heart block include

Increased vagal tone (such as in trained athletes)


Ischaemic heart disease
Rheumatic fever
Hyperkalaemia
Hypokalaemia
Drug therapy such as digoxin or beta-blockers.

A long PR interval on the ECG may also be caused by structural abnormalities such as an
atrial septal defect.

No treatment is usually required.

24. Which of the following features do parathyroid hormone and 1,25(OH)2D3 have in
common?

(Please select 1 option)

Act on osteoclasts

Act on the renal tubules of increase the activity of 1 alpha-hydroxylase

Both act via membrane receptors

Patients with PTH-receptor insensitivity result in increased levels of both

Patients with vitamin D receptor insensitivity result in increased levels of both

This is because these patients tend to have a decrease of levels of calcium resulting in an
increase in PTH, which in itself, results in an increase in 1,25(OH)2D3.

25. A female infant is born at term.

She appears pink except for the extremities which are bluish. Her pulse is 110 bpm
and she is gasping. She has good muscle tone. She responds to nasal stimulation
with airflow by grimacing.

What is her APGAR score?

(Please select 1 option)

6
7 This is the correct answer

The APGAR score was designed by Dr Virginia Apgar in 1952 as a quick scoring system to
be performed at one and five minutes, and can be repeated at 10 minutes.

There are five components:

A - appearance
P - pulse
G - grimace
A - activity
R - respirations.

A low score at one minute may indicate that the baby requires some medical attention. If the
Apgar is still low at five or 10 minutes it can be a poor prognostic indicator suggesting the
potential for neurological damage.

In this case, the Apgar score can be calculated as follows:

Appearance - she is pink but peripherally cyanosed so 1 point


Pulse - above 100 gets her 2 points
Grimace - to catheter gets her 1 point
Activity - movements she gets 2 points
Respiration - gasping gets 1 point.

Therefore, her total is 7 points.

26. Which of the following is the minimum dataset for identifying a patient and a
sample for purpose of a non-emergency blood transfusion?

(Please select 1 option)

The full name and gender

The full name, date of birth and patient identity number

The full name, gender, address and patient identity number

The full name, gender and patient identity number

The full name, gender, previous blood grouping details, address and patient identity number
Given that the largest number of errors and near misses occur because of mislabelling or
mistaken identity of patient or the sample, a minimum data set is prescribed for transfusion
requests.

This involves recording the:

Full name
Date of birth
Patient identity number, and
Address (in some areas, such as Wales).

Previous blood grouping details are not required and missing elements of this minimum
dataset are not acceptable in the transfusion service.

27. A 28-year-old mother is distressed at being told her 6-month-old baby is deaf.
During pregnancy the mother was noted to have pharyngitis, fever and
lymphadenopathy.

What is the most likely infection that would cause the baby's deafness?

(Please select 1 option)

Chorioamnionitis

Cytomegalovirus (CMV)

Group B Streptococcus

Listeriosis

Varicella zoster virus (VZV)

This mother was infected with CMV during pregnancy, with mild symptoms. The fetus has
cytomegalic inclusion disease, histologically identified as viral particles surrounded by
lysosomes.

28. Which vessel lies in the anterior atrioventricular groove of the heart?

(Please select 1 option)

Circumflex artery

Left anterior descending artery

Left main stem

Left marginal artery


Right coronary artery

The RCA runs in the posterior atrioventricular (AV) groove.

The circumflex artery winds its way around the left margin of the heart to sit in the anterior
AV groove, the circumflex supplies blood to the left atrium and side and back of the left
ventricle.

Knowledge of the coronary anatomy, and coronary blood supply is particularly important for
cardiac surgery and cardiology.

It remains a frequently asked examination question.

Reference:

The Cleveland Clinic. The Coronary Arteries.

29. A 48-year-old taxi driver has a Hba1c of 99 mmol/mol and BMI 35 kg/m2. He takes
gliclazide 160 mg twice daily and is intolerant of metformin.

What is the most appropriate hypoglycaemic therapy?

(Please select 1 option)

Acarbose

Glibenclamide

Glimepiride - Amaryl

Orlistat - Xenical

Pioglitazone

Pioglitazone may be used in this case in accordance with NICE prescribing guidelines.
Pioglitazone is contraindicated in heart failure and is associated with bladder tumours, so it
has been withdrawn in some countries.

The gliptins are now another option to add on, sitagliptin, saxagliptin and vildagliptin are
licensed for use as second or third line add on oral antihypoglycaemics.

Conversion to insulin is another option, however this would affect his job as a taxi driver.
30. This is the full blood count of a 63-year-old male who presents with a recent
history of tiredness and indigestion.

Hb 105 g/L

RBC 4.5 ×1012/L

Hct 0.353

MCV 78.6 fL

MCH 23.5 pg

Platelets 325 ×109/L

WBC 7.91 ×109/L

Neutrophils 6.15 ×109/L

Lymphocytes 1.04 ×109/L

Monocytes 0.33 ×109/L

Eosinophils 0.16 ×109/L

Basophils 0.08 ×109/L

Others 0.14 ×109/L

Three months ago he was commenced on aspirin 150 mg per day together with atenolol 50
mg per day for hypertension. He has a strong family history of ischaemic heart disease.
Examination reveals a blood pressure of 155/90 mmHg and a slight tenderness in the
hypochondrium.

What is the most likely explanation for this patient's full blood count and symptoms?

(Please select 1 option)

Achlorhydria

Angiodysplasia

Haemolytic anaemia
Oesophageal varices

Peptic ulceration

This patient has a low haemoglobin concentration on the full blood count accompanied by a
lower end of normal mean cell volume (MCV) suggesting an iron deficiency anaemia. From
the above list, peptic ulceration due to aspirin would be the most likely explanation for this
blood picture and the recent symptoms.

Upper endoscopy would be the investigation of choice to demonstrate gastric or duodenal


lesions and to assess for the presence of H. pylori infection.

31. An 18-year-old with cerebral palsy is admitted after a respiratory


arrest having been intubated by paramedics.

Nobody can gain intravenous access as the patient is too shut down. A femoral line is
not possible due to contractures. You do not have the experience to perform central
venous cannulation.

Which of the following is the best option for administering intravenous


fluids/emergency drugs in this situation of inability to gain venous access?

(Please select 1 option)

Down the endotracheal tube

Intramuscular

Intraosseous

Nasogastric

Subcutaneous

Nasogastric, intramuscular and subcutaneous are too slow and unreliable for emergency
situations (although in cardiac arrest the endotracheal route is recognised). Venous cut
down is a possibility but requires skill in the procedure.

Intraosseous is still perfectly viable in the adult patient 2 cm below the tibial tuberosity on the
antero-medial side or 2 cm proximal to the medial malleolus.
32. A 36-year-old female who is on warfarin after suffering a deep vein thrombosis,
presents with an INR of 8.2 and a conjunctival haemorrhage.

The blood pressure is 125/55 mmHg, heart rate is 65 bpm and the ECG reveals a
normal sinus rhythm.

Which of the following is the most appropriate treatment for this patient?

(Please select 1 option)

Factor VII

FFP

Oral vitamin K 1 mg Correct

Prothrombin complex concentrate

Stop warfarin only

A conjunctival bleed is defined as a minor bleed, and current guidelines suggest that oral or
IV vitamin K together with the omission of warfarin, is the treatment of choice. Local
guidelines should also be available.

One must always weigh up the risks and benefits of reversing the anticoagulation. The
patient described is at low risk if the warfarin-induced coagulopathy is reversed. There is no
suggestion from the ECG and the haemodynamic status that there is pulmonary embolus.

Major bleeds are defined as intraorbital, intracranial, retroperitoneal or muscular bleeding


causing compartment syndrome.

Any acute bleeding with BP less than 90 mmHg, oliguria or Hb drop to less than 90 g/L, also
count as major bleeding episodes and require aggressive reversal of the coagulopathy with
vitamin K, stopping warfarin and prothrombin complex concentrate or fresh frozen plasma
(FFP).

33. A 55-year-old man with a 10 year history of type 2 diabetes is seen by his GP who
has noticed a deterioration in renal function according to a recent U+E.

He has been a poor attender at diabetic annual review with HbA1c of around 10 for
many years, he was reluctant to use insulin and was treated with gliclazide 80 mg a
day. He had had no recent diabetes check ups.
He gave a history of angina on exertion, for which he took an aspirin a day and GTN. He had
been taking quinine at night for many years. On examination he was obese, with bilateral
corneal arcus. There was evidence of proliferative retinopathy and a small haemorrhage in
his right eye.

He was hypertensive with a BP 170/85 mmHg, with no postural drop, heart sounds were
normal. His abdomen was soft with no evidence of bruits. There was evidence of peripheral
oedema. He had some proximal muscle weakness and peripheral loss of vibration and fine
touch. Renal function had deteriorated from urea 10 creatinine 124 to urea 19 creatinine 295
during a two year period, 24 hour protein was 0.9g/l and USS abdomen showed both
kidneys measured at 12 cm.

Which of the following is the likely cause of his renal failure?

(Please select 1 option)

Amyloidosis

Diabetic nephropathy Correct

Glomerulonephritis secondary to malaria

Osteomalacia

Renal artery stenosis

This patient gives a ten year history of Type 2 diabetes with poor follow up and most likely
poor glycaemic control.

He demonstrates many of the complications of uncontrolled diabetes. He has angina - and


has microvascular complications -diabetic retinopathy and neuropathy. He has chronic renal
failure with a raised urea, a raised creatinine, increased urinary protein loss and
derangement of calcium and phosphate levels.

His renal disease is the result of a 'diabetic kidney' which typically presents with initial
microalbuminuria and then progresses to persistent proteinuria.

Diabetic retinopathy is usually associated with diabetic renal disease, as is a normochromic


normocytic anaemia.

Pathological changes range from thickening of the basement membrane to


glomerulosclerosis. Renal osteodystrophy is a complex metabolic disorder.

Glomerular damage leads to phosphate retention and this is accompanied by insufficient


production of 1-hydroxylase with a deficiency in metabolically active 1,25-
dihidroxycholecalciferol.
Deficiency of vitamin D3 decreases absorption of calcium from the gut and secondary
hyperparathyroidism ensues. Parathyroid hormone promotes absorption of calcium from
bone and from the proximal renal tubules.

The end result is a combination of osteomalacia, osteitis fibrosa cystica and osteoporosis.

Ectopic calcification may occur if the solubility of serum calcium and phosphate is exceeded.
This can manifest clinically as limited joint mobility. The serum calcium is low and phosphate
is elevated.

Parathyroid hormone (PTH) will be elevated with reduced vitamin D levels.

34. You are called to the delivery of a 41+4 gestation infant who is being delivered by
emergency caesarian section. Mother is a healthy Caucasian of 27 years, who
smokes 15/d. There have been concerns about fetal growth on serial ultrasound
scans.

Labour commenced 18 hours ago, and mother has been in Stage 2 for the past 90
minutes. There have been three bradycardias with delayed recovery, and the liquor is
thickly meconium stained.

The baby is given to you. She is floppy, pale and covered in thick meconium.

What is the most likely diagnosis?

(Please select 1 option)

Group B Strep septicaemia

Holoprosencephaly

Hypoxic ischaemic encephalopathy

Meconium aspiration

Surfactant deficient lung disease

This child is growth retarded and has had significant fetal distress (birth asphyxia). A cord
gas may help judge the severity of the perinatal insult.

She should be quickly wiped and wrapped in a warm towel, then placed head down on the
resuscitation trolley.

If vigorous no further action is required. If flat, her oropharynx and cords should be inspected
and suction applied to the trachea if there is suspicion that there is some meconium between
them.
She should then be intubated and ventilated. Depending on her response she may require
external cardiac massaage (ECM) or a bolus of fluid, e.g. given via the umbilical vein
catheter (UVC).

Exam Themes 2010

64377

35. A 52-year-old man with a history of acromegaly comes for review.

He has been suffering from pins and needles in his hands during the early hours of
the morning. Tinel's sign is positive.

Which of the following muscles is most likely to be weak?

(Please select 1 option)

Abductor policis brevis

Abductor policis longus

Adductor policis

Flexor digiti minimi

Flexor policis brevis

The answer is abductor policis brevis.

Abductor policis brevis and opponens policis are both innervated by the median nerve.

Flexor policis brevis may be innervated by the median or the ulnar nerve.

The symptoms here are most suggestive of carpal tunnel syndrome, which would fit with the
acromegaly, so abductor policis brevis weakness is possible.

Intervention as early as possible in carpal tunnel syndrome is crucial, as permanent nerve


damage may be present if decompression occurs too late.

36. A 50-year-old man presents with a six week history of progressive dysphagia for
solids.

Endoscopic examination five years earlier showed oesophagitis and Barrett's


oesophagus.
What is the most likely diagnosis?

(Please select 1 option)

Achalasia of the oesophagus

Candidal oesophagitis

Carcinoma of the oesophagus

Globus syndrome

Peptic oesophageal stricture

Barrett's oesophagus is a risk factor for carcinoma of the oesophagus. Clinical features
include dysphagia, weight loss, hoarseness and cough.

Investigations include barium swallow, oesophagoscopy with biopsies and brushings.

37. A 35-year-old schizophrenic man complains of tremor and deterioration of his


mobility. He also has dribbling of saliva which he finds distressing. He is on depot
antipsychotic medication.

What is the most appropriate pharmacological treatment?

(Please select 1 option)

Lithium

Lorazepam

Procyclidine

Propranolol

Thioridazine

This patient has developed drug-induced Parkinson's disease.

Anticholinergic drugs, such as procyclidine, are used to reduce the symptoms of drug-
induced parkinsonism seen with antipsychotic drugs.
38. A 55-year-old lady complains of postmenopausal bleeding. Transvaginal
ultrasound shows endometrial thickness of 10 mm.

What is the most appropriate investigation?

(Please select 1 option)

Colposcopy

Cystoscopy

Hysteroscopy and D & C

Laparoscopy

Laparotomy

Endometriosis is a chronic condition which may cause heavy periods, painful periods, lower
abdominal pain and infertility.

The risk of endometrial pathology is increased when endometrial thickness is >5 mm and
therefore endometrial sampling becomes mandatory in these cases. Infertility is an issue
where the endometrium has grown outside of the uterus and on surrounding organs such as
the ovaries and fallopian tubes.

The cause of endometriosis is not clear, but the symptoms can be managed with analgesics,
hormone therapy and surgery may be indicated where infertlity is an issue.

39. A 66-year-old female with carcinoma of the breast associated with multiple
metastases is receiving palliative care. She develops severe nausea and vomiting
and bone pains.

Investigations reveal a serum calcium concentration of 3.5 mmol/L; she is being


aggressively rehydrated.

What is the most appropriate treatment thereafter?

(Please select 1 option)

Calcitonin

Hydrocortisone IV

Palliative radiotherapy

Pamidronate IV
Zoledronic acid IV

This patient has symptoms of symptomatic hypercalcaemia and so must be treated. Most
hospitals in the NHS may use pamidronate, which is a bisphosphonate and may also be
associated with the relief of bone pain.

However it is well documented that zoledronic acid intravenously has superior outcomes to
pamidronate in reversing hypercalcaemia related to malignancy.

Calcitonin is not a first line drug in treatment of hypercalcaemia, however may be used as a
second line drug in refractory cases.

40. CT scan of a 55-year-old female shows intracerebral haemorrhage.

Which of the following features is not likely to be an aetiological factor?

(Please select 1 option)

Cocaine use

Low cholesterol

Mitral valve prolapse

Moderate alcohol use

Previous history of DVT

Moderate (not excessive!) alcohol use is not associated with increased risk and may even be
protective.

A previous history of DVT may suggest a coagulation disorder or may indicate that the
patient has been warfarinised.

The remaining stems are all associated with increased risk of intracerebral haemorrhage.

41. 4-year-old girl presents with fever, pallor and jaundice. She has had a cold two
weeks previously. She has previously been well.

She had a full term normal delivery with no neonatal complications. Her
immunisations are up to date. There is no FH/SH of note.
On examination she has a fever to 37.9°C, respiratory rate 18/min and heart rate
95/min. She has pale conjunctivae and mildly jaundiced conjunctivae. Chest and ENT
examinations are normal. She has a 2/6 ejection systolic murmur at the upper left
sternal edge. The spleen is 3 cm, smooth and non-tender. Urine dipstix contains 2+
blood. Urine microscopy shows 0 white cells, 0 red cells and no organisms.

What is the most likely diagnosis?

(Please select 1 option)

Haemolytic anaemia, acquired

Haemolytic anaemia, congenital

Infectious mononucleosis

Malaria

Sickle cell anaemia

The picture of antecedent upper respiratory tract infection (URTI) followed by fever, pallor,
mild jaundice and haemaglobinuria all point to haemolysis triggered by infection.

The likely diagnosis is an autoimmune haemolysis due to 'warm' antibodies.

The full blood count may show profound anaemia (for example, 6 g/dl) with spherocytes and
very high reticulocyte count.

Steroids are usually highly effective.

Autoimmune haemolysis associated with 'cold' antibodies may be triggered by Mycoplasma


and Epstein-Barr virus, with symptoms worsened by cold.

42. Abuse of which of the following substances is implicated most frequently as the
cause of occupational accidents and injuries among workers aged 18 to 25?

(Please select 1 option)

Alcohol Correct

Amphetamines

Benzodiazepines

Cocaine

Marijuana
Workers who consume alcohol are almost twice as likely as non-users to experience an
occupational injury.

Researchers found personal and societal costs of moderate and even light drinking are
significant and that most alcohol-related problems in the workplace are caused by light and
moderate drinkers.

Problems are caused not only by on-the-job drinking but also by heavy drinking outside of
work.

Hangover-related health problems include an elevated risk of heart attacks, reduced


cognitive abilities, and a psychiatric disorder (Elpenor syndrome) typified by irrational
behaviour.

43. Which of the following does not cause gastric ulceration?

(Please select 1 option)

Aledronate

Celecoxib

Chronic renal failure

Crohn's disease

Misoprostol

Misoprostol is a prostaglandin analogue used to prevent gastric ulceration.

Celecoxib, a selective COX-2 inhibitor increases the risk of ulceration, although less so than
non-steroidal anti-inflammatory drugs (NSAIDs).

Renal failure reduces the breakdown of gastrin, resulting in increased gastric acid synthesis
and ulceration.

44. A 55-year-old gentleman is admitted with central colicky abdominal pain. The pain
has been present for six days and is getting worse. He has also noticed the
passage of watery diarrhoea for three days prior to presentation.

Past medical history includes angina and peripheral vascular disease and he is a
current smoker. Examination reveals generalised tenderness which is most marked
over the left colon.

An abdominal x ray study is performed:


What is the likely diagnosis?

(Please select 1 option)

Colonic carcinoma

Crohn's disease

Infective colitis

Ischaemic colitis

Ulcerative colitis

The abdominal x ray shows evidence of mucosal oedema and thumb printing typical of
ischaemic colitis.

Given the history of arterial disease (cardiac and peripheral) it is likely that the pathology is a
chronic progressive atherosclerotic lesion at the origin of the superior mesenteric artery.

The patient requires investigation to confirm the diagnosis and management may require
revascularisation of the ischaemic bowel.
45. 15-year-old boy presents with acute left testicular pain. He is not sexually active.

On examination the scrotum appears normal but he has a tender, swollen left testis.
The right testis appears normal.

What is the most likely diagnosis?

(Please select 1 option)

Acute epididymitis

Mumps orchitis

Ruptured epididymal cyst

Testicular neoplasm

Testicular torsion

The features of acute testicular pain suggest testicular torsion and should prompt surgical
referral. This is a clinical diagnosis. There may only be a few hours before irreversible
ischaemia occurs. Investigations, such as ultrasound, may be unhelpful, unavailable or
cause delay.

When in doubt explore (and fix the other side too if it is a torsion). It is better to explore and
be wrong than the other way round.

Torsion: acute pain and swelling of testis, with absent cremasteric reflex.

Epididimitis: acute pain and swelling. Rare before puberty, and commoner in sexually active

46. 25-year-old man presents with a lump in the lower neck he noticed on casual
palpation. There is no pain associated with the lump. He reports tingling along the
inner aspect of his forearm.

The lump is situated in the supraclavicular fossa. It is hard, rounded and seems to be
attached to the seventh cervical vertebra.

What is the most likely diagnosis?

(Please select 1 option)

Cervical rib

Goitre

Jugular glomus tumour

Laryngomalacia
Subclavian artery aneurysm

Neck swellings are usually classified on their location into:

Anterior triangle swellings (anterior to the sternomastoid muscle), and


Posterior triangle swellings (posterior to sternocleidomastoid).

Typically, swellings in the posterior triangle are lymph nodes.

There are more diverse origins for anterior triangle swellings. Common differentials are
lymph nodes and thyroid swellings.

Cervical rib is an extra rib, normally placed just above the normal first rib. The cervical rib
may be fully or partially formed. It is usually asymptomatic, however if it causes
neurovascular compromise by pressing on any nearby structures, this may result in
symptoms of thoracic outlet syndrome.

Lymph node swellings are generally found along veins. They are divided into six levels
mainly to stage metastatic disease and standardise surgical procedures by the digastric
muscle, sternomastoid muscle and hyoid bone.

In the above questions, history, the anatomical situation and examination findings point to
the answers.

47. Which statistical term can be defined as the proportion of patients who do not
have the disease that are correctly identified as negative?

(Please select 1 option)

False negative

False positive

Regression

Sensitivity

Specificity

Sensitivity relates to the test's ability to identify positive results. Where a test has a high
sensitivity it is has few false negatives.

Specificity relates to the ability of the test correctly to identify negative results. A test with
high specificity has few false positives. If a test with high specificity is positive, that person is
likely to have the disease because of the low false positive rate.

Therefore, in order to detect genuine positives (to indicate disease), we need a test with high
specificity rather than a high sensitivity.
48. A 76-year-old man complains of painful microscopic haematuria and urgency.

On examination of the abdomen there is suprapubic tenderness but no palpable


abnormality. He is apyrexial, inflammatory markers and PSA in the blood are normal.

What is the most likely diagnosis?

(Please select 1 option)

Acute bacterial prostatitis

Acute cystitis

Benign prostatic hypertrophy

Bladder calculi

Bladder cancer

Painful haematuria suggests trauma, infection or calculi, whereas painless haematuria


suggests a possible occult malignancy.

This man is apyrexial with normal WCC and CRP which should effectively exclude infection
as a cause for his symptoms.

There is no history of trauma in this scenario (often catheter-associated) which makes this
cause unlikely. It is worth noting that haematuria associated with injury tends to be
macroscopic.

Therefore, bladder calculi are the most likely source of his symptoms. Imaging will help to
determine the presence of calculi.

49. A boy is born weighing 2.7 kg at 39+2/40. His mother has had two previous
children with early jaundice.

He is noted to be pale and jaundiced. Haemoglobin concentration 60 g/L, reticulocytes


10%. Film shows polychromasia, nucleated red cells. Unconjugated
hyperbilirubinaemia of 175 µmol/L (0-18). Direct Coombs +ve. Mother A-ve, baby
O+ve.

What is the most likely diagnosis?

(Please select 1 option)

ABO incompatibility

Alloimmune haemolytic anaemia

Drug-related haemolysis
Hereditary spherocytosis

Rhesus disease

Early jaundice suggests haemolysis, and the previous affected babies suggest
isoimmunisation.

The positive Coombs' test and fact that mother is D rhesus negative make rhesus disease
most likely. This is confirmed by testing specifically for serum anti-D, which is positive.

Reticulocytes contain residual RNA, and polychromasia is due to increased blue staining
due to the very high reticulocyte count.

50. A 15-year-old girl presents with a seven day history of fever and malaise, with
tender neck glands and discomfort in swallowing. She has previously been well.
She had a full term normal delivery with no neonatal problems. Her immunisations
are up to date. There is no family or social history of note.

On examination the temperature is 38.5°C, respiratory rate 15/min and pulse 80/min.
She appears well and is well perfused. She has a scanty blanching erythematous rash
over the trunk, tender cervical and inguinal glands, and a spleen, palpable 4 cm below
the costal margin.

What is the most likely diagnosis?

(Please select 1 option)

HIV

Infectious mononucleosis Correct

Kawasaki disease

Mycobacterial adenitis

Streptococcal pharyngitis

The history is of a mononucleosis-like illness with:

fever
malaise
dysphagia
lymphadenopathy, and
splenomegaly.

A full blood count and film may reveal atypical lymphocytes, and a monospot may be
positive in Epstein-Barr virus (EBV) infection.

Other causes of this presentation includeCytomegalovirus (CMV) infection and


toxoplasmosis.

Treatment is symptomatic. Contact sports should be avoided until the spleen has regressed
to normal.

51. A 45-year-old, overweight shop assistant is admitted to hospital with sudden


onset of breathlessness and right posterior lower chest pain three weeks after
undergoing right total hip replacement surgery.

She has a past medical history of bronchiectasis and asthma, but denies any recent
change in sputum colour or quantity.

Oxygen saturation is 89% on air, but rises to 95% on (35%) oxygen. She is apyrexial.
Chest examination reveals coarse leathery crackles at both lung bases.

Peak flow rate is 350 L/min and chest radiograph shows bronchiectatic changes, also
at both lung bases. Full blood count is normal.

What is the most appropriate investigation to conduct next?

(Please select 1 option)

Blood cultures

CT-pulmonary angiography

Echocardiogram

Full blood count

Ventilation/perfusion scan

Computerised tomography (CT) pulmonary angiography is the investigation of choice in this


patient who almost certainly has a pulmonary embolism.

Ventilation perfusion scan is not ideal in a patient with chronic lung disease, because the
scan can be difficult to interpret.
52. A 45-year-old woman has a fungating metastatic breast cancer and is increasingly
distressed by the malodorous discharge from the affected breast, which is
causing considerable social embarrassment.

From the list below, choose the single treatment which would help alleviate this
symptom.

(Please select 1 option)

Allevyn dressings

Dry dressings

Oral metronidazole or gel

Oral naproxen or gel

Topical Eumovate

Metronidazole may improve smells emanating from anaerobic organisms that infect
fungating tumours.

Charcoal dressings are also of some benefit; these absorb malodorous substances.

The British National Formulary (BNF) contains an excellent section giving guidance on
prescribing in palliative care and we would recommend that you familiarise yourself with the
introductory sections of the BNF (which contains information that is often tested in exams)

53. A 56-year-old woman diagnosed recently with type 2 diabetes mellitus presents
with a progressive decline of vision in one of her eyes.

On fundoscopy macula oedema is noted.

What is the most likely diagnosis?

(Please select 1 option)

Cataracts

Chloroquine toxicity
Diabetic retinopathy

Hypertensive retinopathy

Macula degeneration

Disease of the macula serious enough to affect central vision can accompany any stage of
diabetic retinopathy including background, and may be present in newly diagnosed type 2
diabetes patients.

Macular oedema is due to extravasation of plasma proteins across abnormally leaky


capillaries.

It may cause only retinal thickening which may be undetectable by routine fundoscopy, even
when advanced enough to reduce visual acuity.

Exudates, often circinate, and spotty 'cystoid' changes may occur.

54. Gentamicin (7 mg/kg once daily) is administered to an elderly patient with severe
urosepsis.

Twenty-four hours post-administration gentamicin levels are more than 2 mg/L.

Which of the following complications is this patient at most risk of?

(Please select 1 option)

Encephalopathy

Hepatotoxicity

Nephrotoxicity

Peripheral neuropathy

Retinopathy

Prolonged exposure to gentamicin is likely to cause nephrotoxicity, particularly in elderly


patients with renal impairment. This is more likely to develop if gentamicin accumulates over
a course of treatment, and is usually due to acute tubular necrosis. It is therefore important
to dose gentamicin according to body weight, and monitor levels, the frequency of which
depends on which dosing schedule is being used. Extra caution is needed in elderly patients
who are at a very high risk of acute renal failure.

In this situation the next dose of gentamicin should not be given if the gentamicin trough
exceeds 2 mg/L.
Gentamicin is also a vestibulotoxin, which can result in permanent loss of equilibrioception.
This usually develops if gentamicin is taken at high doses for prolonged periods, although
there are case reports where this developed within three to five days. This side effect can be
used for therapeutic benefit in severe Meniere's disease.

Hepatotoxicity, peripheral neuropathy and retinopathy are not classically associated with
gentamicin.

55. Which one of the following is in keeping with a diagnosis of myeloma induced
hypercalcaemia?

(Please select 1 option)

Acute hepatic failure

Colitis

Polyuria and polydipsia

Prolonged Q-T interval on ECG

Tetany

Hypercalcaemia causes acute renal failure, a shortened Q-T interval on ECG as well as
polyuria and polydipsia.

The abdominal symptoms include:

constipation
nausea and vomiting
peptic ulceration (due to increased gastrin secretion)
abdominal pain, and
pancreatitis.

Tetany is caused by hypocalcaemia.

56. A 1-day-old baby boy born at term is noted to be developing increasing jaundice.

What is the most probable diagnosis?

(Please select 1 option)

Breast milk jaundice


Congenital rubella

Formula feed

Galactosaemia

Rhesus incompatibility

This 1-day-old is most likely to have rhesus incompatibility. Rhesus incompatibility causes
antibodies against the fetal red blood cells, which subsequently lyse and release bilirubin.
This is perceived as jaundice.

Breast milk jaundice and galactosaemia are metabolic causes which would require time to
present.

Formula feed is not noted to produce jaundice in a newborn.

Congenital rubella is not noted to cause jaundice.

57. A 21-year-old non-pregnant woman complains of an 18 hour history of urgency of


micturition, suprapubic pain and malaise.

What is the most likely diagnosis?

(Please select 1 option)

Acute cystitis

Acute pyelonephritis

Acute urethritis

Acute vaginitis

Renal colic

Acute cystitis is fairly common in women; dehydration, constipation and faecal contamination
at the vaginal introitus are common underlying causes.

An episode of cystitis in an otherwise healthy and non-pregnant woman is classed as


uncomplicated. Complicated cases include pregnancy, underlying causes of
immunocompromise and recurrent infection.

Pyelonephritis can be caused by seeding of the kidneys from bacteraemia. Patients usually
present with high temperatures (>38°C), chills, rigors and flank tenderness. A common
organism implicated is Escherichia coli. Intravenous antibiotics are generally required to
clear the infection.
Acute vaginitis may be a differential diagnosis in a woman with symptoms of dysuria,
however the presence of vaginal discharge/odour, pruritus, and dyspareunia (usually without
urgency of micturition) should raise the suspicion clinically of this being cystitis. Common
organisms include yeast infection,Trichomonas, and bacterial vaginosis.

Acute urethritis commonly presents as dysuria and may also demonstrate pyuria. It can be
caused by Neisseria gonorrhoeae (called gonococcal urethritis) or by another organisim
(called non-gonococcal urethritis). Other causative organisms include:

Chlamydia trachomatis
Candida species
Mycoplasma genitalium, and
Herpes simplex virus.

Acute urethritis is something that should be excluded in sexually active women.

Renal stones present quite differently, but are often confused with pyelonephritis. The signs
to look for are blood or protein in the urine with an absence of leucocytes and nitrates.
Symptoms are typically severe pain, colicky in nature and travelling down in a loin to groin
direction. CT kidneys, ureters, and bladder will help to image for calculi.

58.

This 69-year-old male presents to you complaining of deteriorating dyspnoea. He has


a past history of stable angina and is a smoker of 10 cigarettes per day. His chest x
ray is shown.
What is the diagnosis?

(Please select 1 option)

Asthma

Bronchiectasis

COPD

Pneumonia

Pulmonary oedema

This patient has heart failure.

The chest x ray reveals Kerley B lines (which are due to interstitial fluid), bat's wing shadow,
and cardiomegaly, indicating left ventricular failure.

59. A young Asian man presents with a two month history of fever, night sweats,
weight loss and haemoptysis.

On examination, chest x ray shows cavitating lesion in the right upper zone. He is an
office worker and owns two parrots.

What is most likely causative organism?

(Please select 1 option)

Bacillus anthracis

Chlamydia psittaci

Legionella pneumophila

Mycobacterium tuberculosis

Pneumocystis jiroveci

Night sweats, fever, weight loss and cavitating lesions are suggestive of TB, particularly in a
patient who may have travelled to a country where TB is endemic, or come into contact with
an infected person.

Even though this patient has come into contact with birds, his symptoms are not consistent
for infection with Chlamydia psittaci. Psittacosis presents with flu-like symptoms which then
progress to a severe pneumonia.
60. A 65-year-old retired accountant presents to the casualty department after
returning from an overseas holiday with a blood pressure of 230/110 mmHg. He
complains of nose bleeds of two hours duration and dull headaches.

His antihypertensives are reinitiated, but anterior nasal packing is unsuccessful.

What is the most suitable treatment option?

(Please select 1 option)

Cautery

FFP

Ice pack on the bridge of nose and pinching of the nose

Post nasal packing

Vitamin K

Epistaxis occurs in two different sites: anterior and posterior.

Anterior nosebleeds often result from mucosal trauma or irritation. Anterior nosebleeds are
by far the most common among otherwise healthy patients. A large proportion are self-
limited and can be managed definitively in the primary care setting.

Proper tamponade involving grasping the alae distally so all mucosal surfaces are opposed
and application of a cold compress applied to the bridge of the nose are manoeuvres that
can control simple bleeds in the home setting but should also be taught to high-risk patients
for use at home.

Cautery, either chemically or electrically is the first line management of a small visualised
bleeding vessel of an anterior source.

The next step in management of suspected anterior bleeding if cautery and local injection
are unsuccessful is nasal packing to tamponade local bleeding. If anterior packing is
unsuccessful then posterior packing with a Foley 14G catheter usually combined with
anterior packing is used.

Anticoagulated patients are at high risk of epistaxis. Anticoagulant reversal is sometimes


required. Prothrombin complex concentrate is used in the setting of active bleeding, along
with vitamin K.

61. A 65-year-old gentleman with a prosthetic heart valve presents with epistaxis.
He takes anticoagulation medications but does not remember if he has taken his
tablets correctly. He has not seen his general practitioner or the anticoagulation nurse
for the last three months.

What is the most likely cause of epistaxis in this patient?

(Please select 1 option)

Anticoagulant overdose

Coagulopathy

Haemolysis

Septal perforation

Trauma

Epistaxis, or nasal bleed, is a very common condition. Causes for epistaxis can be classified
into:

Traumatic - includes fractures of the nasal bones, maxilla, and floor of the cranial cavity
or soft tissue injury around the nasal cavity
Neoplastic - includes both benign (angiofibroma of nasal cavity) and malignant (maxillary
or other paranasal sinus carcinoma, Kaposi sarcoma)
Infective - rarely causes epistaxis, however severely congested conchae could bleed with
very trivial trauma or conditions like orf
Systemic - a variety of systemic illnesses could give rise to nasal bleed; the common
ones for a spontaneous epistaxis are hypertension, bleeding diathesis including
haematological malignancies
Miscellaneous - includes medications such as anticoagulants.

Nose picking is the most common cause of epistaxis in children.

Nasal polyps are a common cause of epistaxis but these are generally produced by mucosal
hypertrophy rather than true hyperplasia and so are not strictly neoplastic.

62. A case-control study was designed to investigate the link between a low fat diet
and incidence of bowel cancer. One hundred cancer patients were matched to 100
healthy controls.

Of the cancer patients 42 were exposed to a high fat diet. Of the healthy individuals 22
had been exposed to high fat diet.
What is the clinical effectiveness of eating a low fat diet?

(Please select 1 option)

0.61

0.69

0.84

1.01

1.17

Odds ratio = (58 / 78) / (42 / 22) = 0.39

Clinical effectiveness = 1 − odds ratio = 0.61

Relative risk can be calculated for a cohort study.

Odd ratio is useful for a case-control study.

63. A deficiency in which of the following enzymes causes hypokalaemia, increased


plasma ACTH, increased urinary excretion of 17-hydroxycorticosteroids and 17-
ketosteroids?

(Please select 1 option)

3 beta – hydroxysteroid dehydrogenase

11 beta – hydroxylase

17 beta -hydroxylase

21-hydroxylase

Side-chain cleavage enzyme of 11 beta – hydroxylase

11 Beta hydroxylase deficiency in congenital adrenal hyperplasia results in a loss of cortisol,


hence due to feedback mechanisims there is an increase in adrenocorticotropic hormone
(ACTH); there is an increased production of 11-deoxycorticosterone, which is a very potent
mineralocorticoid leading to hypokalaemia.
64. A 10-month-old Afro-Caribbean boy presents with painful swelling of the hands
and feet. This has worsened over the past six hours, making him distressed and
weepy. Full term normal delivery born in Africa, no neonatal problems, moved to
the UK age 4 months. Immunisations up to date. Mother is known to have sickle
trait.

On examination he is apyrexial, with respiratory rate of 30/min and pulse of 105/min.


There is symmetrical swelling of the hands and feet, which are painful to touch. His
spleen is 3 cm below the costal margin.

What is the most likely diagnosis?

(Please select 1 option)

Haemophilia A

Kawasaki disease

Reactive arthritis

Septic arthritis

Sickle cell crisis

The history of symmetrical painful swelling of hands and feet in an Afro-Caribbean child
suggests sickle cell dactylitis ('Hand and foot syndrome'), which is often the first
manifestation of sickle cell disease. It is due to ischaemic necrosis of the long bones as the
bone marrow expands.

Hemoglobin electrophoresis confirms the diagnosis. Oxygen and analgesia should be given.

65. Which of the following is least likely to be associated with Fabry's disease?

(Please select 1 option)

Angiokeratoma corporis diffusum

Disease is more common in men than women

Inherited in an autosomal dominant fashion

Mutations in the gene for alpha-galactosidase A


Pain is the most debilitating symptom in childhood

Fabry's disease, is an X linked lysosomal-storage disorder.

This disorder is due to a deficiency of a lipid breakdown enzyme known as


ceramidetrihexosidase, also called alpha-galactosidase A. Its function is to cleave to a
molecule of galactose from a lipid that arises primarily from old red blood cells. In its
absence galactose remains attached to ceramide.

The resulting accumulation of globotriaosyl ceramide leads to renal, corneal, and apocrine
gland dysfunction.

It causes

Angiokeratoma corporis diffusum


Painful vascular-induced peripheral sensory neuropathy and
Acroparesthaesia.

Pain is the most debilitating symptom, and crises consist of agonising burning pain in the
hands, feet, and proximal extremities lasting for minutes to days.

It is treated by renal transplantation and/or enzyme infusion rather like Gaucher's disease.

66. A 52-year-old male on treatment for depression develops blurred vision and dry
mouth.

With which antipsychotic would you associate these side effects?

(Please select 1 option)

Amitriptyline

Chlorpromazine

Fluoxetine

Lithium

Sertindole

Amitriptyline and other tricyclic antidepressants have anticholinergic side effects (dry mouth,
urinary retention and dry skin).

Next question
67. Which vessel opens into the right atrium of the heart?

(Please select 1 option)

Coronary sinus

Great cardiac vein

Posterior descending artery

Right coronary artery

Right marginal artery

The coronary sinus collects deoxygenated blood from around the myocardium and delivers it
to the right atrium. The superior and inferior venae cavae also open into the right atrium

The coronary sinus is actually a collection of veins, including the great cardiac vein, which
ascends from the apex and opens into the coronary sinus.

Knowledge of the coronary anatomy, and coronary blood supply is particularly important for
cardiac surgery and cardiology.

It remains a frequently asked examination question.

68. A 70-year-old lady presents with a two month history of a swelling in the front of
the neck. There has been a rapid increase in the size of the lump and she has
become hoarse.

On examination she is noted to have an audible wheeze and there is a large hard
mass in the lower anterior part of the neck.

What is the most likely diagnosis?

(Please select 1 option)

Anaplastic carcinoma of thyroid

Branchial cyst

Pharyngeal pouch

Riedel's thyroiditis

Thyroglossal cyst
This is a very short history of two months. The rapid increase in size suggests malignancy.
The other feature which is compatible with this is the hoarse voice due to involvement of the
recurrent laryngeal nerve.

The audible wheeze will also be due to pressure on the trachea and paralysis of the
recurrent laryngeal nerve will also have effects on the vocal chord. The hard mass in the
lower anterior part of the neck is suggestive of anaplastic carcinoma.

Riedel's thyroiditis would also produce a hard, irregular swelling of the thyroid gland, but
have a longer histor

69. Which statistical term can be described as a type 2 error?

(Please select 1 option)

False negative

False positive

Null hypothesis

Power

Regression

A type 1 error is a false positive and a type 2 error is a false negative.

70. Which of the following antibiotics are required during the induction of anaesthesia
in an emergency bowel procedure?

(Please select 1 option)

Amoxicillin

Ceftriaxone

Gentamicin

Linezolid

Metronidazole

Antibiotic prophylaxis is used to prevent infection and is based on the degree of


contamination involved in the surgical procedure.

Breast, thyroid and hernia repair operations, where there is minimal risk of wound
contamination, are classed as clean surgery. Antibiotic prophylaxis is controversial in this
group. Most surgeons choose no antibiotics at all or one single shot of broad spectrum
antibiotic at induction.

Operations such as cholecystectomy, elective/interval appendicectomy or bowel resection,


where the contamination of the wound can be contained/controlled, are classed as clean
contaminated wounds and require broad spectrum antibiotics given at induction and every
six hours intraoperatively if the surgery is prolonged. The use of postoperative antibiotics is
debatable in these situations.

Emergency operations or those where the amount of contamination is difficult to contain or


estimate (including traumatic wounds) are classed as contaminated and are shown to
benefit from 72 hours of intravenous broad spectrum antibiotics. Uncomplicated, early
appendicitis generally only requires anaerobic cover with induction.

Grossly contaminated wounds before the start of surgery are classed as dirty wounds and
need therapeutic antibiotics as opposed to prophylaxis - the choice of antibiotic being
empirical as determined by the source of contamination.

Certain special conditions need specific antibiotic prophylaxis, as otherwise they could result
in severe infections anywhere in the body. These include immune compromised states
(including splenectomy and steroid therapy).

As outlined in the NICE guidance on Prophylaxis against infective endocarditis (CG64),


routine antibiotic prophylaxis is no longer recommended for dental procedures or routine
surgery for patients with congenital or acquired heart disease.

71. An 8-year-old boy presents with epistaxis. He has multiple bruises which
according to the parents result from seemingly trivial injuries. His cousin, who has
a similar problem, receives steroids.

On examination, he is also found to have splenomegaly.

What is the most appropriate diagnosis?

(Please select 1 option)

Drug induced

Epstein-Barr virus

Idiopathic thrombocytopenic purpura (ITP)

Maxillary adenocarcinoma

Sepsis

Idiopathic thrombocytopenia purpura (ITP) of childhood is defined by a platelet count of less


than 100,000 with no other haematological abnormalities. Approximately 15% of children
presenting with ITP develop chronic ITP, which by definition lasts for more than 12 months
from date of onset.

Management of chronic ITP largely depends on the extent of symptoms and triggers.
Infection may trigger an episode where the platelets drop to clinically significant levels and
so steroid therapy may be indicated. Other indicators for pharmacological intervention
include heavy menstrual periods, lifestyle including sports which result in injury and bleeding,
number of episodes of bleeding, etc.

The mainstay of treatment is support and avoidance of triggers, antiplatelets, NSAIDS and
monitoring of platelet levels. Steroids, platelet transfusions and other medications are only
used when indicated and after discussion with a haematologist.

In chronic ITP the spleen is usually enlarged due to rapid turnover of platelets. Splenectomy
is appropriate is a small percentage of patients, after considering the risk and benefits in the
long term.

72. A 25-year old lady presents to her GP with a two day history of bilateral forehead
pain. She recently had an upper respiratory tract infection.

On examination the forehead is tender to touch.

What is the most likely diagnosis?

(Please select 1 option)

Acute ethmoiditis

Acute frontal sinusitis

Guillain-Barré syndrome

Juvenile angiofibroma

Meningitis

This lady has acute frontal sinusitis secondary to an upper respiratory tract infection.

This is a worrying condition because infection can easily spread through the posterior wall
into the anterior cranial fossa resulting in abscess formation.

CT head and treatment with antibiotics is warranted

73. A 23-year-old unmarried woman is referred because of recurrent abdominal pain,


she has had a previous laparotomy.

She is also known to the psychiatric services because of recurrent mood swings.

Her only drug history is that she takes the oral contraceptive.
Which of the following tests would you recommend?

(Please select 1 option)

Serum bilirubin

Serum folate

Serum phosphokinase

Urinary methaemoglobin

Urinary porphobilinogen

The diagosis to be excluded here is acute intermittent porphyria. This is characterised by


raised delta aminolevulinic acid and porphobilinogen in the urine.

Other porphyrias are associated with skin manifestations which are clearly not present in this
woman.

74. A 15-month-old boy presents with fever.

He has been off colour for three days, and has not been eating solids. His temperature
has not settled with paracetamol.

He had a full term normal delivery, and no neonatal problems. His immunisations are
up to date. No family or social history of note.

On examination, he is alert though miserable with a temperature of 38.2°C, respiratory


rate 30/min, no recession and pulse 120/min, well perfused. ENT examination and
abdominal examination are normal. Bag urine is negative to dipstix testing.

Full blood count shows a white cell count of 13.1 ×109/L (45% lymphocytes, 35%
neutrophils) and his C reactive protein is 25 mg/L (less than 5).

What is the most likely diagnosis?

(Please select 1 option)

Kawasaki disease

Meningococcal septicaemia

Non-specific viral infection

Pneumococcal bacteraemia
Upper respiratory infection

The fever and mild constitutional upset suggest a viral illness.

It is important to exclude a urinary tract infection (UTI), as has been done in this case.

In view of the age the full blood count (FBC) and C-reactive protein (CRP) are not
unreasonable to do.

Again, they suggest a viral illness as neither is greatly elevated.

75. A 36-year-old woman attends her GP's surgery. She has been diagnosed with
hypothyroidism recently and takes thyroxine 100 micrograms daily.

The GP has the benefit of thyroid function and other tests from the previous week.

Which test is the best for monitoring progress and treatment?

(Please select 1 option)

Free thyroxine levels

Protein bound iodine levels

Thyroid peroxidase antibody levels

Thyroid stimulating hormone (TSH) levels

Triiodothyronine levels

Thyroxine suppresses the high TSH levels noted in hypothyroidism.

TSH is the best monitoring test and one should aim to get the TSH into the normal range.

Triiodothyronine, free thyroxine (T4), thyroid peroxidase antibody and protein bound iodine
levels are used in initial investigation and diagnosis.

76. A 39-year-old man with Type 1 diabetes has undergone cadaveric renal transplant
for end-stage renal failure.

He is maintained on insulin, prednisolone, azathioprine, cyclosporin and septrin. Ten


weeks after the transplant, he presents with dyspnoea, fever and fatigue. On
examination he has bilateral lung crackles and hepatosplenomegaly.

Investigations:

Hb 89 g/L (130-180)
WCC 1.2 ×109/L (4-11)

Platelets 410 ×109/L (150-400)

Sodium 131 mmol/L (137-144)

Potassium 5.2 mmol/L 3.5-4.9)

Urea 31 mmol/L (2.5-7.5)

Creatinine 321 µmol/L 60-110)

LFTs Normal -

Blood cultures Negative -

48 hours CXR bilateral diffuse interstitial shadowing bone marrow biopsy-normal myeloid
erythroid maturation, negative for AAFB and fungi. Bronchoscopy with transbronchial biopsy
showed no histological abnormality and was negative for AAFB and PCP.

An open lung biopsy showed fibrinous pneumonia with obstructive bronchiolitis associated
with a dense cellular infiltrate of highly atypical lymphoid cells containing pleomorphic nuclei.

What is the most likely diagnosis?

Bacterial pneumonia

CMV

EBV-induced lymphoproliferative disease

Fungal infection

Reactivation of TB

Approximately 1-10% of transplants are complicated by EBV-induced lymphoproliferative


disease. Some immunosuppressants are associated with EBV reactivation especially
cyclosporin, antithymocyte globulin and monoclonal anti-T cell antibodies. T-cell-mediated
immunity is of paramount importance in the killing of EBV-infected B-cells and thus in the
control of EBV-infections.

When T-cell immunity is compromised, these EBV-infected cells proliferate and infiltrate
lymph nodes and multiple organs. Patients present with fever, adenopathy and features
resembling disseminated lymphoma.

Pathology shows B-cell hyperplasia and often polyclonal and monoclonal B-cell lymphomas.
77. A 25-year-old woman is admitted with abrupt onset of a severe occipital headache
whilst decorating at home. This was associated with vomiting and transient loss of
consciousness.

On examination her Glasgow coma scale (GCS) score is 15, she is afebrile, has a
blood pressure of 146/84 mmHg and a pulse rate of 70 beats/minute. Her physical
examination is normal apart from an abrasion over her right temple.

What is the single most appropriate initial management?

(Please select 1 option)

Admit for head injury observation

Carboxyhaemoglobin level

Computed tomography (CT) brain scan

Lumbar puncture

Skull x ray

The history of sudden onset of severe headache in a young woman is suggestive of


subarachnoid haemorrhage. The associated vomiting and loss of consciousness add weight
to this.

The investigation of choice is a CT scan of the brain looking for subarachnoid blood.

If this is normal a lumbar puncture should be performed as a CT scan will miss


approximately 10% of cases of subarachnoid haemorrhage.

78. A 51-year-old woman attends her GP concerned about her risk of osteoporosis.

She had a hysterectomy and oophorectomy because of uterine fibroids one year ago,
followed by mild hot flushes which have since resolved. Recently her mother broke her hip at
the age of 72, and the patient is worried about the possibility of fracture. She asks about
medications for osteoporosis.

Her body mass index is 17.3 kg/m2 and the subsequent T score is <−2.5. She received
Depo-Provera from the age of 39 to 45, during which time she was amenorrhoeic. Breast
examination and the remainder of the physical examination are normal.

Which of the following would you recommend to her?


Bisphosphonate

Calcium carbonate alone

No treatment

Oestrogen replacement therapy

Vitamin D alone

This woman has a number of risk factors for osteoporosis including low BMI, a positive
family history and has undergone oophorectomy, albeit at a rather appropriate menopausal
age.

She no longer has menopausal symptoms.

Osteoporosis is defined as a T score of <−2.5 SD and would denote severe osteoporosis in


the presence of one or more fragility fractures

Therefore, the most appropriate therapy in this patient's case would be a bisphosphonate.

79. An 82-year-old gentleman with a long history of dementia and advanced oesophageal
carcinoma is admitted from a nursing home as an emergency with sudden onset of total
dysphagia. He is unable to tolerate solids or liquids which he immediately regurgitates.

He had been stented two weeks previously and had initially had excellent symptomatic relief.
Examination is unremarkable.

What is the likely diagnosis?

(Please select 1 option)

Bolus obstruction

Globus hystericus

Reflux oesophagitis

Stent displacement

Tumour overgrowth

Given the history, bolus obstruction due to inadvertent ingestion of a large piece of food is
most likely.
Despite issuing detailed instructions regarding diet following stenting, readmission with
obstruction is common, due to patients 'forgetting' the rules.

Stent displacement tends to occur early with metal stents before they are fully deployed and
tumour overgrowth is a longer term complication.

80. You are asked to provide the first part of a cremation form.

Which of the following would need to be removed prior to cremation?

Breast implants

Cochlear implant

Hip replacement

Pacemaker

Ventricular shunt

The answer is pacemaker.

Guidance for medical practitioners on the Ministry of Justice website states that some
electrical or radioactive implants cause a serious health and safety risk at the crematorium
and must be removed before cremation.

These are:

Bone growth stimulators


Brachytherapy to the prostate, that is, radioactive iodine-125 seeds (advice from the
Department of Radiology and Urology at the Royal London Free and Whittington
Hospitals states that iodine 125 seeds produce radiation for about one year, therefore if
death occurs within 12 months following implantation the seeds should be removed)
Cardiac resynchronisation therapy devices (CRTDs)
Dental mercury amalgam
Fixion nails
Hydrocephalus programmable shunts
Implantable cardioverter defibrillators (ICDs)
Implantable drug pumps including intrathecal pumps
Implantable loop recorders
Neurostimulators (including for pain and functional electrical stimulation)
Pacemakers
Ventricular assist devices (VADs): left ventricular assist devices (LVADs), right ventricular
assist devices (RVADs), or biventricular assist devices (BiVADs)
Any other battery powered implant.

In the above scenario, the other implants are not on this list, and so do not require removal
prior to cremation.

Current advice from the British Cochlear Implant Group states:

"At present it is not necessary for a cochlear implant to be removed prior to cremation
because it will not explode. But some crematoriums do require all implantable devices to be
removed. Parts of the implant may remain after cremation. If new implants are available in
the future which have batteries contained in the internal device, then it will be necessary for
the implant to be removed prior to cremation."

81. Which of the following does not cause liver cirrhosis?

Galactosaemia

Methotrexate

Schistosomiasis

Venoocclusive disease

Wilson's disease

Schistosomiasis causes portal hypertension and periportal fibrosis but not cirrhosis. Indeed,
liver function remains remarkably good in chronic infection.

Causes of cirrhosis include

Alcohol
Hepatitis B +/- D
Hepatitis C.

Autoimmune causes include

Primary and secondary biliary cirrhosis and


Autoimmune hepatitis.

Inherited conditions include

Hereditary haemochromatosis
Wilson's disease
Alpha-1 anti-trypsin deficiency
Galactosaemia
Glycogen storage disease
Cystic fibrosis.
Vascular causes include

Hepatic venous congestion


Budd-Chiari syndrome
Veno-occlusive disease.

Finally, intestinal bypass surgery for obesity has been implicated as a cause of cirrhosis.

82. A previously well 68-year-old man presents with jaundice weight loss and anorexia.

Biochemistry revealed:

Bilirubin 125 µmol/L (1-22)

AST 30 U/L (1-31)

Alkaline phosphatase 850 U/L (45-105)

Albumin 35 g/L (37-49)

Total protein 68 g/L (61-76)

U/S: Normal liver, a collapsed gallbladder with no stones: Normal cystic duct and common
bile duct, dilated intra-hepatic ducts.

Which of the following is the most likely diagnosis?

(Please select 1 option)

Primary biliary cirrhosis

Cholangiocarcinoma

Hepatocellular carcinoma

Hepatic Adenoma

Carcinoma of the head of the pancreas

The fact that the gallbladder is empty suggests an obstruction above the cystic duct. The
dilatation of bile ducts in both lobes of the liver tells you that the obstruction is at or below the
bifurcation.

The possibilities are a cholangiocarcinoma or external compression of the porta hepatitis


(usually by lymph nodes).
83. A 16-year-old girl presented with acute Guillain-Barré syndrome and has developed
worsening proximal muscle weakness.

Which one of the following tests should be used to monitor her respiratory function?

Arterial blood gas

Chest expansion

FEV1/FVC ratio

Peak expiratory flow rate

Vital capacity Correct

Diaphragmatic weakness occurs in one-third of patients with patients with Guillain-Barré


syndrome and involvement of the neck muscles, tongue and palate leads to further
respiratory compromise.

Respiratory muscle function is best monitored by frequent assessment of the forced vital
capacity (FVC).

ITU admission is recommended when FVC is less than 20 mL/kg and intubation is
recommended in most cases when FVC is less than 15 mL/kg.

FVC is also the best way to monitor respiratory function in any neurological disorders that
can affect the respiratory muscles (e.g. GBS, myasthenia gravis).

84. An 18-year-old primigravida of 12 weeks gestation would like to terminate her


pregnancy.

What is the most appropriate course of action?

Myomectomy

Oral misoprostol

Postcoital pill

Suction curettage

Uterine artery embolisation

It is too late for postcoital contraception. Suction curettage is usually the accepted form of
termination of pregnancy between 9-12 weeks gestation.

Termination of pregnancy (TOP) may be conducted legally up to 24 weeks gestation in the


UK. Reasons for wanting a TOP or abortion may include personal circumstances - including
risk to the wellbeing of existing children, a health risk to the mother or a high chance the
baby will have a serious abnormality, either genetic or physical.
Generally abortions should be conducted as early as possible in the pregnancy.

A patient with TB is receiving the following drugs as induction phase during treatment:

Ethambutol
Isoniazid
Pyrazinamide
Pyridoxine, and
Rifampicin.

85. Which drug is most likely to cause peripheral neuropathy?

(Please select 1 option)

Ethambutol

Isoniazid

Pyrazinamide

Pyridoxine

Rifampicin

The British National Formulary states that peripheral neuropathy is the only common side-
effect of isoniazid. It is more likely to occur in the presence of pre-existing risk factors such
as:

diabetes
alcohol dependence
chronic renal failure
pregnancy
malnutrition, and
HIV infection.

Supplementation with pyridoxine, vitamin B6, reduces the risk.

86. Which statistical term can be described as a type 1 error?

False negative

False positive

Null hypothesis
Power

Regression

Type 1 error is the incorrect rejection of the null hypothesis (that there is no difference
between groups) and may also be interpreted as a 'false positive' observation, that is, you
say that there is a difference when there is not.

87. A 2-week-old baby has hydrocephaly, seizures and chorioretinitis.

What is the single most probable diagnosis?

(Please select 1 option)

Cytomegalovirus (CMV)

Herpes simplex

Listeriosis

Syphilis

Toxoplasmosis

Maternal and fetal Toxoplasma infection may be avoided by advising pregnant women to
wear gloves when gardening or handling cat litter and to cook meat thoroughly.

Affected babies are treated with pyrimethamine, sulfadiazine and folic acid.

Further Reading:

AT Still University (ATSU). Infections of the Fetus and Newborn Infant.

88. A 56-year-old male presents to the clinic regarding red discolouration of the urine.

Two months ago he was diagnosed with a deep vein thrombosis and has been commenced
on warfarin. His most recent INR performed two days ago is 2.7.

On examination, there are no abnormalities to find yet his dipstick urine result reveals +++ of
blood and + protein. A MSU reveals no growth.

Which of the following is the most appropriate action for this man?
Reassure and monitor INR and warfarin dose closely

Monitor INR closely and refer to urology

Request repeat INR

Suggest discontinuing warfarin

Treat with trimethoprim.

This patient has unexplained haematuria and in the context of a previous DVT, underlying
occult neoplasia of the renal tract must be considered.

Therefore, an urgent referral to the urologists is the most appropriate course of action in this
patient.

In such cases, with a therapeutic INR, warfarin may unmask a potential neoplasm and one
must not attribute the haematuria to the warfarin.

89. A young child is admitted with a two day history of feeling unwell, fever and has
developed bloody diarrhoea.

Which of the following is the most likely cause?

Crohn’s disease

Escherichia coli 0157

Giardiasis

Polio

Threadworm infestation

Enterohaemorrhagic verocytotoxin-producing E coli0157:H7 is the most likely cause and is a


cause of haemolytic uraemic syndrome.

It would be unusual for Crohn's to present as acutely as this.

Polio usually first manifests as a non-bloody diarrhoea as does giardiasis.

90. A 60-year-old gentleman is seen complaining of 'chest pains'.


On further discussion he reports post-prandial retrosternal burning that has been present for
the last three to four months. It has been persistent and occurring daily over that time. He
reports that he has used an over-the-counter alginate antacid at least once a day for the last
two to three months and it has not really provided significant benefit.

He is usually fit and well with no significant past medical history. He is a smoker of 20 a day
and has done so since the age of 15. He drinks alcohol occasionally.

He reports that his swallow is normal with no dysphagia or odynophagia. His weight is
stable. He has had occasional associated vomiting when symptomatic but no persistent
vomiting. He denies any haematemesis and his bowel habit is stable with no rectal bleeding
or black stools.

On examination he has some angular stomatitis and mild glossitis. His abdomen is soft and
non-tender with no palpable masses.

What is the most appropriate management strategy?

(Please select 1 option)

Advise he takes the alginate preparation after meals TDS and also at night

Prescribe a daily high dose proton pump inhibitor for eight weeks (for example, omeprazole) then
review

Prescribe a daily H2 receptor antagonist (for example, ranitidine) for four weeks then review

Refer routinely for endoscopy

Refer urgently under the two week wait referral system

When managing someone with symptoms of gastro-oesophageal reflux disease (GORD) it is


important to look for the so-called 'alarm' features:

unintentional weight loss


dysphagia
GI bleeding
persistent vomiting, and
signs of anaemia.

New onset GORD symptoms in someone over the age of 55 is also a feature that should
raise concern.

In this case the patient has new onset GORD and is over the age of 55. He also has
examination findings that suggest iron deficiency anaemia. Of course, a full blood count
would be requested to look into this further, but the most appropriate approach would be to
refer this gentleman urgently for further assessment.

In the absence of alarm features, mild GORD symptoms that occur less than once a week
can managed with antacids when required. In patients with more frequent symptoms,
especially those who suffer on a daily basis, a proton pump inhibitor (PPI) can be used.
Initial treatment of GORD is with a high dose PPI for eight weeks. A PPI should be taken
once daily 30-60 minutes before the first meal of the day.

In patients without alarm features who display a partial response, you can increase the dose
to twice daily (e.g. omeprazole 40 mg BD). If nocturnal symptoms are troublesome the timing
of the dose can be adjusted.

If there is a no response to PPI treatment then you should reconsider the diagnosis and
consider specialist referral. A H2 receptor antagonist can be added to a PPI for patients who
have a partial response to PPI treatment.

91. You suspect Cushing's disease in a 50-year-old woman who has attended clinic with
glycosuria, hypertension and a suggestive body habitus.

Initial investigations point you towards a diagnosis of Cushing's disease.

Which of the following findings would be against this diagnosis?

A normal 8 am cortisol

Failure to suppress morning cortisol with dexamethasone

Hypertension requiring more than two antihypertensive agents

Impaired growth hormone response to glucose loading

Unilateral adrenal enlargement

Unilateral adrenal enlargement suggests an adrenal adenoma, and so Cushing's syndrome,


not disease.

A flat growth hormone response in a glucose tolerance test is also seen in Addison's
disease, hypopituitarism, and hypothyroidism.

Steroids can cause a lag response.

Pituitary surgery cures Cushing's disease in 80% of cases, with 15% late recurrence.
Radiotherapy may be needed in addition to surgery in adults, but is the first choice in
children.

Failure to suppress morning cortisol with dexamethasone suggests elevated cortisol.

Treatment of hypertension is standard, and may require more than one drug: the Cushing's
disease obviously needs treatment also.
92. Which of the following regular medications should be discontinued prior to general
anaesthesia?

(Please select 1 option)

Carbamazepine

Lisinopril

Morphine sulphate

Phenelzine

Pyridostigmine

Phenelzine and tranylcypromine are monoamine oxidase inhibitors which should be stopped
at least two weeks prior to elective surgery.

They can cause life threatening interactions with pethidine and indirect sympathomimetics.
They also prolong the action of suxamethonium by decreasing the concentration of plasma
cholinesterase.

Carbamazepine is an anticonvulsant and should be continued throughout the perioperative


period.

Gliclazide, a short acting oral hypoglycaemic, can be taken if the anticipated duration of
surgery is short.

Lisinopril, an angiotensin-converting enzyme inhibitor (ACEi) and digoxin, a cardiac


glycoside, should be continued pre-operatively.

Morphine sulphate tablets should be continued pre-operatively and a morphine infusion


(PCA) considered for postoperative analgesia.

Pyridostigmine is used in the management of myasthenia gravis and should be continued


prior to minor surgery. However, if perioperative muscle relaxation were required, then
omitting one or more doses of pyridostigmine would allow a reduction in the dose of the
muscle relaxant.

93. A 22-year female student is referred to the clinic having gained 3 kg in weight over the
last six months.

On examination she is obese, and has increased hair production in a male distribution.
Serum DHEA sulphate is markedly elevated. Testosterone and androstenedione are normal.

Which one of the following possible causes should be excluded?

Cushing's syndrome
Phenytoin

Polycystic ovaries

Prednisolone

Prolactinoma

This is a typical presentation of Cushing's syndrome.

The best investigations would be 24 hour urinary free cortisol, and an overnight
dexamethasone suppression test.

If these are normal then polycystic ovaries (PCOS) are the most likely cause, although
classically in PCOS the dehydroepiandrosterone (DHEA) is only slightly elevated or normal,
and the testosterone and androstenedione levels will be slightly elevated.

94. A 3-year-old child presents with brief generalised seizure. He has previously been
completely well. He had a full term normal delivery with no neonatal complications. His
immunisations are up to date. His sister is on imipramine for nocturnal enuresis.

On examination he is apyrexial, with respiratory rate of 18/min and pulse of 150/min. He


responds to voice and has a dry mouth and symmetrically large pupils. Glucose measured
by BM stix is 4.9 mmol/L.

What is the most likely diagnosis

Electrolyte disturbance

Encephalitis

Idiopathic epilepsy

Meningitis

Tricyclic overdose

The history is of brief generalised seizure in a child with tachycardia, dry mouth and dilated
pupils.

The potential exposure to tricyclic antidepressants suggests possible accidental overdose.

About 15% of children get fits, with central nervous system effects more common in children
than in adults, in whom CVS side-effects are more prominen
95. A 48-year-old woman with chronic pancreatitis due to gallstones is noted to have a
macrocytic anaemia.

What is the most likely cause of the anaemia?

(Please select 1 option)

Bone marrow dysfunction

Folate deficiency

Hyposplenism

Hypothyroidism

Vitamin B12 deficiency

Chronic pancreatitis and the resultant pancreatic insufficiency results in the failure of splitting
of dietary B12 from R-binders, a reaction that requires trypsin.

This inhibits the binding of intrinsic factor to the vitamin B12 so it is not absorbed.

In alcoholic chronic pancreatitis (which accounts for up to 90% of cases of chronic


pancreatitis) macrocytosis is also caused by folate deficiency and bone marrow dysfunction.

96. A 25-year-old schizophrenic man has required readmission to hospital after stabbing his
mother's hand with a screwdriver. Now he is symptom free.

From the list below choose the most suitable form of psychotherapy you should offer in this
situation.

Brief psychodynamic psychotherapy

Cognitive behavioural therapy

Counselling

Interpersonal therapy

Systemic desenisitisation

There is evidence that family interventions can reduce relapse rates in schizophrenia.

Brief psychodynamic psychotherapy addresses the patient's difficulties by primarily


relying on insight, bringing unconscious or unclear material into awareness, and linking past
and present experiences.

Cognitive behavioural therapy combines principles of both behavioural and cognitive


therapy, focusing simultaneously on the environment, behaviour, and cognition. It is also:

Structured
Goal directed
Problem focused, and
Time limited.

Interpersonal therapy is derived partially from a psychodynamic perspective and focuses


primarily on the patient's interpersonal relationships. Interpersonal therapy is fairly non-
directive and addresses issues such as:

Grief
Role transitions
Interpersonal role disputes, and
Interpersonal deficits

as they relate to the patient's current symptoms.

Family therapy helps family members learn about the disorder, solve problems, and cope
more constructively with the patient's illness. It may also be useful in this situation.

Systemic desensitisation is exposure-based behavioural treatment which utilises gradual,


systematic, repeated exposure to the feared object or situation to allow patients with anxiety
disorders to become desensitised to the feared stimulus.

97. Which of the following statements regarding thyrotoxicosis is correct?

(Please select 1 option)

Hyperemesis gravidarum rarely affects thyroid blood tests

In pregnancy carbimazole is preferable over propylthiouracil

Most patients will need radioiodine at some point

The time course of active eye disease has a predictable pattern

TSH towards the upper limit of normal excludes the diagnosis

The time course of eye disease has an active then plateau phase totalling approximately two
years.

Radioiodine, if given alone, may worsen hyperthyroidism - if necessary, steroids are given at
the same time.

Propylthiouracil is commonly used in pregnancy; regular review of patients is required.

Hyperemesis affects thyroid function tests.

A thyroid-stimulating hormone (TSH) towards the upper limit of normal may underlie a (rare)
TSH-oma.
20382
98.

This 55-year-old male presents with these lumps in his groin which have been present for
about one year and are now aching a little. He describes that on lying flat they entirely
disappear.

On examination whilst lying flat no masses are palpable or seen. On standing these masses
are seen and are soft and fluctuant to palpation.

What is the likely diagnosis?


Direct inguinal hernia

Femoral hernia

Indirect inguinal hernia

Lipoma

Saphena varix

The likely diagnosis is a saphena varix, which is a varicosity of the long saphenous vein.

Classically, it disappears on lying flat.

99. Which of the following statements is of aetiological significance in a patient with


cardiomyopathy?

AST of 50 IU/l with a bilirubin of 12 mmol/l in a 50-year-old lady

The presence of a soft pan-systolic murmur in the mitral area

The presence of diabetes mellitus in a tanned patient

The presence of sinus tachycardia with BP 140/80 mmHg

T-wave flattening in the inferior ECG leads in a 60-year-old man

The most common form of cardiomyopathy is dilated cardiomyopathy, with dilatation of the
left ventricle and reduction in the ejection fraction.

Often no cause is found, although rarely, it can be familial. A cause should be looked for in
most cases.

The commonest causes are

Ischaemic heart disease (Q-waves on the ECG, history of myocardial infarction [MI]) and
Hypertension.

Non-specific ECG changes and atrial fibrillation are common and do not point to a specific
cause.

Minor abnormalities of liver function result from liver congestion.

More marked abnormalities and diabetes in a pigmented patient would suggest


haemochromatosis.
Mitral regurgitation and arrhythmias are common in dilated cardiomyopathy of any cause.

100. The following commonly cause a rise in serum alkaline phosphatase (ALP) activity
with the exception of which one?

(Please select 1 option)

Juvenile idiopathic arthritis (JIA)

Osteomalacia

Osteoporosis

Paget's disease of bone

Polymyalgia rheumatica (PMR)

ALP activity may rise as part of an acute phase response, for example in active PMR,
rheumatoid arthritis (RA) or JIA.

Bone-specific ALP will rise in Paget's disease of bone and more severe cases of
osteomalacia and rickets.

It may be used as a guide to disease activity in the former. It only rises in osteoporosis
following an acute fracture.

The ALP may also be elevated in parathyroid bone disease and secondary to metastases.

101. A 12-year-old boy on treatment for generalised tonic clonic epilepsy, develops
features of hepatic failure within a few weeks of starting treatment.

Which drug is most likely responsible for this presentation?

Carbamazepine

Ethosuximide

Lamotrigine

Phenytoin

Sodium valproate

Side effects of valproate include:

weight gain
nausea
vomiting
hair loss
easy bruising
tremor
hepatic failure
pancreatitis.

102. A 9-year-old girl presents with knee swelling. She had a diarrhoeal illness four weeks
before, but has otherwise been healthy.

Full term normal delivery, no neonatal problems. Immunisations up to date. No family or


social history of note.

On examination the temperature is 36.9°C, respiratory rate is 15/min and pulse is 80/min.
She has slight swelling of the right knee, which has 20 degrees loss of both flexion and
extension.

What is the most likely diagnosis?

(Please select 1 option)

Dermatomyositis

Oligoarticular juvenile rheumatoid arthritis

Polyarticular juvenile rheumatoid arthritis

Reactive arthritis

Systemic juvenile rheumatoid arthritis

The history of mild joint inflammation a few weeks after a diarrhoeal illness suggests a
reactive arthritis.

This usually settles within six weeks, which helps to distinguish it from juvenile chronic
arthritis.

Bacterial triggers include:

Campylobacter
Shigella
Salmonella, and
Yersinia.

Viral triggers include:

Rubella
Hepatitis B
Parvovirus, and
Epstein-Barr virus (EBV).

Treatment is with anti-inflammatories and physiotherapy

103. A 25-year-old pregnant woman presents with paraesthesia in her right hand and
Tinel's sign is positive.

What is the most appropriate management for this patient?

Allopurinol

Colchicine

Oral steroids

Physiotherapy

Splinting

She has carpal tunnel syndrome which is often self-limiting in pregnancy. It is best to treat
conservatively till the swelling settles and compression is relieved off the median nerve. Try
splinting in the first instance.

Local steroid injection may be helpful if splinting fails.

Tinel's sign is a tingling sensation on percussion of the nerve over the carpal tunnel.

104. A 68-year-old woman presents with severe colicky central abdominal pain, vomiting,
and the passage of abnormal stool which had the appearance of redcurrant jelly.

On examination, temperature was 37.8°C, she has a pulse of 123 bpm with an irregular rate.
Palpation of the abdomen revealed generalised tenderness and peritonitis.

Investigations reveal:

Haemoglobin 126 g/L (120-160)

White cell count 28 ×109/L (4-11)

Lactate 8 mmol/L (<2)


pH 7.15 (7.36-7.44)

She was taken to theatre for emergency surgery.

What is the likely diagnosis?

(Please select 1 option)

Acute mesenteric ischaemia

Chronic mesenteric ischaemia

Crohn’s disease

Infective colitis

Ulcerative colitis

The history is consistent with acute mesenteric ischaemia and given the atrial fibrillation that
had not previously been diagnosed the underlying pathology is probably embolic occlusion
of the superior mesenteric artery.

105. A 70-year-old lady presents with a two month history of a swelling in the front of the
neck. There has been a rapid increase in the size of the lump and she has become
hoarse.

On examination she is noted to have an audible wheeze and there is a large hard mass in
the lower anterior part of the neck.

What is the most likely diagnosis?

Anaplastic carcinoma of thyroid

Branchial cyst

Pharyngeal pouch

Riedel's thyroiditis

Thyroglossal cyst
This is a very short history of two months. The rapid increase in size suggests malignancy.
The other feature which is compatible with this is the hoarse voice due to involvement of the
recurrent laryngeal nerve.

The audible wheeze will also be due to pressure on the trachea and paralysis of the
recurrent laryngeal nerve will also have effects on the vocal chord. The hard mass in the
lower anterior part of the neck is suggestive of anaplastic carcinoma.

Riedel's thyroiditis would also produce a hard, irregular swelling of the thyroid gland, but
have a longer history.

106. A 72-year-old man is referred with a two month history of progressive disorientation
and falls.

Two weeks beforehand, he locked his wife out of their house, claiming that she was trying to
steal his clothes. He had also telephoned the police in the middle of night, claiming that he
could see men hiding under his bed.

On examination, his face is expressionless, his speech is quiet and monotonic. There are no
cranial nerve palsies, otherwise. Increased tone is present in all four limbs, with a slow
festinant gait. Reflexes, power and sensation are all normal. Halfway through your
examination he tells you that he is leaving the room, because of the "lobsters coming
through the window". Unfortunately, therefore, formal cognitive testing and basic
investigations cannot be performed.

On the basis of this evidence, what is the most likely diagnosis?

(Please select 1 option)

Lewy body dementia

Multiple system atrophy

Normal pressure hydrocephalus

Parkinson's disease

Progressive supra-nuclear palsy

This man has parkinsonism, with bradykinesia and rigidity.

The florid visual hallucinations and paranoid ideation make Parkinson's disease unlikely.

The normal eye movements and normal postural blood pressure mitigate against a
parkinsonism plus syndrome, while the lack of incontinence and gait abnormalities make
normal pressure hydrocephalus less probable.

Parkinsonism with dementia, paranoia and visual hallucinations is found in dementia with
Lewy bodies.
107. A young infant presents with a suspected diagnosis of pyloric stenosis.

Which of the following is a feature of this diagnosis?

Anorexia

Hyperchloraemic acidosis

Loose stools

More frequent in girls

Projectile vomiting

Projectile vomiting of large quantities of curdled milk is characteristic.

Anorexia and loose stools are not clinical features.

It is commonest in first-born male children.

The classical biochemical picture is of hypokalaemic, hypochloraemic metabolic alkalosis.

108. Fat necrosis is hard, craggy and painful and hence mimics breast cancer. However,
because the woman was in a car crash two weeks previously, the history suggests
trauma to the breast.

A smooth mobile lump in a woman aged 40-50 should be treated suspiciously. However, in
the absence of an option of breast cancer, the most likely diagnosis is a phyllodes tumour.

109. You are told that her potassium result is low at 3.1 mmol/L (3.5-4.9). She is
asymptomatic and is currently eating her lunch.

This is classed as mild hypokalaemia and is best treated conservatively.

Foods such as tomatoes and bananas contain high levels of potassium and could be
offered.

As she is asymptomatic and able to eat, intravenous fluids are unnecessary. Furthermore,
this hypokalaemia is probably due to large volumes of normal saline given over the last few
days.

A loop diuretic is not indicated.


110. Which of the following women does not have an increased risk of acute myocardial
infarction (MI)?

(Please select 1 option)

A 50-year-old woman with a past history of Kawasaki disease

A 48-year-old woman with a 12 year history of haemochromatosis

A 48-year-old woman with systemic lupus erythematosus (SLE) diagnosed six years ago

A 38-year-old woman with anti-phospholipid syndrome

A 42-year-old woman with longstanding rheumatoid arthritis

It is now recognised that chronic inflammatory conditions, in particular SLE, can cause
accelerated atherosclerosis.

Kawasaki disease leads to coronary aneurysms, which can cause MI by related thrombosis
or stenosis.

MI can also result from coronary thrombosis, predisposed to by anti-phospholipid syndrome.

Haemochromatosis can result in a cardiomyopathy, but not MI.

111. Which of the following is correct regarding proton pump inhibitors?

(Please select 1 option)

Omeprazole is a cytochrome p450 inducer

They are excreted unchanged by the kidney

They cause hair loss, diarrhoea and headache

They lower serum gastrin levels

They reversibly block parietal cell proton pumps

Proton pump inhibitors irreversibly block the hydrogen-potassium adenosine triphosphatase


enzyme system (the proton pump) of the gastric parietal cells.

Eighty percent of these pumps are recycled each day, resulting in a duration of action of 16
to 20 hours. They result in profound acid suppression, causing a reflex hypergastrinaemia.

The majority of their metabolism is hepatic and they suppress cytochrome p450 activity.

The common side effects are abdominal pain, diarrhoea and headache, although alopecia is
recognised.
112. A 16-year-old male presents with acute severe asthma.

On examination his peripheral pulse volume fell during inspiration.

Which one of the following is the most likely explanation for this clinical sign?

(Please select 1 option)

A falling heart rate on inspiration

Myocardial depression due to hypoxia

Peripheral vasodilatation

Reduced left atrial filling pressure on inspiration

The cardiac effect of high dose beta agonist bronchodilator drugs

This patient is demonstrating pulsus paradoxus.

The right heart responds directly to changes in intrathoracic pressure, while the filling of the
left heart depends on the pulmonary vascular volume.

At high respiratory rates, with severe air flow limitation (for example, acute asthma) there is
an increased and sudden negative intrathoracic pressure on inspiration and this will enhance
the normal fall in blood pressure.

113. A 10-year-old girl presents with a swelling in the lower part of the inferior constrictor.
Her mother reports that as an infant the girl was troubled by regurgitation and has always
had problems swallowing.

The lump in her neck increases after eating and the girl reports that on pressing the swelling
she feels food in her throat again. There is cough impulse on examination.

What is the most likely diagnosis?

(Please select 1 option)

Cystic hygroma

Goitre

Pharyngeal pouch Correct

Subclavian artery aneurysm


Thyroglossal cyst

Neck swellings are usually classified on their location into:

Anterior triangle swellings (anterior to the sternomastoid muscle), and


Posterior triangle swellings (posterior to sternocleidomastoid).

Typically, swellings in the posterior triangle are lymph nodes.

There are more diverse origins for anterior triangle swellings. Common differentials are
lymph nodes and thyroid swellings.

Swellings with cough impulse are pharyngeal and laryngeal pouches/diverticula and cystic
hygroma. It is true cough impulse in the former conditions while the latter is an ill-defined sac
resulting from remnants of an undeveloped lymph sac and the cough impulse is transmitted.

Lymph node swellings are generally found along veins. They are divided into six levels
mainly to stage metastatic disease and standardise surgical procedures by the digastric
muscle, sternomastoid muscle and hyoid bone.

In the above question, history, the anatomical situation and examination findings point to the
answer.

114. A 4-month-old infant presents with three day history of runny eyes and nose. There is
associated breathlessness, wheezing and coughing. There is also refusal to feed.

Examination reveals widespread polyphonic wheezes, tachypnoea, tachycardia and


intercostal recession. There is a family history of asthma and hay fever.

What is the most likely diagnosis?

(Please select 1 option)

Asthma

Bronchiolitis

Cystic fibrosis

Gastroesophageal reflux

Hyperventilation

Even though there is a family history of asthma the baby is too young to make this diagnosis.
The symptoms suggest a preceding history of coryza and some respiratory distress.

In this age group the most likely diagnosis is bronchiolitis.

Signs of respiratory distress in an infant include:


nasal flaring
intercostal retractions
grunting
respiratory rate >70 breaths per minute
dyspnoea, and
cyanosis.

Along with this poor feeding, lethargy and hypoxaemia are all indications to hospitalise the
patient.

Next question

115. A 31-year-old female has thin vaginal discharge and pain during micturition. On
examination she had multiple ulcers at the vaginal introitus.

What is the most likely diagnosis?

(Please select 1 option)

Bacterial vaginosis

Carcinoma vulva

Chlamydial trachomatis

Herpes simplex

Trichomonas vaginalis

HSV is a sexually transmitted disease.

Burning dysuria, thin discharge and multiple small ulcers are suggestive of genital herpes. It
is a painful condition, which may become a chronic illness. Investigation for HIV and other
STDs should be conducted.

Treatment is with antiviral therapy

116. Which complication of chronic kidney disease is most likely to be associated with
failure of 1-alpha hydroxylation of cholecalciferol?

(Please select 1 option)

Anaemia

Gout

Hyperlipidaemia
Hypertension

Renal osteodystrophy

Failure of 1-alpha hydroxylation of vitamin D results in hypocalcaemia and secondary/tertiary


hyperparathyroidism, that is, renal osteodystrophy. Patients require treatment with 1-alpha
cholecalciferol.

Anaemia in renal failure patients is due to deficiency in erythropoietin.

Gout is due to defective metabolism of uric acid. In chronic kidney disease NSAIDS should
be avoided, so this is usually treated with steroids.

Hypertension may be a cause of kidney disease and is also a manifestation of


glomerulonephritis, or fluid retention in a patient suffering with CKD.

Hyperlipidaemia is not generally caused by renal disease

117.

The figure illustrates the appearance of a patient's foot. He complains of parasthesia in his
feet and has been feeling hot and sweaty.

Choose the single most appropriate treatment from the list:


(Please select 1 option)

Gabapentin

IV antibiotics

Larval therapy

Oral antibiotics

Surgical debridement

Diabetes results in defective responses to infection and the presence of an infected ulcer in
the setting of peripheral neuropathy should be treated very seriously.

Protracted antibiotic therapy is often required and preventative measures need to be taken
to prevent recurrence, including attention to foot care and provision of adequate footwear.

This patient will need admission to facilitate further treatment, either following discussion
with your local diabetologist or via the acute intake, depending on local facilities and
pathways.

Failure to respond to antibiotic therapy may require surgical debridement or even


amputation.

Topical larval therapy has been used with some effect and may reduce the need for surgical
removal of necrotic tissue.

118. Which of the following conditions is NOT associated with hyperuricaemia?

(Please select 1 option)

Lesch-Nyhan syndrome

Thiazide diuretic therapy

Osteoarthritis

Severe exfoliative psoriasis

Low dose aspirin therapy

Uric acid is the final product of purine metabolism. Raised levels are most commonly
associated with gout, but more recently they have been identified as a marker for a number
of metabolic and haemodynamic abnormalities (including ischaemic heart disease, diabetes
mellitus, hypertension and hyperlipidaemia). Blood levels depend of the rate of breakdown
and excretion, and most cases of hyperuricaemia are associated with defective elimination.

Causes of hyperuricaemia include:

Underexcretion:
• Renal impairment
• Metabolic syndrome
• Drugs: diuretics, low-dose aspirin, cyclosporin, tacrolimus, pyrazinamide, ethambutol,
levodopa, nicotinic acid
• Acidosis
• Pre-eclampsia and eclampsia
• Hypothyroidism, hyperparathyroidism
• Sarcoidosis
• Familial juvenile gouty nephropathy
• Chronic lead intoxication
• Trisomy 21
• Idiopathic

Overproduction:
• Purine rich diet
• Tumour lysis syndrome
• HGPRT deficiency (Lesch-Nyhan and Kelley-Seegmiller syndromes)
• Increased PRPP synthetase activity
• Polycythaemia, both primary and secondary
• Idiopathic

Combined
• Alcohol
• Exercise
• Glucose-6-phosphatase deficiency
• Aldolase B deficiency

Management of hyperuricaemia relies on determining whether the patient is symptomatic


and asymptomatic, and identifying aetiology and comorbidities. Symptomatic patients
present with gout and nephrolithiasis. Most patients with hyperuricaemia never develop gout
or stones. Treatment of these patients is not considered beneficial or cost-effective, and is
therefore generally not recommended. The exception to this is in patients at risk of tumour
lysis syndrome, where prophylactic treatment is recommended to reduce the incidence of
renal impairment.
Allopurinol is the mainstay of treatment for hyperuricaemia. However, treatment depends on
the cause and clinical setting and newer agents such as rasburicase and pegloticase are
increasing in use.

References:

Uric acid in the etiology of psoriasis. Goldman M. Am J Dermatopathol 1981


Winter;3(4):397-404
Hyperuricemia in psoriatic arthritis: prevalence and associated features. J Clin Rheumatol
2000 Feb;6(1):6-9

119. A 2-month-old child presents with snuffliness and difficulty in feeding.

He has a crusty nose and low-grade fever. His respiratory rate is 40/min. Symmetrical
conducted secretion sounds are heard in the chest.

What is the most likely diagnosis?


(Please select 1 option)

Acute sinusitis

Asthma

Croup

Gastroesophageal reflux

Upper respiratory tract infection

The history is suggestive of a viral upper respiratory tract infection.

Babies are often obligate nasal breathers in the first few months of life.

Normal saline nose drops may assist feeds by thinning nasal secretions

120. A 25-year-old shop attendant presents to the local emergency department with left
loin pain, which radiates into his groin. He is clearly in some discomfort.

Upon being informed that he has renal colic, the patient expresses an urgent desire for some
analgesia.

Which of the following is the most appropriate medication to prescribe in the circumstances?

(Please select 1 option)

Gabapentin

Intramuscular diclofenac

Oral amitriptyline

Oral morphine

Oral paracetamol

Non-steroidal anti-inflammatory drugs (NSAIDs), either intramuscularly or by suppository,


are the first line treatment for renal colic.

Strong opiates are regarded as appropriate second line therapy.

121. Which of the following conditions is not associated with Wolff-Parkinson-White


syndrome?
(Please select 1 option)

Ebstein's anomaly

Hypertrophic cardiomyopathy

Mitral valve prolapse

Secundum atrial septal defect

Ventricular tachycardia in the absence of drug therapy

Wolff-Parkinson-White syndrome (WPW) occurs in 1.5/1000 people. An accessory pathway


connects atria to ventricles making the patient prone to atrio-ventricular re-entry tachycardia
(AVRT), atrial fibrillation, and atrial flutter. A broad complex tachycardia can result from
conduction down the accessory pathway and up the AV node during AVRT, but VT does not
occur. AF can degenerate very rapidly to VF as the accessory pathway does not slow
conduction, and this is the main cause of death.

122. Which of the following treatments does not reduce mortality with regard to chronic
heart failure?

(Please select 1 option)

Bisoprolol

Digoxin

Enalapril

Nitrates and hydralazine

Spironolactone

Bisoprolol (CIBIS II), spironolactone (RALES), enalapril (CONSENSUS) and nitrates and
hydralazine (V-HEFT) have all been shown to improve mortality in chronic heart failure.

Digoxin reduces the risk of death due to heart failure but overall cardiovascular mortality is
similar to placebo, probably reflecting a small increase in the risk of arrhythmic death with
digoxin therapy.

A 45-year-old carpenter attends the Emergency department after cutting his left index finger
with a saw.

On examination there is a 2 cm incised wound on the radial aspect of the index finger. The
neurovascular supply is intact and there is no evidence of injury to other structures.
123. Which is the appropriate local anaesthetic to use to explore the wound and suture the
laceration?

(Please select 1 option)

Lidocaine 1% - 20 ml

Lidocaine 1% - 250 ml

Lidocaine 1% with adrenaline - 20 ml

Lidocaine 1% with adrenaline - 250 ml

Topical ethyl chloride

For routine use 1% lidocaine is the most frequently used anaesthetic for cleaning, exploring
and suturing wounds in the Emergency department.

Adrenaline should not be used in sites supplied by end arteries, such as fingers and toes.

The maximum dose of plain lidocaine in a healthy adult is 3 mg/kg or 200 mg (20 ml of 1%).
Remember that 1% lidocaine = 10 mg/ml.

If lidocaine with adrenaline is used the maximum dose is 7 mg/kg or 500 mg (50 ml of 1%).
The duration of action is 30-60 minutes for plain lidocaine and approximately 90 minutes for
lidocaine with adrenaline.

Ethyl chloride is a topical anaesthetic that can be sprayed onto the skin and causes rapid
cooling. It is very short acting (lasts less than 60 seconds) and would provide inadequate
analgesia in this case.

124. A 17-year-old boy complains of recurrent syncope and presyncope over the past
year, usually during physical activity.

Clinical examination reveals a double apical impulse and an ejection systolic murmur at the
lower left sternal edge. His ECG shows pathological Q waves in the anterior chest leads.

What is the most likely cause of syncope in this patient?

(Please select 1 option)

Arrhythmogenic right ventricular cardiomyopathy

Congenital long QT syndrome

Hypertrophic cardiomyopathy

Left atrial myxoma


Vasovagal syncope

Syncope during exercise, double apical impulse and the Q waves on electrocardiogram
suggest hypertrophic cardiomyopathy (HOCM).

125. Which of the following is a bronchodilator that inhibits phosphodiesterase?

(Please select 1 option)

Dobutamine

Doxapram

Ipratropium

Salbutamol

Theophylline

Theophyllines cause bronchial smooth muscle relaxation by inhibiting phosphodiesterase to


intracellular cyclic adenosine monophosphate (CAMP).

Dobutamine is predominantly a beta 1 agonist.

Ipratropium is an antimuscarinic bronchodilator.

Doxapram is a specialist respiratory infusion used to stimulate the breathing in the absence
of non-invasive ventilation.

Salbutamol is a beta 2 agonist.

126. A 55-year-old man with poor physical hygiene presents with sudden onset severe left
earache and deafness after taking a shower. He says he tried to clean his ears but it only
worsened the pain.

It is very difficult to get views of his eardrums.

What is the most likely diagnosis?

(Please select 1 option)

Acoustic neuroma

Acute otitis media

Ototoxicity
Perforation of tympanic membrane

Wax impaction

Deafness could result when there is impediment to conduction of sound waves from the
exterior to the brain or conductive bones. Lesions could be present in the auditory canal,
middle ear or sensory neural (lesions of inner ear or cochlear) and acoustic nerve.

Clinical examination includes tuning fork tests which assess air conduction (AC) and bone
conduction (BC) in the individual compared to that of the examiner (presuming their hearing
is normal). The common tests used are the Rinne test and the Weber test.

Rinne's test has two components:

Firstly the AC of the examiner and the patient are compared


Secondly it differentiates conductive deafness and sensory neural deafness.

In normal individuals, the AC is greater than the BC. But in conductive deafness BC is
greater than AC. If a person is found to be deaf by Rinne's 1 and has a normal AC/BC
relation, he has sensory neural deafness.

Webers' test compares BC on both sides and lateralises to the normal side.

Gentamicin is well recognised to be ototoxic and is particularly toxic to the cochlear.


Although ensuring adequate therapeutic concentrations may minimise this side effect,
ototoxicity may still occur even with concentrations maintained in the therapeutic range.

Hearing reduction after a shower is not uncommon with water in the auditory canal and may
exacerbate hearing decline associated with wax. However, discomfort and hearing loss may
be compounded by trying to displace the water through stuffing objects into the auditory
canal.

A perforated tympanic membrane should be visible on examination.

Wax impaction can be softened using over the counter drop preparations and syringing may
also help.

127. 68-year-old female presents 24 hours after hysterectomy with breathlessness.

On examination, she has a raised jugular venous pressure and bibasal chest crepitations.

What is the most likely fluid or electrolyte abnormality?

(Please select 1 option)

Fluid overload

Hypokalaemia
Metabolic acidosis

Metabolic alkalosis

Respiratory alkalosis

This woman clearly has cardiac failure with fluid overload, as indicated by the raised JVP
and bibasal crepitations. If the patient does not have a premorbid history of congestive
cardiac failure, this may have been iatrogenic secondary to intravenous fluids.

A chest x ray will help to confirm the diagnosis along with a BNP blood level. Signs to look
for on the chest x ray include:

A Alveolar shadowing

B Kerly B lines

C Cardiomegaly

D Upper lobe diversion

E Pleural effusion

F Fluid in the fissure

128. A 40-year-old female solicitor attends her general practitioner's surgery and
complains of postcoital bleeding of two months duration.

She does not experience pain during intercourse and has not had any vaginal discharge
other than the post-coital bleeding. She is still having regular periods.

Which of the following is the least likely diagnosis in this case?

(Please select 1 option)

Cervical polyps

Cervical trauma

Cervicitis

Salpingo-oophoritis
Vaginitis

Postcoital bleeding usually occurs due to trauma to superficial lesions within the vaginal
tract. Typical causes of postcoital bleeding include:

Cervical trauma
Cervical polpys
Endometrial and vaginal carcinoma
Cervicitis, and
Vaginitis of any cause.

Invasive cervical carcinoma may be found in 3.8% of those referred to hospital.

Vaginitis is a possibility but would be more common in elderly patients with low oestrogen.

Salpingo-oophoritis, usually due to pelvic inflammatory disease from sexually transmitted


infections, most frequently presents with deep dyspaneuria (pain during intercourse) and
purulent vaginal discharge. Post-coital bleeding is highly unlikely to be caused by salpingo-
oophoritis

129. A toddler who has von Willebrand's disease is admitted after a fall from his bicycle.

He has a haemathrosis of his left knee and bruises on his right lower leg.

What transfusion is indicated?

(Please select 1 option)

Factor VIII concentrate Correct

Factor IX

Fresh frozen plasma

Packed cells

Platelet transfusions

von Willebrand disease (vWD) is an autosomally inherited mucocutaneous bleeding disorder


caused by a deficiency or dysfunction of the protein termed von Willebrand factor (vWF).

Consequently, primary haemostasis is impaired because of defective interaction between


platelets and the vessel wall.

vWD is divided into three major categories:

Type I - Partial quantitative deficiency


Type II - Qualitative deficiency
Type III - Total deficiency.

The two main treatment options for patients with vWD are DDAVP and vWF-containing FVIII
concentrate transfusion therapy.

Fresh frozen plasma contains functional von Willebrand factor but should be avoided
whenever possible because of the potential transmission of viral disease. An additional
drawback of fresh frozen plasma is the large infusion volume required.

Only occasionally are platelet transfusions required.

130. Which of the following is true regarding a common peroneal nerve palsy?

(Please select 1 option)

Does not cause weakness of dorsiflexion of the foot

Is associated with sensory loss over the dorsum of the foot

Is commonly caused by damage to the nerve in the popliteal fossa

Recovery will be delayed if a foot splint is used

Will result in an upgoing plantar response

Common peroneal palsy occurs most commonly as a result of damage to the nerve at the
neck of the fibula.

It produces weakness of eversion, dorsiflexion and inversion of the foot and sensory loss
over the dorsum of the foot.

An L5 root lesion will also cause a foot drop but only dorsiflexion is affected.

131. A 75-year-old man with atrial fibrillation presents with epistaxis. He takes warfarin
regularly, but has not bothered to see his general practitioner or the anticoagulation
nurse for the last three months.

He has maintained a regular dose of 5 mg daily. His INR is 8.

Which of the following is the most suitable treatment option?

(Please select 1 option)

Change to heparin infusion

Fresh frozen plasma

Monitor INR

Protein complex concentrate


Reverse the INR using vitamin K

Anticoagulated patients are at high risk of epistaxis. The risk of bleeding increases
exponentially at an INR >5.

Anticoagulant reversal is sometimes required. A decision should be made if there is active


bleeding and if it is minor or major.

Vitamin K is the specific antidote for warfarin, however in cases of major life threatening
bleeds (for example, GI bleeds, intracerebral haemorrhage), protein complex concentrate
can be used.

Note FFP is not recommended as a reversal for warfarin as first line.

132. A 60-year-old man with diabetes complains of recent onset of postural syncope,
impotence, diarrhoea and profuse inappropriate sweating.

What is the most likely cause of syncope in this patient?

(Please select 1 option)

Autonomic neuropathy

Complete heart block

Mitral regurgitation

Paroxysmal atrial fibrillation

Vasovagal syncope

The patient has autonomic features. A diabetic autonomic neuropathy should be suspected
as the cause for his syncope.

Mitral regurgitation, unless associated with atrial fibrillation is not expected to cause
syncope.

PAF often presents with palpitations and light headedness followed by syncopal episodes
(not always).

133. A 4-month-old infant presents with three day history of runny eyes and nose. There is
associated breathlessness, wheezing and coughing. There is also refusal to feed.

Examination reveals widespread polyphonic wheezes, tachypnoea, tachycardia and


intercostal recession. There is a family history of asthma and hay fever.

What is the most likely diagnosis?


(Please select 1 option)

Asthma

Bronchiolitis Correct

Cystic fibrosis

Gastroesophageal reflux

Hyperventilation

Even though there is a family history of asthma the baby is too young to make this diagnosis.
The symptoms suggest a preceding history of coryza and some respiratory distress.

In this age group the most likely diagnosis is bronchiolitis.

Signs of respiratory distress in an infant include:

nasal flaring
intercostal retractions
grunting
respiratory rate >70 breaths per minute
dyspnoea, and
cyanosis.

Along with this poor feeding, lethargy and hypoxaemia are all indications to hospitalize the
patient

134. Which of the following drugs is a long acting beta-adrenoceptor agonist?

(Please select 1 option)

Aminophylline

Fluticasone propionate

Formoterol Correct

Oxitropium bromide

Terbutaline

Formoterol is a long acting beta-adrenoceptor agonist (LABA) and relaxes bronchial smooth
muscle causing bronchial dilatation. Similar long acting agents include salmeterol.
A LABA is used as an add on therapy in step 3 of the British Thoracic Society (BTS) Asthma
Guidelines.

A SABA is a short acting beta agonist and is used as a first line of symptom control in those
with asthma.

Next question

135. Which complication of chronic kidney disease is most likely to be associated with
failure of 1-alpha hydroxylation of cholecalciferol?

(Please select 1 option)

Anaemia

Gout

Hyperlipidaemia

Hypertension

Renal osteodystrophy Correct

Failure of 1-alpha hydroxylation of vitamin D results in hypocalcaemia and secondary/tertiary


hyperparathyroidism, that is, renal osteodystrophy. Patients require treatment with 1-alpha
cholecalciferol.

Anaemia in renal failure patients is due to deficiency in erythropoietin.

Gout is due to defective metabolism of uric acid. In chronic kidney disease NSAIDS should
be avoided, so this is usually treated with steroids.

Hypertension may be a cause of kidney disease and is also a manifestation of


glomerulonephritis, or fluid retention in a patient suffering with CKD.

Hyperlipidaemia is not generally caused by renal disease.

136. Theme:Breathlessness

A Asthma (non-occupational)

B Atelectasis

C Bronchiectasis

D Hypersensitivity pneumonitis
E Idiopathic pulmonary fibrosis

F Inhaled foreign body

G Occupational asthma

H Pneumonia

I Pneumothorax

J Pulmonary embolism

K Sarcoidosis

For each of the following cases, select the most likely diagnosis from the given list:

A 19-year-old male smoker presents with sudden onset of breathlessness and pain in the left
shoulder. On auscultation there were reduced breath sounds in the left upper zone.
Pneumothorax

Correct
This case presents with typical features of pneumothorax.

A 48-year-old farm worker presents with dry cough, breathlessness, fever, and a flu-like illness
that had developed over a period of hours. He reported previous milder attacks, all occurring
after he had been forking hay.
Hypersensitivity pneumonitis

Correct
This is a case of farmer's lung - a hypersensitivity pnuemonitis (previously known as
extrinsic allergic alveolitis).

A 3-year-old girl presents with a four hour history of severe cough and wheeze. On auscultation
there was a monophonic wheeze in the right lung.
Inhaled foreign body

Correct
This case has features suggestive of an inhaled foreign body.
A 60-year-old woman presents one day after having had a cholecystectomy, with mild
breathlessness and temperature of 37.5°C. On auscultaion breath sounds were reduced in both
lung bases.
Atelectasis

Correct
This woman has developed a slight fever and breathlessness occurring one day after a
procedure which suggests atelectasis.

A 5-year-old girl presents with a two month history of nocturnal dry cough. Her parents have also
noticed that she sometimes coughs following exertion. No abnormalities are noticed on chest
examination and normal height and weight are noted.
Asthma (non-occupational)

Correct
This girl has typical symptoms of childhood asthma.

137. Theme:Gynaecological Investigations

A CA 125

B Day 21 progesterone

C FSH/LH

D Hysterosalpingogram

E Laparoscopy

F Prolactin

G Transabdominal ultrasound

H Urodynamics

For each case below choose the single most appropriate investigation from the given list of
options.

A 68-year-old lady with persistent abdominal distention is found to have a likely ovarian mass on
ultrasound.
CA 125

Correct
In this age group ovarian cancer is the most likely diagnosis.

A CA 125, although not positive in all cases of ovarian cancer, is elevated in most of them.
CA 125 may be elevated in benign conditions such as endometriosis or pelvic inflammatory
disease and in fact any cause of ascites will mildly elevate it. It is however a useful tumour
marker and also useful for follow-up in the long term.Nowadays Ca125 is recommended
prior to scanning.Ref:http://www.patient.co.uk/doctor/Ovarian-Carcinoma.htm

A 36-year-old lady complaining of headaches, occasional blurred vision and nipple discharge.
Prolactin

Correct
The history is suggestive of a prolactinoma.

This proximity of the pituitary to the optic chiasm, may produce visula disturbance, in
particular a bitemporal hemianopia.

The hyperprolactinaemia is associated with galactorrhoea and amenorrhoea. Treatment is with


dopamine agonists such as bromocriptine.

A 26-year-old lady complaining of severe dyspareunia.


Laparoscopy

Correct
Laparoscopy is the correct answer. As causes of dyspareunia tend to be endometriosis, or
occasionally a retroverted uterus, neither of which will be particularly obvious on scan.

Also a laparoscopy will be helpful in noting the severity of the disease and planning further
treatment such as laser therapy and would therefore be the investigation choice.

It is important to understand the use of the different gynaecological investigations as there is


certainly an overlap. However, most gynaecological investigations involve an intimate
examination and/or an anaesthetic so their use needs to be carefully monitored. There are a
limited number of blood tests which should be reasonably easy to master.

138. Theme:The Full Blood Count

A Hb 102, MCV 83, WCC 5.5, Platelets 212

B Hb 104, MCV 75, WCC 7.3, Platelets 200


C Hb 105, MCV 110, WCC 3.1, Platelets 98

D Hb 122. MCV 72, WCC 5.6, Platelets 173

E Hb 135, MCV 99, WCC 8.2, Platelets 152

F Hb 137, MCV 93, WCC 6.6, Platelets 354

G Hb 185, MCV 88, WCC 10.2, Platelets 450

Select the most appropriate full blood count from the given list that would be typically found
in the following cases:

Normal ranges:

Haemoglobin (Hb) 120-180 g/L

MCV 80-95 fL

White cell count (WCC) 4-11 ×109/L

Platelets 150-400 ×109/L

A 54-year-old male presents with concerns over his drinking and its effects on his work. He
confesses to a long history of alcohol abuse and has spider naevi on his chest.
Hb 135, MCV 99, WCC 8.2, Platelets 152

Incorrect - The correct answer is Hb 105, MCV 110, WCC 3.1, Platelets 98
With alcohol excess, a high MCV would be expected as the alcohol has an effect on the
alkaline phosophatase in the RBC membrane producing a relative swelling of the cell -
macrocytosis. Low platelets occurs due to hypersplenism. Poor diet deficient in multiple
haematinics and chronic disease results in anaemia.

A 55-year-old female presents with tiredness and dyspepsia. She has been taking Brufen for
osteoarthritis of the right hip. On further questioning she states that she has recently noted that
her stools are darker.
Hb 104, MCV 75, WCC 7.3, Platelets 200

Correct
With chronic blood loss as suggested here, probably due to NSAID induced peptic
ulceration, an iron deficiency anaemia would be expected with a low MCV and Hb.
FBC is one of the commonly requested investigations and its interpretation can reveal much
about the patient.

139. Theme:Testicular disease

A Acute epididymitis

B Cysts of the epididymis

C Hernia

D Hydrocele

E Neoplasm of the testes

F Torsion of the testes

G Varicocele of the testes

From the given list choose the most likely diagnosis for the following:

A 28-year-old male presents with a small painless lump in his left testis. On examination the lump
lies within the testes and does not transilluminate.
Neoplasm of the testes

Correct

Testicular tumours are the most common in males between the age of 20-40. In 80% of
cases the patient notices a painless lump in one testis, or that one testis is larger than the
other.

A 32-year-old male presents with pain in his left testis. On examination his left testicle is red
tender and swollen. Prehn's sign is positive.
Acute epididymitis

Correct
Acute epididymitis mostly occurs in young males. Organisms may reach the epididymis by
retrograde spread from the prostatic urethra and seminal vesicles or less commonly, through
the bloodstream.

A 25-year-old army officer presents with a discomfort of his left testicle. On examination his
testicle feels like a bag of worms with a cough impulse.
Varicocele of the testes

Correct

A varicocele is a dilation of the pampiniform venous plexus and the internal spermatic vein.

A 14-year-old boy presents with severe pain in his right testicle. On examination the testis is
tender and high in the scrotum. Prehn's sign is negative.
Torsion of the testes

Correct

Torsion of the testes is a surgical emergency and typically presents with severe painful,
swollen and tender testes. Prehn's sign distinguishes between bacterial epididymitis and
testicular torsion. Scrotal elevation relieves pain in epididymitis but not torsion.

140. You are asked to review a 75-year-old lady who presents with lower abdominal pain,
frequency and dysuria.

On closer questioning, it appeared she had noticed debris in her urine as well as air at the
end of micturition. Past medical history included several admissions with diverticulitis.

Examination revealed mild suprapubic tenderness. No masses were palpable and rectal
examination was normal. A urine dipstick sample revealed nitrates and protein and an MSU
was dispatched.

What is the likely diagnosis?

(Please select 1 option)

Colovesical fistula Correct

Inguinal hernia
Perforated bladder

Renal colic

Urinary tract infection

The history of pneumaturia and faecuria is typical of a colovesical fistula.

An x ray, if performed, may show an air fluid level in the bladder (also seen in long term
catheterisation).

The common causes are diverticular disease, colonic carcinoma and inflammatory bowel
disease.

The patient requires investigation of the colon by flexible sigmoidoscopy or contrast study to
exclude a tumour and this may identify the fistula tract although this is not always seen.

The patient may have a history of recurrent urinary tract infections (UTIs) due to bacteria
migrating through the fistula from the colon to the bladder.

141. Theme:Surgical access to the abdomen

A Flank (oblique) incision

B Midline incision

C Paramedian incision

D Pfannenstiel incision

E Right subcostal incision

F Roof top incision

G Skin crease incision

H Transverse incision

I Umbilical incision

For each case below, choose the single most appropriate surgical incision from the given list
of options.
Each option may be used once, more than once, or not at all.

An 18-year-old lady undergoing caesarean section.


Pfannenstiel incision

Correct

A 21-year-old man undergoing urgent open appendicectomy. He has local peritonism, a pyrexia
and a raised white count.
Midline incision

Incorrect - The correct answer is Skin crease incision

A patient with a perforated duodenal ulcer admitted for emergency laparotomy.


Midline incision

Correct

A 61-year-old lady undergoing a Whipple's procedure for carcinoma of the pancreas.


Transverse incision

Incorrect - The correct answer is Roof top incision

A patient under going insertion of the first port for a laparoscopic cholecystectomy.
Umbilical incision

Correct

The choice of incision for any surgical procedure is determined by:

Its ability to gain access to the organs involved


The ability for the incision to be extended
Its ease of closure, and
Its cosmetic appearance.

The midline laparotomy incision allows access to most of the abdominal organs with incision
through the linea alba and safe en-masse closure.
142. Theme:Bleeding in early pregnancy

A Cervical ectropion

B Dysfunctional uterine bleeding

C Ectopic pregnancy

D Implantation bleed

E Inevitable miscarriage

F Placenta praevia

G Postcoital bleed

H Threatened miscarriage

For each case of bleeding in early pregnancy below, choose the single most likely diagnosis
from the list of options:

Slight painless vaginal bleeding at the expected time of menstruation.


Implantation bleed

Correct

This type of bleeding is quite typical of early implantation.

Vaginal bleeding at six weeks' gestation, associated with unilateral abdominal pain radiating to
the left shoulder tip.
Ectopic pregnancy

Correct

Unilateral abdominal pain with bleeding at six weeks' gestation is typical of ectopic
pregnancy.
Frequent vaginal spotting throughout the first trimester with a closed cervix and viable fetus in a
sexually active mother.
Cervical ectropion

Incorrect - The correct answer is Postcoital bleed

The increased vascularity/size of the cervix during pregnancy makes it more likely for
bleeding to occur during and following intercourse. The lack of any pathology here makes it
most likely that, of the options given, postcoital is probably the best selection.

Painless unprovoked vaginal bleeding at seven weeks' gestation. The external os is shut.
Threatened miscarriage

Correct

The painless bleeding at an early stage of pregnancy suggests a threatened miscarriage.

Painful vaginal bleeding at 10 weeks' gestation with an open cervical os.


Inevitable miscarriage

Correct

The open cervix indicates that miscarriage is inevitable.

Threatened miscarriage is painless vaginal bleeding with a closed cervix; inevitable is


bleeding associated with pain and an open cervical os.

Placenta praevia cannot be diagnosed before 24 weeks gestation.

Local causes of vaginal bleeding should always be considered, such as cervical ectropion
and polyps, which are common.

Dysfunctional uterine bleeding (DUB) cannot occur in pregnancy

143. Theme:Chest pain

A Acute myocardial infarction

B Aortic dissection

C Cholecystitis
D Costochrondritis

E Heartburn

F Hiatus hernia

G Hyperventilation

H Oesophageal spasm

I Pericarditis

J Pleurisy

K Stable angina

L Unstable angina

For each patient, select from the given list the most likely diagnosis of his/her chest pain:

A 71-year-old man reports chest tightness and pain in his throat when walking his dog after his
evening meal. He has had no previous symptoms. These symptoms have not changed since
they started two months ago. The resting electrocardiogram is normal.
Stable angina

Correct
These symptoms are classical of angina. There is typical exertional chest pain suggestive of
stable angina. This may be exacerbated after eating a meal due to the parasympathetic
diversion of blood to the GI tract, hence making angina more likely.

A 41-year-old woman with a family history of coronary artery disease, complains of a sharp pain
under her left breast followed by central upper chest tightness, prior to a heated discussion with
her boss about her working arrangements. During symptoms, she is also breathless and dizzy.
The routine electrocardiogram shows a sinus tachycardia.
Hyperventilation

Correct
This case has anxiety related to thoughts that she may have coronary disease as suggested
by the resting tachycardia but nil else. Coronary artery disease would be rather unusual in a
woman of this age.
A 37-year-old man has had fever and malaise for five days. For the past 12 hours he has had
severe left-sided chest pain which is exacerbated by movement or respiration. The 12-lead ECG
shows T wave inversion in aVR.
Pleurisy

Correct
This case has pleurisy rather than any heart disease as suggested by the five day history of
malaise and the chest pain on movement or respiration. T wave in aVR is non-specific.

A 64-year-old man has developed sudden severe pain between his shoulder blades while
digging in his garden. There is no past medical history. Clinical examination is normal except that
he is overweight, sweaty and distressed. Transthoracic echocardiography demonstrates left
ventricular hypertrophy.
Aortic dissection

Correct
The history of severe chest pain radiating through to the back during exertion should prompt
the diagnosis of aortic dissection. The presence of hypertension (LVH) would be a risk factor
and a transthoracic echo may not adequately show the aorta.

A 58-year-old woman has a three week history of episodes of central chest pain while lying flat,
which is relieved when she sits up. Similar symptoms can occur if she gets upset. The resting
electrocardiogram shows T wave inversion in leads, aVL and V3-6.
Unstable angina

Correct
This case has a history suggestive of unstable angina with angina decubitus (that is, chest
pain provoked by lying flat) and is further supported by the ECG changes, which suggest
inferolateral ischaemia. Although pericarditis may be considered, the ECG changes
associated with this condition are ST segment elevation (concave upwards) in incongruous
leads.

144. Which of the following are features of neurofibromatosis?

(Please select 1 option)

Cushing's syndrome

Haemangiomas of the retina Incorrect answer selected

Nephroblastoma
Osteosarcoma

Scoliosis This is the correct answer

Features of neurofibromatosis/von Recklinghausen's disease include:

neurofibrosarcomas
phaeochromocytoma
optic nerve tumours
scoliosis, and
acoustic neuromas.

145. A 25-year-old woman is admitted with abrupt onset of a severe occipital headache
whilst decorating at home. This was associated with vomiting and transient loss of
consciousness.

On examination her Glasgow coma scale (GCS) score is 15, she is afebrile, has a
blood pressure of 146/84 mmHg and a pulse rate of 70 beats/minute. Her physical
examination is normal apart from an abrasion over her right temple.

What is the single most appropriate initial management?


(Please select 1 option)

Admit for head injury observation

Carboxyhaemoglobin level

Computed tomography (CT) brain scan Correct

Lumbar puncture

Skull x ray

The history of sudden onset of severe headache in a young woman is suggestive of


subarachnoid haemorrhage. The associated vomiting and loss of consciousness add
weight to this.

The investigation of choice is a CT scan of the brain looking for subarachnoid blood.

If this is normal a lumbar puncture should be performed as a CT scan will miss


approximately 10% of cases of subarachnoid haemorrhage.
146. A 31-year-old woman who is 5 months pregnant presents with a 2 month history of a
slowly growing swelling in her neck.

On examination the thyroid is enlarged and feels smooth. Thyroid function tests show the
following:

Free T4 19 pmol/L (10-22)

TSH 5.4 mu/L (0.4-5.0)

Which of the following is the most appropriate diagnosis?

(Please select 1 option)

Graves' disease

Muultinodular goitre

Simple colloid goitre Correct

Thyroglossal cyst

Thyroid adenoma

The finding of an asymptomatic smooth goitre, with normal thryoid function tests is strongly
suggestive of a simple colloid goitre. This occurs most commonly due to iodine deficiency,
but can occur in pregnancy or puberty where the demand for thyroxine is increased.

The gland produces more colloid in response to the stimulus of TSH released by the low
levels of T4, and hence the finding of a normal T4 and a slightly raised TSH.

147. 23-year-old woman is admitted with pain on passing urine and fever. She says she
has been unwell for the past three days and has now developed left loin pain. Only
medication of note is the combined oral contraceptive pill.

On examination she is pyrexial 38.8°C and has a BP of 105/60 mmHg, her pulse is 94 and
regular. She has left renal angle and supra-pubic tenderness.

Investigation shows:

Haemoglobin 114 g/L (115-165)

White cells 14.9 ×109/L (4-11)

Platelet 193 ×109/L (150-400)


Sodium 140 mmol/L (135-146)

Potassium 4.1 mmol/L (3.5-5)

Creatinine 132 µmol/L (79-118)

Urine blood, nit+, leuk++

Which of the following is the most appropriate first action?

(Please select 1 option)

Advise her to take cranberry juice and discharge

Arrange an outpatient CT of the abdomen

IV antibiotics Correct

Start trimethoprim and discharge

Ultrasound renal tract

Given her raised white count, elevated creatinine and significant hypotension, IV antibiotics
with a broad spectrum cephalosporin should be given immediately.

The left renal angle pain raises the possibility of pyelonephritis and significant ureteric
obstruction therefore needs to be excluded and many urologists suggest USS renal tract in
first episode of pyelonephritis

The best way to do this is by ultrasound, looking for signs of obstruction. If ureteric dilation is
seen then an opinion from a urologist should be obtained.

148. A 20-year-old female horse rider was brought into the emergency department on a
spinal board having fallen off her horse. She was complaining bitterly about being
restrained on the spinal board because her back was hurting.

On examination, she had a pulse of 120/min, blood pressure of 84/30 mmHg, and a normal
chest examination. Abdominal examination showed a bruise and tenderness on her left
hypochondrium and lumbar regions. She had a decreased sensation below her knees and
she could not move her toes.

What is the most appropriate management for this patient?

(Please select 1 option)

CT thorax and abdomen


Laparotomy

Large bore IV access and fluid resuscitation Correct

Log roll and rectal examination

Decompression of spinal haematoma

In situations of trauma, it will help to remember ABCDE.

In all circumstances management of airway comes before breathing; breathing before


circulation;circulation before dysfunction/disability,disability before exposure and secondary
survey.

This patient probably has a spinal injury and may also have a splenic haemorrhage.
However, she is haemodynamically compromised and so before proceeding further she
needs venous access and fluid resuscitation.

149. This is the retina of a 70-year-old man with tunnel vision.

What is the diagnosis?

(Please select 1 option)

Choroidal melanoma

Laser photocoagulation scars

Macular degeneration

Papilloedema

Retinitis pigmentosa Correct


There are clumps of pigmentation in the retina in a pattern of 'bone spicules' typical of
retinitis pigmentosa

150. 60-year-old gentleman is seen complaining of 'chest pains'.

On further discussion he reports post-prandial retrosternal burning that has been present for
the last three to four months. It has been persistent and occurring daily over that time. He
reports that he has used an over-the-counter alginate antacid at least once a day for the last
two to three months and it has not really provided significant benefit.

He is usually fit and well with no significant past medical history. He is a smoker of 20 a day
and has done so since the age of 15. He drinks alcohol occasionally.

He reports that his swallow is normal with no dysphagia or odynophagia. His weight is
stable. He has had occasional associated vomiting when symptomatic but no persistent
vomiting. He denies any haematemesis and his bowel habit is stable with no rectal bleeding
or black stools.

On examination he has some angular stomatitis and mild glossitis. His abdomen is soft and
non-tender with no palpable masses.

What is the most appropriate management strategy?

(Please select 1 option)

Advise he takes the alginate preparation after meals TDS and also at night

Prescribe a daily high dose proton pump inhibitor for eight weeks (for example, omeprazole) then
review

Prescribe a daily H2 receptor antagonist (for example, ranitidine) for four weeks then review

Refer routinely for endoscopy

Refer urgently under the two week wait referral system Correct

When managing someone with symptoms of gastro-oesophageal reflux disease (GORD) it is


important to look for the so-called 'alarm' features:

unintentional weight loss


dysphagia
GI bleeding
persistent vomiting, and
signs of anaemia.

New onset GORD symptoms in someone over the age of 55 is also a feature that should
raise concern.
In this case the patient has new onset GORD and is over the age of 55. He also has
examination findings that suggest iron deficiency anaemia. Of course, a full blood count
would be requested to look into this further, but the most appropriate approach would be to
refer this gentleman urgently for further assessment.

In the absence of alarm features, mild GORD symptoms that occur less than once a week
can managed with antacids when required. In patients with more frequent symptoms,
especially those who suffer on a daily basis, a proton pump inhibitor (PPI) can be used.
Initial treatment of GORD is with a high dose PPI for eight weeks. A PPI should be taken
once daily 30-60 minutes before the first meal of the day.

In patients without alarm features who display a partial response, you can increase the dose
to twice daily (e.g. omeprazole 40 mg BD). If nocturnal symptoms are troublesome the timing
of the dose can be adjusted.

If there is a no response to PPI treatment then you should reconsider the diagnosis and
consider specialist referral. A H2 receptor antagonist can be added to a PPI for patients who
have a partial response to PPI treatment.

151. A 52-year-old banker has recurrent episodes of short lived, severe pains in his jaw on
the left. These cause him to screw his face up.

He has seen a dentist who thinks that the most likely cause for his problem is trigeminal
neuralgia. He seeks your advice.

Which of the following are not true of this condition?

(Please select 1 option)

Around 40% of cases are bilateral Correct

It affects women more commonly than men

It can be a manifestation of multiple sclerosis

It may respond to phenytoin treatment

Onset is most commonly after the age of 50

This is Trigeminal neuralgia:

More common after the age of 50


More common in women than men
Paroxysmal attacks lasting seconds - affect any of the three divisions of the trigeminal
nerve
Triggered by touching the face, chewing or speaking
Treated with carbamazepine or phenytoin, may respond to surgical microvascular
decompression.

Differentiate from
Postherpetic neuralgia
Structural lesion V
Cluster headache
Migraine
Tempero-mandibular joint pain

by length / location of attacks.

152. A 13-year-old girl is rushed into hospital, having become rapidly drowsy after running
the 1500 metres in the school athletics competition.

She has been on insulin for diabetes for three years. Her latest HbA1c is 62 mmol/mol.

On examination she responds to pain, and is pale and sweaty. Her temperature is 36.5°C,
Respiratory rate 12/min and heart rate 80/min. There are no focal neurological findings.

What is the most likely diagnosis?

(Please select 1 option)

Hypoglycaemia Correct

Postural hypotension

Substance abuse

Vasovagal episode

Viral encephalitis

The history suggests tight diabetic control, with neurological deterioration following exercise.

Hypoglycaemic coma is most likely, and the sweatiness and pallor are suggestive.

The BM stix in this case was 1.2 mmol/L and she recovered rapidly with a bolus of IV 10%
dextrose.

153. A 40-year-old woman had upper respiratory tract infection with a productive cough
two weeks ago. She reports burning in her chest during the coughing.

One week ago she coughed up about one teaspoonful of yellow sputum that contained
flecks of blood. The patient coughed up a small amount of blood-tinged sputum again the
next morning but has had no subsequent haemoptysis. She is beginning to feel better, and
the cough seems to be resolving. She has never smoked cigarettes and has no history of
respiratory problems.
The patient appears healthy with no abnormalities to find on chest, heart or abdominal
examination. Chest x ray is normal.

Which of the following would you now recommend?

(Please select 1 option)

CT of the chest

Fibreoptic bronchoscopy

Indirect laryngoscopy

Observation only Correct

Sputum cytology

Acute bronchitis is a lower respiratory tract infection that causes reversible bronchial
inflammation. In up to 95 % of cases, the cause is viral

While antibiotics are often prescribed for patients with acute bronchitis, little evidence shows
that these agents provide significant symptomatic relief or shorten the course of the illness.

Viruses that cause acute bronchitis include:

Adenovirus
Coronavirus
Influenzaviruses A and B
Parainfluenza virus
Respiratory syncytial virus
Coxsackievirus A21
Rhinovirus, and
viruses that cause rubella and measles.

In this case there is no evidence of bronchoconstriction or bacterial infection and at the same
time the patient is not in respiratory distress, so observation is advised.

154. You see a 12-year-old boy in the Emergency department who is attending with his
concerned parents.

He has a four hour history of an exquisitely tender left testicle with pain radiating towards the
umbilicus, and also describes a sensation of nausea. His testis is of normal size but is
extremely tender.
You inform the registrar who is scrubbed in theatre. He explains he will be an hour and asks
you to follow the department protocol.

What is the most appropriate plan of action?

(Please select 1 option)

Administer antibiotics after collecting a urine specimen

Arrange a testicular ultrasound

Prepare the patient for testicular exploration and bilateral orchidopexy and inform theatres Correct

Reassure the patient and his parents that this is probably due to a virus

Submit a referral to Urology

Torsion is a urological emergency and delay beyond six hours is associated with an
increased risk of testicular ischaemia.

All patients with a history suggestive of torsion should be booked without delay for
exploration of the painful testis followed by three point orchidopexy of the reduced testis and
prophylactic pexy of the normal testis

155. A 55-year-old gentleman is admitted with central colicky abdominal pain. The pain
has been present for six days and is getting worse. He has also noticed the passage of
watery diarrhoea for three days prior to presentation.

Past medical history includes angina and peripheral vascular disease and he is a current
smoker. Examination reveals generalised tenderness which is most marked over the left
colon.

An abdominal x ray study is performed:


What is the likely diagnosis?

(Please select 1 option)

Colonic carcinoma

Crohn's disease

Infective colitis

Ischaemic colitis Correct

Ulcerative colitis

The abdominal x ray shows evidence of mucosal oedema and thumb printing typical of
ischaemic colitis.

Given the history of arterial disease (cardiac and peripheral) it is likely that the pathology is a
chronic progressive atherosclerotic lesion at the origin of the superior mesenteric artery.

The patient requires investigation to confirm the diagnosis and management may require
revascularisation of the ischaemic bowel.
156. A 43-year-old man, known to be of no fixed abode, was found to have pulmonary TB
and was treated with quadruple therapy. He developed a severe hepatitis and died.

Which of the following anti-tuberculous drugs is least likely to cause a hepatitis?

(Please select 1 option)

Ethambutol Correct

Isoniazid

Pyrazinamide

Rifampicin

Thiacetazone

Rifampicin commonly causes some mild hepatitis, usually just consisting of a raised alanine
aminotransferase (ALT).

Rash, gastrointestinal upset and thrombocytopenia are all well-recognised with rifampicin.

Isoniazid causes hepatitis in about 1% of cases. This is more common in people over 35,
especially in the elderly. The main side effect of isoniazid is peripheral neuropathy however,
which is usually preventable with pyridoxine (vitamin B6) 10mg od.

Pyrazinamide often causes some raised transaminases but seldom causes severe toxicity
except in people with pre-existing liver disease. The main side effect of pyrazinamide is gout
(it inhibits renal excretion of uric acid).

Ethambutol's main effect is optic neuritis - there is nothing to link it with hepatitis.

Thiacetazone (which inhibits mycolic acid synthesis) can cause hepatitis but its famous side
effect is Stevens-Johnson syndrome, which occurs in less than 0.5% of HIV negative but in
5% of HIV positive people, with a 3% mortality.

If the liver function tests are properly disturbed all anti-TB drugs should be stopped and
reintroduced one by one adding each in for five days before adding the next one.

157. A 32-year-old female complains of foul smelling vaginal discharge. Smear


examination shows numerous squamous cells and clue cells.

What is the most likely diagnosis?

(Please select 1 option)

Bacterial vaginosis This is the correct answer

Candidiasis
Gonococcus

Herpes simplex

Trichomonas vaginalis Incorrect answer selected

Here there is a foul smelling discharge with smear demonstrating squamous cells and clue
cells.

'Clue cells' are irregularly-bordered squamous epithelial cells whose cell outlines are
obliterated by sheets of small bacteria. These suggest a diagnosis of bacterial vaginosis
rather than Trichomonas, as with the latter motile flagellated organisms should be seen.

Like Trichomonas, bacterial vaginosis should be treated with metronidazole.

158. Which of the following statements about epistaxis is correct?

(Please select 1 option)

Always responds to silver nitrate cautery

Commonly cause anaemia in children

Is often due to angiodysplasia in middle aged adults Correct

Is usually from the lower septum in adults

Nearly always due to untreated hypertension

Often caused by anticoagulants

Little's area is a mesh of blood vessels on the lower septum and is the usual site of bleeding
in children.

Recurrent epistaxis does not cause anaemia in children unless there is a predisposing cause
such as a haematological deficiency or disease.

In adults the bleeding may be septal but is usually higher up than Little's area and is often
due to angiodysplasia.

Hypertension and anticoagulation may make bleeding worse but are not usually the cause.

Treatment with cautery is often successful, but other measures are often necessary with
torrential bleeding.

159. Regarding preimplantation genetic diagnosis (PGD) which statement is incorrect?

(Please select 1 option)


It is not widely available

It is strictly controlled by the Human Fertilisation and Embryology Authority

It usually involves the woman taking large doses of follicular stimulating hormone

The procedure should only be considered in families who would not contemplate a therapeutic
termination for an affected fetus Correct

The technique can be used to select the sex of the embryo

PGD has been possible for several decades.

It involves the careful removal of a cell from an embryo at around the 8 cell stage, detailed
analysis (karyotype or mutation screen for a known mutation), followed by the return of
unaffected embryos.

PGD has become much more available with the advent of in-vitro fertilisation techniques as
this provides several embryos for testing which do not need removal from the
uterus/fallopian tubes. Thus the recipient of treatment undergoes the same procedure as IVF
in terms of drug administration.

FSH is administered to stimulate superovulation.

PGD is tightly controlled by the HFEA and is only available in a limited number of specialised
centres. It can be used to ascertain the sex of a fetus (thaat is, for X linked recessive
diseases).

It is unethical to offer PGD only to those who would not undergo a therapeutic termination for
an affected fetus as it is the right of parents to choose how they want to proceed.

160. Which of the following is least likely to be a feature of herpes simplex encephalitis ?

(Please select 1 option)

Absence of fever Correct

Clinical evidence of mucocutaneous HSV

Neurological sequelae after recovery

Origin due to subtype 1 HSV

Temporal lobe involvement on MRI

Herpes simplex encephalitis is a rare disease (approximately 1 case in half a million people
a year) although we often "cover" for the possibility.

Neurological sequelae occur in around a third of cases. Fever is normally present. (The first
answer is correct, note the double negative!).
Typically the temporal lobes are involved with temporal lobe signs, and MRI is diagnostically
useful.

Most people with herpes simplex virus (HSV) encephalitis show clinical or serological
evidence of HSV-1 infection, and subtype 1 virus accounts for 95% of cases.

Brain biopsy is the "gold standard" for diagnosis but in practice a positive polymerase chain
reaction (PCR) for HSV in cerebrospinal fluid confirms the diagnosis.

Treatment is with intravenous aciclovir. Many more "cases" are treated than actually occur,
but this is appropriate as early treatment is safe and improves prognosis.

161. Which of the following drugs is matched correctly with its action?

(Please select 1 option)

Amiodarone shortens the action potential

Digoxin stimulates Na/K ATPase

Simvastatin inhibits HMGCoA reductase Correct

Sotalol has class I and class III properties

Streptokinase increases fibrinogen levels

Sotalol has class II (beta-blocker) and class III (amiodarone) like actions.

Amiodarone prolongs the action potential and the QT interval.

Both sotalol and amiodarone carry the risk of precipitating ventricular arrhythmias.

After thrombolysis, fibrinogen levels are low, due to consumption during thrombosis.

Digoxin inhibits the cardiomyocyte membrane Na/K ATPase.

Next question

162. Which of the following statements regarding thyrotoxicosis is correct?

(Please select 1 option)

Hyperemesis gravidarum rarely affects thyroid blood tests

In pregnancy carbimazole is preferable over propylthiouracil

Most patients will need radioiodine at some point

The time course of active eye disease has a predictable pattern Correct
TSH towards the upper limit of normal excludes the diagnosis

The time course of eye disease has an active then plateau phase totalling approximately two
years.

Radioiodine, if given alone, may worsen hyperthyroidism - if necessary, steroids are given at
the same time.

Propylthiouracil is commonly used in pregnancy; regular review of patients is required.

Hyperemesis affects thyroid function tests.

A thyroid-stimulating hormone (TSH) towards the upper limit of normal may underlie a (rare)
TSH-oma.

163. An 8-month-old child presents with spots on the legs. He is well and feeding well.
39+6/40 3.5 kg, no neonatal problems. No drugs nor medications, fully immunised. No
FH/SH of note.

On examination temperature 37.6°C (tympanic), RR 30/min, HR 110/min. Well perfused,


capillary refill time of 1 second. There are 20-30 1-2 mm non-blanching purpuric spots over
the shins.

What is the most likely diagnosis?

(Please select 1 option)

Child physical abuse

Cough petechiae

Enteroviral infection Correct

Henoch-Schönlein purpura

Idiopathic Thrombocytopaenic purpura

This child is well, and presents with purpuric spots and a low-grade fever. Although about
20% of such children have serious bacterial infection and 7-10% have meningitis/
septicaemia, this still leaves 70% who have some sort of viral infection.

A large number of viruses (e.g. Varicella and EBV) can present in this way, although in
clinical practice the specific cause is rarely found.

In ITP there is often a preceding viral illness 2-4 weeks before, the child is usually pre-school
age, and the petechiae are more marked.
164. A previously healthy 71-year-old man describes visual loss in his right eye preceded
by a intermittent flashes and a curtain-like loss of lateral vision which began when he
awoke this morning and has deteriorated.

Which of the following is the most likely explanation?

(Please select 1 option)

Atheroembolic occlusion of a lateral branch of the right retinal artery Incorrect answer selected

Occipital lobe seizure

Ocular migraine

Retinal detachment This is the correct answer

Retinal vein occlusion

Retinal detachment occurs when the retina's sensory and pigment layers separate and is
one of the most time-critical eye emergencies.

It may be associated with:

congenital malformations
metabolic disorders
trauma (including previous ocular surgery)
vascular disease
high myopia
vitreous disease, and
degeneration.

Floaters, grey curtain or veil moving across the field of vision and sudden decrease of vision
are suggestive symptoms.

Floaters, grey curtain or veil moving across the field of vision and sudden decrease of vision are
features of retinal detachment and it's more common in high myopes.

165. A 72-year-old man is referred with a two month history of progressive disorientation
and falls.

Two weeks beforehand, he locked his wife out of their house, claiming that she was trying to
steal his clothes. He had also telephoned the police in the middle of night, claiming that he
could see men hiding under his bed.
On examination, his face is expressionless, his speech is quiet and monotonic. There are no
cranial nerve palsies, otherwise. Increased tone is present in all four limbs, with a slow
festinant gait. Reflexes, power and sensation are all normal. Halfway through your
examination he tells you that he is leaving the room, because of the "lobsters coming
through the window". Unfortunately, therefore, formal cognitive testing and basic
investigations cannot be performed.

On the basis of this evidence, what is the most likely diagnosis?

(Please select 1 option)

Lewy body dementia Correct

Multiple system atrophy

Normal pressure hydrocephalus

Parkinson's disease

Progressive supra-nuclear palsy

This man has parkinsonism, with bradykinesia and rigidity.

The florid visual hallucinations and paranoid ideation make Parkinson's disease unlikely.

The normal eye movements and normal postural blood pressure mitigate against a
parkinsonism plus syndrome, while the lack of incontinence and gait abnormalities make
normal pressure hydrocephalus less probable.

Parkinsonism with dementia, paranoia and visual hallucinations is found in dementia with
Lewy bodies.

166. Which of the following is characteristic of cluster headache?

(Please select 1 option)

Attacks are more common during the day

Inhaled oxygen can precipitate an attack

Is associated with lacrimation Correct

Is exacerbated by sumatriptan

Is unlikely when accompanied by a Horner's syndrome

Cluster headache causes severe, usually unilateral, pain felt in the region of the eye and
forehead associated with lacrimation, conjunctival injection and occasionally transient
Horner's syndrome.
Attacks are more common at night, often waking sufferers at the onset of REM sleep.

Oxygen and triptans may arrest attacks.

Prophylaxis with propranolol, pizotifen, and especially verapamil may prevent further
episodes.

167. A newborn baby presents with rudimentary digits, limb hypoplasia and convulsions.

What is the single most probable diagnosis?

(Please select 1 option)

Cytomegalovirus (CMV)

Herpes simplex

Listeriosis

Toxoplasmosis

Varicella Correct

Chicken pox infection within the first 20 weeks of pregnancy may result in the congenital
varicella syndrome. This is characterised by:

Cerebral cortical and cerebellar hypoplasia


Microcephaly
Convulsions
Limb hypoplasia, and
Rudimentary digits.

Prevention is by administering varicella vaccine, even before pregnancy. Varicella


immunoglobulin is administered to pregnant women who are exposed to infection.

Infection during pregnancy is treated with aciclovir.

168. You see a 24-year-old homeless man.

He is listless, cachectic and anorectic. He is bleeding from his painful gums and nose. He
has small haemorrhages around hair follicles and cockscrew hairs.

What is the most likely diagnosis?

(Please select 1 option)

Platelet function
Vitamin B1 deficiency

Vitamin B6 deficiency

Vitamin B12 deficiency

Vitamin C deficiency Correct

Vitamin C (ascorbic acid) deficiency is usually dietary in origin. Historically sailors who spent
months at sea would develop scurvy due lack of fresh fruit and vegetables in the diet.

Symptoms include bleeding gums, but bleeding could also be into joints, bladder and gut.
Perifollicular changes result in the corkscrew hair appearance.

No test is completely satisfactory, but WBC ascorbic acid is low.

Treatment is with oral ascorbic acid and dietary improvement.

169. In addition to hypoglycaemia, relatively more common organic causes of anxiety


include

Alcohol withdrawal
Drug intoxication or withdrawal
Thyroxine
Paroxysmal supraventricular tachycardias.

Phaeochromocytoma is rare.

Carcinoid does not cause anxiety.

Carcinoma of the bronchus and hyperparathyroidism are more likely to present with
depression.

170. 35-years-old woman is referred with a microcytic anaemia.

She has a history of ulcerative colitis.

She has the following results:

Hb 88 g/L (120-160)

WCC 3.6 ×109/L (4-11)

Platelets 222 ×109/L (150-400)

MCV 70 fL (80-96)
Haptoglobins <0.04 g/L (0.13-1.63)

Liver function test:

Lactate dehydrogenase 850 U/L (100-250)

Bilirubin 68 µmol/L (1-22)

Alkaline phosphatase 100 U/L (45-105)

ALT 23 U/L (5-40)

What investigation would confirm the underlying diagnosis?

(Please select 1 option)

Blood film

Direct Coombs' test Correct

Fibrin degradation products

Indirect Coombs' test

Ultrasound scan

This is autoimmune haemolytic anaemia, a rare complication of Salazopyrin therapy.

Indeed there are relatively few patients with inflammatory bowel disease (IBD) on this drug
now. The sulphasalazine drugs are much more common.

A direct Coombs' test looks for erythrocytes already coated with antibody, whereas the
indirect test is used to detect potential red cell antibody interactions.

171. A patient on clinical examination is found to have a third heart sound.

Which of the following statements is true?

(Please select 1 option)

Is lost in AF

It is accentuated by expiration

It is lost in constrictive pericarditis


It may be a normal finding in women up to the age of 50 Correct

It only arises from the left ventricle

The third heart sound is caused by early diastolic filling due to ventricular relaxation, 0.14-
0.16 seconds after closure of the aortic valve (corresponds to Y descent in JVP).

It may arise from either ventricle and is a low-pitched sound accentuated by inspiration.

It is most commonly heard in cardiac failure but is also prominent in constrictive pericarditis
('knock') and with atrial myxomas ('tumour plop').

It is a normal finding in children and young adults and may persist in women up to the age of
50.

S4 corresponds to ventricular filling in atrial systole (a wave in JVP), is never normal and is
lost in AF

172. A 30-year-old man with mild learning difficulties has bilateral wasting of temporalis
muscle, he has bilateral cataracts and temporal recession of the hair line.

On shaking hands he is slow to withdraw his hand.

Which feature is not seen in his condition?

(Please select 1 option)

Arrhythmias

Dysphagia

Hypogonadism Incorrect answer selected

Hypothyroidism This is the correct answer

Insulin resistance

The description is suggestive of the autosomal dominant disorder, myotonic dystrophy.

Non-striated muscle manifestations include:

Cardiac muscle - arrhythmias, cardiomyopathy


Smooth muscle - dysphagia (oesophageal involvement), constipation, incontinence
CNS - low IQ, daytime somnolence
Endocrine - testicular atrophy, reduced fertility, male pattern baldness, glucose
intolerance
Eyes - cataracts.
173. A 50-year-old gentleman is admitted following an episode of collapse associated with
loss of consciousness.

He is noted to have a prolonged QT interval on the ECG.

Which statement is incorrect regarding QT prolongation?

(Please select 1 option)

Characteristically predisposes to atrial fibrillation Correct

Is a feature of treatment with amiodarone

Is a recognised feature of sotalol therapy

Is a recognised feature of treatment with cisapride

Predisposes to ventricular tachycardia

Drug causes of long QT interval include:

Anti-arrhythmics: quinidine, procainamide, sotalol, amiodarone, bretylium;


Other drugs: Tricyclic antidepressants, phenothiazines, erythromycin, astemizole,
terfenadine, pentavalent antimonials - sodium stibogluconate.

Although causing QT prolongation, amiodarone reduces QT dispersion and therefore may


have a stabilising effect.

Cisapride was withdrawn in July 2000 due to serious arrhythmias.

The Medicines Control Agency (MCA) has recently restricted the indications for thioridazine
and droperidol has been discontinued, because of their effects on the QT interval and
potential for serious arrhythmias.

174. Poor prognostic indicators in the first 48 hours of acute pancreatitis include:

Age more than 55 years


White cell count (WCC) more than 15
Glucose more than 10
Urea more than 16
Albumin less than 30 g/L
Alanine aminotransferase (ALT) more than 200
Calcium less than 2
Lactate dehydrogenase (LDH) more than 600 and
Partial pressure of oxygen in arterial blood (PaO2) less than 8.

175. A 62-year-old man presented with difficult walking.


He had a past history of diabetes mellitus and cervical spondylosis, which had required
surgical decompression eight years previously.

He drank 40 units of alcohol weekly. On examination there was fasciculation, wasting and
weakness in the left deltoid and biceps, with weakness in the shoulder girdle muscles
bilaterally. There was fasciculation in the glutei and quadriceps bilaterally, weakness of hip
flexion and foot dorsiflexion, brisk reflexes in upper and lower limbs and extensor plantar
responses. There was no sensory impairment.

What is the diagnosis?

(Please select 1 option)

Alcoholic myopathy

Diabetic amyotrophy

Motor neurone disease Correct

Recurrent cervical cord compression

Syringomyelia

The clinical signs are of lower (wasting, fasciculations) and upper (brisk reflexes, extensor
plantar response) motor neuron involvement in the presence of normal sensation.

Motor neuron disease is the most common cause of such a presentation.

Alcoholic myopathy and diabetic amyotrophy do not share upper motor neuron signs.

Syringomyelia presents with sensory symptoms and signs (spinothalamic).

You expect sensory involvement with cervical cord compression.

176. A 35-year-old student presents with breathlessness.

Lung function tests show the TLCO to be very low, however the KCO is 190% predicted.

Which of the following is the most likely diagnosis?

(Please select 1 option)

A patient with the ZZ genotype for alpha 1-antitrypsin

Hereditary haemorrhagic telangiectasia

Neuromuscular chest wall disorder Correct

Primary pulmonary hypertension (PPH)


Scleroderma

In patients with extra-pulmonary restriction (for example, neuromuscular chest wall


disorders) the lungs cannot fully inflate.

Thus the surface area available for gaseous exchange is decreased (low TLCO). However,
the cardiac output is unchanged so that a higher density of blood per unit volume is obtained
resulting in a raised KCO.

The diffusing capacity of the lungs for carbon monoxide (DLCO) is a test of lung function. It
measures the ability of the lungs to transfer gas from inhaled air to the red blood cells in
pulmonary capillaries.

The gas transfer co-efficient (KCO) across alveoli is calculated by measuring carbon
monoxide uptake from a single inspiration in a standard time (usually 10 s).

It is reduced in emphysema and interstitial lung disease but high in alveolar haemorrhage.

177. Which of the following abnormal facial movements is not a well recognised
association?

(Please select 1 option)

Blepharospasm - Progressive supra-nuclear palsy

Facial synkinesis - Wilson's disease Correct

Facial tics - Tourette's syndrome

Myokymia - Multiple sclerosis

Tardive dyskinesia - Chronic neuroleptic use

The following causes of abnormal facial movements are recognised;

Dyskinesia: Repetitive, abnormal movements of the mouth and lips - neuroleptics, ageing
Dystonia: Prolonged dystonic contracture of facial muscles; tardive dyskinesia (including
secondary to antipsychotics); Wilson's disease; midbrain and thalamic lesions
Tics: Brief twitches; Tourette's
Synkinesis: Abnormal voluntary movements, but retained involuntary movements of face,
unilaterally; any facial nerve lesion; bag of worms appearance; brainstem lesion; MS;
bilateral spasms of orbicularis oculi; PSP; midbrain and thalamic lesions
Blepharospasm - Blinking, progresses to clonic and later tonic (sustained) contractions of
the orbicularis oculi leading to forceful closure of eyelids; associated with tardive
dyskinesia; disease of basal ganglia including PSP
Myokymia: Rippling muscular movement without joint movement; MS; drugs; exercise.
178. In considering the anatomical location of intracranial meningiomas, which relation is
well recognised?

(Please select 1 option)

Chiasmal - monocular blindness

Olfactory groove - unilateral exophthalamus

Parasagittal - spastic paraparesis Correct

Sphenoid ridge - anosmia

Temporal - urinary incontinence

Localisation of intracranial lesions on the basis of history and examination is important, even
where detailed scanning is available.

Meningiomas are slow growing and may exert subtle effects that are distinct from the typical
lobar pattern of more aggressive, intrinsic lesions.

Olfactory groove lesions affect sense of smell and may produce ipsilateral optic atrophy.

Sphenoid ridge lesions will produce exophthalamus.

Chiasmal lesions typically produce bitemporal hemianopia.

179. Which of the following is most true about repaglinide?

(Please select 1 option)

Has a long duration of action

Is superior to glibenclamide in improving postprandial glycaemic control Correct

It is contraindicated if the patient is on metformin

Must be taken at regular times

Reduces peripheral insulin resistance

Repaglinide is a novel insulin secretagogue that was developed as a prandial glucose


regulator for the treatment of people with type 2 diabetes mellitus. It is aimed at restoring the
first-phase insulin response that follows consumption of a meal, which is missing in patients
with type 2 diabetes.

Although control of the overall glycaemic load is the most important factor for the success of
long term management of type 2 diabetes, control of postprandial hyperglycaemia also has
positive implications for preventing the development of diabetic complications.

Repaglinide is used flexibly, taken prior to meals on a 'one meal, one tablet; no meal, no
tablet' basis. Repaglinide is quickly absorbed appearing in the bloodstream within 15 to 30
minutes of dosing. It stimulates short-term insulin release from the pancreatic beta-cells by
binding to a unique site on the beta-cell membrane. It is rapidly eliminated so that
postprandial insulin levels quickly return to preprandial levels reducing the risk of
hypoglycaemia (cf sulphonylureas).

It is effective in patients who have not previously been treated with an oral antidiabetic
agent, significantly reducing glycosylated haemoglobin (HbA1c) levels by 1.6%.

Its biliary route of elimination makes it suitable for use in patients with renal impairment
(although careful titration of repaglinide dose is recommended for patients with mild to
moderate renal impairment). No dosage adjustment is otherwise needed in the elderly.

Repaglinide has few contraindications or drug interactions and can be used in a wide range
of patients.

Short-term clinical studies showed that repaglinide is superior to glibenclamide in improving


postprandial glycaemic control. Longer term studies confirmed that the overall glycaemic
control is at least equivalent to that achieved by sulphonylurea treatment.

Repaglinide is also effective in combination therapy: acting synergistically with metformin.

180. A 34-year-old office worker attends the casualty department complaining of


increasing problems with constipation.

Over the past few months he has suffered severe dyspepsia and has been taking large
amounts of antacids. Examination is unremarkable apart from a loaded rectum.

Which of the following is the likely cause of his symptoms?

(Please select 1 option)

Aluminium hydroxide This is the correct answer

Calcium carbonate Incorrect answer selected

Cimetidine

Magnesium trisilicate

Sucralfate

Aluminium and magnesium containing antacids such as magnesium carbonate, hydroxide


and trisilicate and aluminium glycinate and hydroxide, being relatively insoluble in water, are
long acting and retained in the stomach.

They give good symptomatic relief but rarely effect healing of the cause of the dyspepsia.

Magnesium containing antacids tend to be laxative while aluminium containing preparations


may be constipating especially if taken in large quantities.

Aluminium accumulation is not a risk if renal function is normal.


181. In the treatment of an individual with type 1 diabetes who presents in DKA (diabetic
ketoacidosis) with a pH of 7.1, which of the following statements is correct?

(Please select 1 option)

Bicarbonate should be given immediately

DKA in pregnancy carries a significant mortality to the to the fetus This is the correct answer

Hyperphosphataemia occurs following treatment

Serum osmolality will not help in monitoring response to treatment Incorrect answer selected

The glucose level correlates well with the degree of acidosis

Diabetic ketoacidosis in pregnancy carries an increased mortality to both mother and fetus.

Serum osmolality is more reliable than glucose.

Phosphate and potassium fall with treatment.

The use of bicarbonate is not evidence based in any robust way, there is no threshold at
which there is evidence for its use, and if given at all, it should be given with care, with close
monitoring of the patient.

182. A 75-year-old retired journalist is referred to the outpatient clinic with a three month
history of weakness, malaise, unsteadiness, dizziness, headaches and several episodes
of epistaxis.

On examination she was pale but otherwise no obvious abnormalities were found.

Investigations reveal:

Haemoglobin 103 g/L (115-165)

White cell count 11.2 ×109/L (4-11)

Platelets 219 ×109/L (150-400)

Na 134 mmol/L (137-144)

K 4.3 mmol/L (3.5-4.9)

Urea 6.8 mmol/L (2.5-7.5)


Creatinine 112 µmol/L (60-110)

AST 35 U/L (1-31)

Bilirubin 15 µmol/L (1-22)

Alkaline phosphatase 98 U/L (60-110)

Albumin 31 g/L (37-49)

Total protein 110 g/L (61-76)

CRP 11 g/L (<10)

ESR 112 mm/1st hour (0-30mm/1st hour)

Further results:

IgG 9.9 g/L (7.2-19)

IgA 1.3 g/L (0.8-5.0)

43 g/L (0.5-2.0)
IgM
IgM paraprotein level of 41 g/L

Plasma viscosity 5.9 (1.4-1.8)

Whilst sitting in the department, she loses consciousness with no warning and has a grand
mal seizure.

What is the optimal treatment?

(Please select 1 option)

60 mg prednisolone

Cyclophosphamide

IV phenytoin

Plasmaphoresis Correct
Venesection

Waldenstrom's macroglobulinaemia can present in this manner with hyperviscosity


symptoms. The correct treatment is urgent plasmaphoresis.

Waldenstrom's macroglobulinaemia is a lymphoproliferative disorder - a plasma cell


dyscrasia which results in a monocolonal IgM gammopathy. The bone marrow, lymph nodes,
and spleen are the organs most often involved. Pulmonary involvement can also occur.

Waldenstrom's macroglobulinaemia has features in common with myeloma, lymphoma and


chronic lymphocytic leukaemia, from which it can be differentiated by serum protein
electrophoresis, which reveals very high levels of monoclonal IgM.

The increased plasma viscosity (due to aggregation of red blood cells by IgM) presents as
mucosal haemorrhage, for example, epistaxis, visual abnormalities due to retinal bleeding,
as well as peripheral and central neurological problems including cerebral haemorrhage,
seizures, chorea and coma.

Peripheral neuropathy can be caused by anti-myelin activity of the monoclonal IgM protein.

The excess monoclonal protein in the serum interferes with normal immunity.

183. Which of the following features favours a diagnosis of absence seizures over partial
complex seizures?

(Please select 1 option)

Dysphasic speech

Incontinence

Induction by hyperventilation Correct

Post-ictal confusion

The presence of aura

Absence seizures consist of suspension of awareness lasting a few seconds. They occur
without warning, are provoked by overbreathing, and are not usually associated with
complex motor movements or post-ictal confusion.

The characteristic ictal EEG is 3 Hz spike and wave.

Conversely, partial seizures may be preceded by an aura, can be associated with complex
behavioural and cognitive phenomena, and can be followed by post-ictal confusion.

184. 50-year-old man attends casualty complaining of deteriorating vision.

Exposure to which of the following drugs is unlikely to be relevant?

(Please select 1 option)


Chloroquine

Ethambutol

Irinotecan Correct

Quinine

Vincristine

Chloroquine (classically the 'Bull's eye retinopathy') and quinine cause visual disturbance.

Ethambutol may lead to loss of visual acuity, colour blindness, and restriction of visual fields.

Irinotecan is a topoisomerase I inhibitor used in colorectal cancer.

Vincristine may cause a toxic optic neuropathy.

185. A 50-year-old woman with a long history of alcohol abuse is taking phenytoin for
epilepsy.

Examination was normal except for a liver edge.

Her full blood count reveals:

Haemoglobin 100 g/L (115-165)

MCV 122 fL (80-96)

White cell count 2.2 ×109/L (4-11)

Platelet count 85 ×109/L (150-400)

What is the most likely explanation for these results?

(Please select 1 option)

Alcoholic liver disease

Aplastic anaemia

Folic acid deficiency Correct

Hypothyroidism

Vitamin C deficiency
Folic acid deficiency would give all these results. In addition she has good reason to be
folate deficient - she drinks a considerable amount and is on anticonvulsants.

Alcoholic liver disease on its own would not make you leucopenic.

Hypothyroidism, would cause a raised mean corpuscular volume (MCV), but not the other
parameters.

Scurvy does not cause this picture.

Aplastic anaemia could cause this haematological picture, but the clinical scenario leads you
towards folic acid deficiency.

186. 10-year-old boy presents with difficulty kicking a ball and weakness over the lower
legs which he has noted deteriorating over the last 12 months.

On examination he has pes cavus deformity of both feet, weakness of the lower leg with loss
of muscle bulk in the calves bilaterally but normal strength of the thigh muscles. Knee and
ankle reflexes are lost but there is no sensory loss.

What is the likely diagnosis?

(Please select 1 option)

Becker's muscular dystrophy

Chronic Guillain-Barré syndrome

Charcot-Marie-Tooth disease Correct

Hereditary spastic diplegia

Werdnig-Hoffman disease

This young child has typical features of Charcot-Marie-Tooth disease (hereditary


sensorimotor neuropathy type II).

This is an autosomal dominant condition associated with de-innervation and results in


weakness of feet and ankle muscles with gradual ascension to involve mid-thigh.

A sensory neuropathy may not be evident at first presentation but may develop.

9519

187. A 3-month-old girl presents with apnoea. She had been well that morning, but had
become unsettled, crying inconsolably and gradually more mottled. Mother was bringing
her to Accident and Emergency when she stopped breathing. She responded to physical
stimulation.
She was born at 40+3/40 weighing 3.6 kg and there were no neonatal problems.

On examination she has a temperature of 36.3°C (tympanic), RR 30/min and HR of 240/min.


Her pulse in thready. She has a 3 cm liver and gallop rhythm.

What is the most likely diagnosis?

(Please select 1 option)

Acute life-threatening event

Bronchiolitis

Gastro-oesophageal reflux

Seizure

Supraventricular tachycardia Correct

The history suggests apnoea precipitated by tachyarrhythmia. This is most likely to be a


supraventricular tachycardia.

This can be confirmed by ECG monitoring, and is usually successfully reverted by adenosine
with digoxin maintenance therapy. An echocardiogram will exclude the rare possibility of an
underlying structural defect

188. You are a senior house officer on call when a 40-year-old patient who is suicidal and
known to be suffering from paranoid schizophrenia threatens to leave the hospital.

Under which section of the Mental Health Act can you detain this patient?

(Please select 1 option)

5 Correct

Section 5 of the Mental Health Act is an emergency section that can be invoked by all
physicians, and allows patients suspected of having a psychiatric cause for their illness to be
detained in a place of safety (that is, hospital) for up to 72 hours whilst awaiting further
psychiatric evaluation.

189. An 8-year-old boy develops generalized edema with proteinuria of 4.8 gm/24 hours,
hypoproteinaemia and hypercholesterolaemia. He is started on steroids and improves.
Which of the following statements are true, regarding this condition?

(Please select 1 option)

Complete resolution is seen in all cases following a course of steroids

Ninety percent of steroid responders have no relapses.

Ninety percent of patients respond to first course of steroids Correct

Recurrence in 20% of cases with gradual resistance to steroids

Steroids have no effect on mortality

This patient has nephrotic syndrome. Suggestive features are usually:

oedema
proteinuria (of more than 4 g/24h), and
hypoproteinaemia with or without hypercholesterolaemia.

In 90% the cause is unknown, histology usually points minimal change glomerulonephritis as
the cause.

Most children with nephrotic syndrome respond to corticosteroids, but many experience a
relapsing course with recurrent oedema and proteinuria.

Corticosteroids reduce mortality to approximately 3%. Ninety percent respond in eight


weeks.

Approximately 30% of responders have no relapse. Ten to twenty percent of these are cured
after four or less courses of steroids. The rest may become dependent on steroids.

190. Active immunisation with tetanus toxoid is given as part of routine vaccination at 2, 3,
and 4 months of age (as DPT vaccine). A fourth dose is given >3 years later, and a fifth
dose before leaving school.

Once a patient has been given five injections at the allotted intervals, no further toxoid is
generally needed because of risks of side effects, and decreased immunity by over
stimulation.

This man is fully immunised.

191. An 18-year-old woman attends her general practitioner's surgery because of an


insect which had lodged in her right ear.

What is the most appropriate approach to remove this?

(Please select 1 option)


Instill 70% alcohol ear drops

Remove the insect with a hook

Syringe the ear with warm water Correct

Under general anaesthesia

Use a magnet

Syringing the ear is an option for many foreign bodies providing the tympanic membrane is
not perforated.

Suction under microscopy, which is not listed, would be useful if syringing fails.

Use of a general anaesthetic for this sort of incident is too risky.

Use of a hook may push the insect further inwards and magnets are used to aid removal of
foreign bodies made of iron.

192. 24-year-old drug addict attends casualty with a cold and painful right forearm.

The pain had been getting progressively worse for 24 hours prior to his attending. He has
injection site marks around his brachial artery.

On examination the hand is cold and paralysed with ischaemic areas at the tips of the digits.
The radial and ulnar pulses are palpable.

What is the first line in management?

(Please select 1 option)

Amputation

Embolectomy

Heparinisation Correct

Vascular reconstruction

It is likely that the patient has embolised the digital arteries with bulking agent mixed with his
heroin.

This is usually too distal for embolectomy and so heparinisation whilst ensuring adequate
hydration (to prevent myoglobinuria) is the treatment of choice.

If the main arteries are occluded then thrombolysis or embolectomy may be beneficial
193. 35-year-old shop worker presents with pain in her calves which develops after 50
yards of walking. The pain settles with rest.

On examination she has yellow discolouration of her palmar creases.

Her fasting lipid profile reveals:

Cholesterol 9.6 mmol/L (<5)

Triglycerides 7.3 mmol/L (<2)

What is the likely diagnosis?

(Please select 1 option)

Chylomicronaemia

Familial hypercholesterolaemia

Hypoalphalipoproteinaemia

Type III hyperlipidaemia Correct

Type IV hyperlipidaemia

Remnant hyperlipidaemia (type III hyperlipidaemia) is a rare disorder associated with:

Hypercholesterolaemia, typically 8-12 mmol/L


Hypertriglyceridaemia, typically 5-20 mmol/L
Normal ApoB concentration
Palmar xanthomata - orange discoloration of skin creases
Tuberoeruptive xanthomata - elbows and knees
Early onset of cardiovascular disease
Early onset of peripheral vascular disease.

Remnant hyperlipidaemia is due to a combination of abnormal ApoE receptor function


(which is normally required for clearance of chylomicron remnants and IDL from the
circulation) and a metabolic disorder such as diabetes, obesity or hypothyroidism.

The receptor defect causes levels of chylomicron remnants and IDL to be higher than normal
in the blood stream. The receptor defect is an autosomal recessive mutation or
polymorphism.The genotype of the homozygous condition is apo E-2/E-2 and occurs with a
frequency of 1:100.

The disorder responds well to treating the metabolic condition and lipid lowering medication.

Hyperlipidaemias are classified according to the Fredrickson classification.

Chylomicronaemia (type I) is associated particularly with severe hypertriglyceridaemia and


not with large elevations in cholesterol. There is no increased risk of atherosclerotic disease.
Hypoalphalipoproteinaemia is a rare, familial condition and is associated with low HDL.

Familial hypercholesterolaemia (type IIa) is due to LDL-receptor deficiency, and is not


associated with elevated triglyceride levels.

In type IV hyperlipidaemia cholesterol levels are within the normal range and triglyceride
level are elevated. VLDL levels are also elevated.

194. 17-year-old boy whose brother had hypertrophic cardiomyopathy was referred for a
cardiological assessment. His echocardiogram confirmed the condition.

Which one of the following echocardiographic features is the most important risk factor for
sudden cardiac death?

(Please select 1 option)

A gradient of 10 mmHg across the left ventricular outflow tract

An enlarged left atrium

Significant thickening of the interventricular septum Correct

Systolic anterior motion of the mitral valve

The presence of mitral regurgitation

In hypertrophic obstructive cardiomyopathy the cause of death is usually ventricular


tachycardia or ventricular fibrillation. Therefore, the thicker the muscle the more abnormal
the cardiac architecture and the higher the risk of arrhythmia and sudden death
Next question Go to summary

True

195. Which of the following should receive treatment with varicella immunoglobulin?

(Please select 1 option)

A non-immune pregnant woman who is exposed to her mother who has shingles Correct
A pregnant woman non-immune to varicella zoster (VZV) exposed to a child with chicken pox 12 days
previously.

A pregnant woman previously treated with varicella zoster immunoglobulin 10 days ago who has been
re-exposed to a case of chicken pox.

A pregnant woman who has no history of chicken pox but develops shingles in pregnancy

A pregnant woman with asthma taking steroids, who has had chicken pox as a child but is now exposed
to her daughter who has chicken pox.

Varicella immunoglobulin is effective if used sufficiently early in patients proven to be non-


immune to VZV and in whom exposure to VZV is confirmed.

The beneficial effects may last up to three weeks following initial treatment and beyond this;
it can be used again should re-exposure occur.

However, it is still important to check VZV antibodies as subclinical disease may have
occurred due to its prior use.

VZV can be given up to ten days with efficacy following exposure.

196. GLUCOCORTICOID DOSE CALCULATOR

Approximate Half-life
Equivalent dose (mg) (Biologic) hours

Short-Acting
Cortisone 25 8-12
Hydrocortisone 20 8-12
Intermediate-Acting
Methylprednisolone 4 18-36
Prednisolone 5 18-36
Prednisone 5 18-36
Triamcinolone 4 18-36
Long-Acting
Betamethasone 0.6 - 0.75 36-54
Dexamethasone 0.75 36-54
197. You are called to see a 65-year-old lady who is 30 hours post-operation for a
laparoscopic cholecystectomy.

Her operation and immediate recovery were uneventful. However, she now complains of
sore throat and some right lower chest pain.

She has a temperature of 38°C, pulse 80 beats per minute, blood pressure 140/80 mmHg
and oxygen saturation of 95% on air. She smokes 20 cigarettes a day.

What is the most likely post-operative complication?

(Please select 1 option)

Atelectasis Correct

Chest infection

Laryngitis

Pulmonary embolism

Wound dehiscence

Although all options can cause pyrexia, the key point here is the timing; atelectasis is often
encountered in the 48 hours following the operation.

The other options listed tend to present later than this.

It is one of the most common post-operative complications and occurs more frequently in
those with existing lung disease and smokers.

It should be noted that a sore throat is extremely common following operations as a result of
the minor trauma caused by the intubation.

198. HYPERKALEMIA The ECG suggests cardiotoxicity related to hyperkalaemia and the
history of palpitations is suggestive of arrhythmias.

Therefore cardio protection with calcium chloride or gluconate should be first priority and
lowering potassium levels immediately thereafter.

Calcium antagonises the effects of hyperkalaemia on a cellular level by a number of


mechanisms. These all return myocyte excitability to normal thereby reducing the risk of
arrhythmias. Calcium gluconate is the preferred preparation, but calcium chloride can be
used. Caution is needed if the patient is taking digoxin, as hypercalcaemia can potentiate its
toxicity. The effects of intravenous calcium occur within one to three minutes but last for only
30-60 minutes, and therefore more definitive treatment is needed to lower potassium levels.

After calcium is given, treatment is required to shift potassium intracellularly. Insulin is most
commonly used, which stimulates the Na-K ATPase pump. The effect is seen within 10-20
minutes and usually decreases potassium levels by 0.6-1mEq/L. Salbutamol can also
increase the action of the Na-K ATPase pump.

Sodium bicarbonate infusion can shift potassium intracellularly by increasing blood pH, but
its use is controversial. It is therefore reserved for cases of severe acidosis, or where there is
another indication for its use (for example, TCA overdose).

If the above treatments fail, and the cause of hyperkalaemia cannot be treated, then
potassium may need to be removed from the body. The most efficient way to do this is with
haemodialysis. This is only done in resistant cases, or in patients who are already on
haemodialysis. For most patients, treatment with an exchange resin such as sodium
polystyrene sulphonate is more appropriate.

199. A 60-year-old man presented with a rash over his forearms, shins and face when he
visited the cardiology clinic in the summer.

Which of the following medications is the most likely to be associated with this
photosensitive rash?

(Please select 1 option)

Atenolol

Bendroflumethiazide This is the correct answer

Clopidogrel

Digoxin Incorrect answer selected

Ezetimibe

Photosensitivity is a common adverse effect and the cardiology drugs affected include
amiodarone and thiazide diuretics.

Angiotensin-converting enzyme (ACE) inhibitors and angiotensin 2 receptor blockers


(A2RBs) commonly cause rashes some of which appear to be photosensitive.
200. A 32-year-old lady on holiday presented with a swollen left calf.

She says that she has 'factor V Leiden mutation'.

Which one of the following is the most appropriate description of the pathophysiology of this
mutation?

(Please select 1 option)

Activated protein C resistance Correct

Antithrombin deficiency

Protein C deficiency

Protein S deficiency

Prothrombin mutation

In the normal person, factor V functions as a cofactor to allow factor X to generate the active
form of an enzyme called thrombin. Thrombin in turn cleaves fibrinogen to fibrin, which
polymerises to form the dense meshwork that makes up the majority of a clot. Activated
protein C (aPC) is a natural anticoagulant that acts to limit the extent of clotting by cleaving
and degrading factor V.

Factor V Leiden is an autosomal dominant condition in which the coagulation factor cannot
be destroyed by aPC. Mutation of the gene encoding factor Va single nucleotide substitution
of adenine for guanine changes the protein's 506th amino acid from arginine to glutamine.
Since this amino acid is normally the cleavage site for aPC, the mutation prevents efficient
inactivation of factor V. When factor V remains active, it facilitates overproduction of
thrombin leading to excess fibrin generation and excess clotting.

201. West Nile virus is a mosquito-borne zoonotic arbovirus belonging to the genus
Flavivirus. It is thought it is spread when a mosquito bites an infected bird and then bites
a human. Few of those bitten develop symptoms and even fewer progress to severe
disease.

West Nile virus can be spread via vertical transmission as well as blood transfusions and
organ transplant.

If infected with the virus there are generally three different outcomes:

1. Asymptomatic (estimated 90%)


2. A mild febrile syndrome known as West Nile fever or rarely, and
3. Neuro-invasive disease termed West Nile meningitis or encephalitis.

West Nile fever can present with several vague 'generally unwell' symptoms that tend to last
three to six days such as:

Abdominal pain
Diarrhoea
Fever
Headache
Arthralgia
Nausea and vomiting
Rash
Sore throat, and
Lymphadenopathy.

The following symptoms are suggestive of West Nile encephalitis/meningitis and prompt
medical attention is required:

Extrapyramidal signs include:

Confusion and seizures


Loss of consciousness or coma
Muscle weakness
Stiff neck, and
Weakness of one arm or leg (a poliomyelitis-like paralysis).

Diagnosis can be via blood or cerebral spinal fluid serology for West Nile antibodies. More
rapid techniques using polymerase chain reaction may be used.

Due to the viral nature of the infection the current best treatment is supportive. In general it
has an excellent prognosis. For those rare cases with severe infection it may lead to brain
damage and death. Approximately 10% of patients with brain inflammation do not survive.

In 2003 there were 276 deaths attributed to West Nile virus.

Interestingly, West Nile Virus is endemic in the avian population. The deaths of large
numbers of birds in an area may thus herald an imminent epidemic of West Nile virus.

202. 59-year-old woman is admitted with a two month history of neck pain, upper limb
weakness and gait disturbance. She describes occasional episodes of electrical
sensation shooting down her spine on flexing her neck.

She has a long history of lumbar pain, primary generalised osteoarthritis and vitiligo; she has
also had a left hip replacement. She takes NASIDs regularly. She neither smokes nor drinks.
There is no history of recent foreign travel.
On examination she is afebrile. General examination is unremarkable except for vitiligo.
Examination of the cranial nerves is normal. There is no wasting of the limbs but there are a
few fasciculations in brachioradialis and biceps on the right. Tone is mildly increased. There
is weakness of elbow, wrist and finger flexion and extension, more marked on the right,
power is normal. There is inversion of the right supinator reflex and absent biceps reflex on
the right, Triceps reflex is brisk bilaterallly. There is increased tone in lower limbs bilaterally,
and bilaterally brisk knee and ankle jerks, right plantar is extensor, the left flexor. Sensation
and co-ordination are not impaired. Gait is mildly spastic.

Investigations reveal:

Blood tests including FBC, U+E LFT Calcium ESR and CRP are all normal; normal CXR;
normal x ray cervical spine; extensive osteophytes; normal alignment.

At what level is the lesion?

(Please select 1 option)

Brainstem

C3/4

C4/5

C5/6 Correct

C6/7

Inversion of the supinator jerk refers to brisk finger responses and little else on attempting to
elicit the supinator jerk.

It effectively localises the lesion to C5/6.

The pyrimidal tract damage at this level produces lower motor neuron signs in that myotome
with upper motor neuron signs below - hence the brisk triceps jerk.

Sphinter disturbance is unusual, but pain in the neck is common

203. 54-year-old accountant presents with symptoms suggestive of cluster headache.

Which is most appropriate agent for prophylaxis?

(Please select 1 option)

Amitriptyline

Aspirin
Methysergide

Sumatriptan

Verapamil Correct

This is cluster headache.

The treatment strategies can be divided into the acute and the prophylactic.

Acute treatment:

High flow oxygen


Sumatriptan injections / nasal spray
Because of the safety and tolerability issues, verapamil would be the first line
prophylactic
Ergotamine (not if also taking sumatriptan).

Prophylactic:

Verapamil
Methysergide (not for prolonged use because of risk of retroperitoneal fibrosis - very
important that patients are warned)
Sodium valproate
Lithium (need to monitor levels).

204. The jugular venous waveform in tricuspid regurgitation characteristically has which of
the following?

(Please select 1 option)

Cannon waves

Giant a wave

Large v waves This is the correct answer

Prominent x descent Incorrect answer selected

Prominent y descent

a Wave is related to atrial systole (prominent when RVEDP high, lost in AF).

x Descent is related to atrial relaxation (pronounced in constriction, tamponade).

The c wave is related to tricuspid closure.

The v wave is related to to atrial filling (during ventricular systole) (in TR, reflects
ventricularisation of pressure).
Finally the y descent is related to ventricular relaxation (prominent in constriction).

205. Which one of the following statements is correct regarding bupropion (Zyban®)?

(Please select 1 option)

It acts by inhibiting central noradrenergic activity

It has an anti-depressant effect Correct

It is less effective than nicotine replacement patches in smoking cessation

It should be given to all smokers with the incentive to quit smoking

There is an increased incidence of Churg-Strauss syndrome with its use

Bupropion has an antidepressant effect and was originally used extensively for treating war
veterans with PTSD (post traumatic stress disorder). It was then noticed that many of them
stopped smoking.

It is twice as effective as nicotine replacement therapy.

There are a number of contraindications due to the medication reducing the seizure
threshold. Risk of seizure is 1:1000 (similar to other antidepressants such as selective
serotonin reuptake Inhibitors). The data sheet should be referred to before initiating therapy.

FACTOR VIII CONCENTRATE REGIMEN

Desired

Dose and Duration of


Type of Hemorrhage
FVIII-C Activity Therapy

Minor 20-30% 10-15 U/kg IV q12-24h


for 1-2 d

Uncomplicated

hemarthroses
Superficial large

hematomas

Moderate 25-50% 15-25 U/kg IV q12-24h


for 3-7 d

Hematoma with dissection


(shorter time for oral
hemorrhages; higher
dose for hematuria)
Oral/mucosal hemorrhages and
epistaxis*

Hematuria

Dental extraction(s)† 50-100% 25-50 U/kg IV q12h for


2-5 d
Major ~50-100% until bleeding is 25-50 U/kg IV q12h for
controlled; then, gradually decrease 5-10 d
the dosage to the minimum that is
required to prevent rebleeding
Pharyngeal/retropharyngeal

Retroperitoneal

GI bleeding

CNS bleeding surgery

*Concomitant administration of EACA or AMCA (both inhibitors of fibrinolysis) can help


reduce the dose of concentrate that is required to treat such bleeding. Approximately 50% of
the initial dose is given as the second dose approximately 8 hours after the first; all
subsequent doses are given every 12 hours.

†For dental extractions, a single preoperative dose of factor VIII of 15 U/kg and oral or
intravenous Amicar at 5 g is given, followed by an Amicar maintenance dose of 1 g/h, as
discussed below, for 5-7 days, with a gradual taper.

206. Which of the following statements is of aetiological significance in a patient with


cardiomyopathy?

(Please select 1 option)

AST of 50 IU/l with a bilirubin of 12 mmol/l in a 50-year-old lady

The presence of a soft pan-systolic murmur in the mitral area

The presence of diabetes mellitus in a tanned patient Correct

The presence of sinus tachycardia with BP 140/80 mmHg

T-wave flattening in the inferior ECG leads in a 60-year-old man

The most common form of cardiomyopathy is dilated cardiomyopathy, with dilatation of the
left ventricle and reduction in the ejection fraction.

Often no cause is found, although rarely, it can be familial. A cause should be looked for in
most cases.

The commonest causes are

Ischaemic heart disease (Q-waves on the ECG, history of myocardial infarction [MI]) and
Hypertension.

Non-specific ECG changes and atrial fibrillation are common and do not point to a specific
cause.

Minor abnormalities of liver function result from liver congestion.

More marked abnormalities and diabetes in a pigmented patient would suggest


haemochromatosis.

Mitral regurgitation and arrhythmias are common in dilated cardiomyopathy of any cause.
207. Which of the following features is most inconsistent with a diagnosis of cervical
myelopathy ?

(Please select 1 option)

Bladder disturbance Correct

Extensor plantar responses

Finger pseudoathetosis

Inverted biceps jerks

Loss of vibration sense in the hands

Cervical myelopathy usually due to spondylosis occurs at C5-C7.

There is spatic weakness of the legs with hypertonia, clonus and upgoing plantars. Sphincter
involvement is unusual.

Joint position sense and vibration tend to be lost rather than the spinothalamic tract sensory
modalities.

An inverted biceps jerk refers to the brisk finger flexion that attempts to elicit it - the biceps
jerk, itself, does not occur.

Although spondylosis is the most common cause, an intrinsic lesion may cause a similar
clinical picture.

-year-old man reports diplopia and difficulty swallowing.

On examination he has an expressionless face with bilateral ptosis, there is diplopia worse
on lateral gaze. Pupils are slowly reactive to light and there is decreased palatal movement.
Limbs reveal proximal weakness of the upper and lower limbs, reflexes are preserved.

208. Which of these investigations is most likely to provide the diagnosis?

(Please select 1 option)

Acetyl choline receptor antibodies

Anti-ganglioside G1Q antibodies

CSF microscopy and culture

Nerve conduction studies

Tensilon test Correct

This patient has the features of myasthenia gravis.

Facial, palatal, oculomotor and neck muscles are most likely to be affected.
The test which confirms the diagnosis is a tensilon test.

Acetyl choline receptor antibodies are positive in up to 90% of cases and may take some
time to come back from the laboratory.

Nerve conduction studies per se are unlikely to be helpful, although single fibre EMG may
reveal characteristic jitter pattern.

Anti G1Q antibodies are found in Guillan-Barre syndrome.

Cerebrospinal fluid (CSF) may be entirely normal in myasthenia.

209. In Gullain-Barre syndrome which of the following statements does not suggest a
worse prognosis?

(Please select 1 option)

Evidence demyelination on nerve conduction studies Correct

History of diarrhoea in the preceding few weeks

Markedly reduced FVC

Presence of postural hypotension

Severe weakness

The diagnosis is Gullain-Barre syndrome.

Denervation (rather than demyelination) shown on nerve conduction studies implies a poor
prognosis as do the remaining stems, in addition to old age.

In around 3 % of cases, death occurs. Up to 10 % have severe residual disability.

210. Which of the following is least true regarding etanercept?

(Please select 1 option)

Has been associated with demyelination

Has been associated with serious blood disorders

Is an inhibitor of TNF-alpha

Is licensed for polyarticular-course juvenile idiopathic arthritis in children over 4 years age

Must be given intravenously Correct

Etanercept is an inhibitor of TNF-alpha.


It is a novel (recombinant) fusion protein of a soluble TNF receptor bound to the Fc of human
immunoglobulin. This competes with the endogenous TNF receptors for TNF-alpha binding.

Etanercept may be given subcutaneously and is licensed for the treatment of rheumatoid
arthritis in adults whose response to disease-modifying antirheumatic drugs has been
inadequate.

Five percent of patients have suffered skin reactions at the injection site, or urticaria. Serious
blood disorders and demyelination have also been associated.

211. 57-year-old female, with a history of hypertension and diabetes, presents with acute,
painful pupil-involving third nerve palsy.

CT head and LP are normal.

Which of the following investigations should be performed next?

(Please select 1 option)

Cerebral angiography Correct

CSF protein electrophoresis

HbA1c

Serum protein electrophoresis

T2-weighted MRI brain

The acute painful pupil-involving third nerve palsy is caused by an enlarging posterior
communicating artery aneurysm until proven otherwise.

This patient should therefore have urgent cerebral angiography or a magnetic resonance
angiogram.

212. 41-year-old plasterer suffers acute onset of right orbital pain at work. The next day he
notices that his right eye does not appear normal.

On examination, he has a right ptosis, and the right pupil is smaller than the left, but both
react normally to light. Visual acuity, fields and eye movements are normal. He does not
have anhydrosis.

The site of injury may be to fibres from which of the following structures?

(Please select 1 option)

Left occipital cortex


Medial longitudinal fasciculus

Right oculomotor nerve

Superior cervical ganglion Correct

T1 nerve root

The combination of ptosis and a unilaterally small pupil implies a Horner's syndrome.

Sympathetic fibres from the ipsilateral hypothalamus pass through the brainstem and
cervical cord to T1/T2. These synapse on preganglionic sympathetic fibres, travel up the
sympathetic chain to the superior cervical ganglion, and then synapse onto postganglionic
fibres which travel with the common and internal carotid arteries.

Pancoasts's tumour (apical lung)


Carotid artery tumour or
Dissection and
Syringomyelia

can all cause a Horner's.

The presence, absence, and/or location of anhydrosis is an important localising sign that
may be elicited from the history.

The presence of a normal sweating pattern localises the lesion to the ganglion or more
peripherally.

213. 44-year-old call centre worker with asthma is prescribed a leukotriene inhibitor.

He presents with severe abdominal pain and a pleural effusion.

Which of the following is the most likely cause of the effusion?

(Please select 1 option)

Carcinoma

Churg-Strauss syndrome Correct

Heart failure

Pancreatitis

Pneumonia

Churg-Strauss syndrome has a predilection for serosal surfaces and therefore can cause
both pleural and pericardial effusions.

Cytological analysis of this fluid would confirm an eosinophilia.


Leukotriene inhibitor use is associated with an increased incidence of the disease

214. Which of the following organs is in direct contact with the anterior surface of the left
kidney, without being separated from it by visceral peritoneum?

(Please select 1 option)

Duodenum

Jejunum

Pancreas Correct

Spleen

Stomach

The spleen and stomach, though in contact, are covered in omentum.

The pancreas is retroperitoneal.

215. Which of the following statements regarding bariatric surgery is correct?

(Please select 1 option)

Associated with a significant post-operative mortality

Contraindicated in adolescents

Indicated in patients with a BMI of greater than 30

Indicated in patients with a BMI less than 35 kg/m2

Reduces cardiovascular mortality Correct

Bariatric surgery is the only intervention for obesity that has significant and enduring weight
loss and comorbidity resolution. The Swedish Obesity Study demonstrated significantly
reduced cardiovascular event and death rates up to 15 years following bariatric surgery
compared with usual care.1

Bariatric surgery in adolescents raises social, psychological and developmental issues, but
adolescents are not excluded from surgery, and some hospitals have specialised
programmes for younger patients.
Potential candidates for surgery are those with a body mass index (BMI) exceeding 40, or
BMI greater than 35 with serious co-morbidities (for example, sleep apnoea, type 2
diabetes).2

Post-operative mortality ranges from 0.1-2%. The rate of post operative complications
following bariatric surgery is no greater than other elective major abdominal operations but
there is a higher risk of intervention if a complication occurs.

Specific post bariatric complications deoend on the procedure used:

Laparoscopic adjustable gastric band - band slippage, erosion, infection, pouch


dilatation, band/tubing leak, megaoesophagus.
Laparoscopic roux en y gastric bypass - stomal stenosis, internal hernia, malnutrition.
Laparoscopic sleeve gastrectomy - reflux, staple line leak, sleeve dilatation and weight
gain.

216. 5-year-old man with a history of ischaemic heart disease presents at the outpatients
department being unable to climb two flights of stairs before developing chest pain.

Investigations revealed:

Total cholesterol 5 mmol/l (<5.2)

Echocardiogram Good left ventricular function

Exercise test 3 mm ST segment depression in stage 3

Which one of the following treatments is most likely to improve his prognosis?
(Please select 1 option)

Aspirin Correct

Atenolol

Diltiazem

Isosorbide mononitrate

Nicorandil

Aspirin has been shown to reduce the incidence of non-fatal myocardial infarction (MI)
and cardiac events in angina pectoris.

Evidence for benefits from beta-blockers comes from post-MI subjects.

The other treatments give mainly symptomatic benefit from angina.


217. A 7-month-old girl presents with fever and a rash.

She was completely well until five days ago, when she developed a slight cold. The next day
she developed fever to 39.7°C, which has persisted despite antipyretics. Despite this she
has remained relatively well and continues to drink, though her appetite is poor.

Today she has developed a rash over the face and trunk. She was born at term weighing 3.8
kg and there were no neonatal problems. She is fully immunised to date and there is no
family history or social history of note.

On examination she has a temperature of 36.8°C, respiratory rate 25/min and heart rate
100/min. The rash is macular, profuse, pink and blanching. It is most prominent over the face
and trunk. She has shotty cervical lymphadenopathy.

What is the most likely diagnosis?

(Please select 1 option)

CMV infection

Infectious mononucleosis

Measles

Parvovirus infection

Roseola infantum Correct

The history of a well child with high fever for a few days followed by resolution of fever at
around the time of appearance of a rose-coloured rash is characteristic of roseola infantum.

Since the introduction of the measles, mumps, rubella (MMR) vaccine, this is by far the
commonest cause of a measles-like rash.

The peak incidence is six to eighteen months. 5% develop febrile seizures.

It is caused by human herpes virus 6 and 7.

218. 13-year-old girl presents to the clinic as her parents are concerned regarding a two
month history of recurrent episodes of abdominal pain associated with loss of appetite
and nausea.

On further questioning the pain is quite severe, tends to occur in the mornings, is intermittent
and can occur during holidays. Her weight is on the 50th centile and no abnormalities are
found on examination.
What is the most likely diagnosis?

(Please select 1 option)

Abdominal migraine Correct

Appendicitis

Crohn's disease

Mesenteric adenitis

Psychosomatic

Of the options given, abdominal migraine is most likely.

Abdominal migraine is an idiopathic disorder seen mainly in children. The symptoms are of
recurrent episodes of midline abdominal pain with attacks lasting 1-72 hours and complete
normality between episodes.

The pain is of moderate to severe intensity and is felt in the midline of the abdomen, usually
around the umbilicus, or poorly localised. The attacks of pain are usually accompanied by
anorexia and nausea and about half of the patients will vomit with at least some attacks.

Marked pallor is commonly noted during the attacks although some patients may appear
flushed. The pain is severe enough to interfere with normal daily activities and many children
describe their mood during the attack as one of intense misery. The attacks are self limiting
and resolve spontaneously and patients are completely well and symptom free between
attacks.

This is unlikely to be psychosomatic as the description is so episodic and it can occur


outside school times.

219. Which of the following infusion times would be appropriate during the transfusion of a
blood product in a stable patient?

(Please select 1 option)

A packed cell transfusion should be given over 20 minutes

A packed cell transfusion should be given over 90 minutes Correct

A platelet transfusion should be given over 60 minutes

A platelet transfusion should be given over 90 minutes

A platelet transfusion over 120 minutes


In a stable patient red cell packs may be transfused over 90-120 minutes while a platelet
transfusion should not take more than 20-30 minutes.

Rapid infusion of red cells or fresh frozen plasma may be required in acutely bleeding patient
but not in this patient who is stable.

220. Approximately 1% of pregnant women develop clinically important red cell antibodies,
the most common being rhesus antibodies. The women negative for D antigen develop
antibodies on exposure to D positive blood (such as fetomaternal haemaorrhage,
abortions and transfusions). This increases the risk of Hemolytic Disease of the Newborn
(HDN) in subsequent pregnancies.

Which of the following statements about rhesus antibodies in pregnancy is correct?

(Please select 1 option)


D positive women are less likely than D negative women to form antibodies to other red cell antigens
(such as Kelland Duffy)

Following delivery, the degree of FMH should be calculated on a blood sample from a D negative
mother Correct

Maternal antibody titres do not predict haemolytic disease of new born

Pregnant women should be checked for antibodies at 28 weeks as faetomaternal haemorrhage (FMH)
occurs only after the second trimester

The fetal Rh type is not dependent on the paternal Rh grouping

Following delivery, the degree of FMH should be calculated on a blood sample from a D
negative mother to adjust the dose of anti D in the D negative mother delivering a D positive
child.

D positive women and D negative women have the same chances of developing antibodies
to other red cell antigens. All pregnant women should have a blood group and antibody
screen in their first trimester or at presentation, whichever is earlier. The fetal Rh type
depends on the paternal and maternal Rh typing. Maternal antibody titres correlate with the
degree of HDN.

221. Which of the following potential blood donors would be excluded from donating whole
blood or plasma?

(Please select 1 option)

A 27-year-old man who has had a tattoo on his arm for the last ten years

A 32-year-old lady whose sister was diagnosed with Creutzfeld-Jakob disease three years ago
Correct

A 38-year-old lady who underwent acupuncture for leg pain five months ago with a registered health
professional
A 42-year-old man with a history of hepatitis when he was aged 18 years

A 56-year-old man who had a course of antibiotics for an upper respiratory tract infection which was
completed four weeks ago

Any potential donor whose immediate family member (parent or sibling) has had Creutzfeld-
Jakob disease needs to be excluded from blood donation.

All other options cannot be excluded on the basis of the current blood donation guidelines
(National Blood Service UK).

Hepatitis or jaundice would be an exclusion only if it occurred in the last 12 months.

Acupuncture in the last four months (unless performed by a registered professional) or body
piercing /tattoos in the last six months would also be an exclusion.

Any infection in the preceding two weeks or a course of antibiotics within the last seven days
would be considered as exclusion

222. Which of the following medications are available OTC (over the counter) for
treatment of heavy menstrual blood loss?

(Please select 1 option)

Combined oral contraceptives

Non steroidal anti-inflammatory drugs (NSAIDS)

Norethisterone acetate Incorrect answer selected

MIRENA

Tranexamic acid This is the correct answer

Since March 2011 tranexamic acid has been available for purchase from pharmacies for the
management of heavy menstrual bleeding

NSAIDS such as ibuprofen and naproxen are available OTC but help with dysmenorrhea
and not menorrhagia. MIRENA, Norethisterone and COCP are not available OTC for the
treatment of heavy blood loss but COCP is available for contraceptive purposes.

Next question

223. A 44-year-old woman presents to the clinic with increasing lethargy and fatigue. She
has been treated previously with interferon alpha and has a diagnosis of chronic myeloid
leukaemia.
Her white blood cell count has risen to 22 ×103 cells per microlitre, and she is anaemic with a
recent haemoglobin of 89 g/L.

You decide to start her on imatinib.

Which of the following correctly describes the mode of action of imatinib?

(Please select 1 option)

Bcr-abl tyrosine kinase inhibitor Correct

Epidermal growth factor receptor (EGFR) kinase inhibitor

Fibroblast growth factor receptor inhibitor

HER2 receptor inhibitor

Vascular endothelial growth factor (VEGF) inhibitor

The answer is Bcr-abl tyrosine kinase inhibitor.

In chronic myeloid leukaemia (CML) this particular kinase is stuck chronically in the "on"
position.

By targeting the kinase, imatinib inhibits the unregulated cell division which occurs in CML
and can maintain many patients in remission for a number of years.

There is also a role for imatinib in the treatment of gastrointestinal stromal tumours, where
targeting of Bcr-abl tyrosine kinase has been shown to impact on progression of tumour size.

Other small molecules used in the treatment of cancer include trastuzumab which targets the
human epidermal growth factor 2, used in the treatment of breast cancer, and sunitinib,
which inhibits multiple kinases which are stimulated by agonism of a number of receptors
including VEGF receptors, RET and platelet-derived growth factor (PDGF) receptors.

This man has a stable pelvic fracture and target blood pressure is around 100 mmHg.
Undershooting this target increases the risk of renal hypoperfusion and prerenal failure, and
overshooting the target runs the risk of precipitating further bleeding.

As such blood pressure should be regularly measured and fluid titrated accordingly.

Whilst the haemoglobin is only slightly below the normal range currently, resuscitation may
reveal the true extent of blood loss.

224. 30-year-old woman develops a right sided facial weakness very quickly over 24
hours. You think that the most likely diagnosis is Bell's palsy and you carry out an
examination to confirm this.

Which one of the following findings is in keeping with a diagnosis of Bell's palsy?

(Please select 1 option)


Right sided facial paralysis with pain discharge and tinnitus in the right ear

Right sided facial paralysis with sparing of the forehead muscles

Right sided facial paralysis with swelling of the right parotid gland and asymmetry of the right tonsil

Right sided facial paralysis with weakness of the forehead muscles Correct

Right sided facial paralysis with weakness of the right side of the tongue

Bell's palsy is paralysis of the VII cranial nerve (facial nerve).

Typical features of Bell's palsy are usually of sudden onset and complete within 24 hours.
Less frequently, the condition is progressive developing over a course of less than four days.

It is almost always unilateral, onset may be preceded by post-auricular pain which develops
over a 48 hour period There is paralysis of the upper and lower facial muscles, the eyebrow
droops and the wrinkles of the brow are smoothed out; frowning and raising the eyebrows
are impossible.

If there is an upper motor neurone lesion affecting the facial nerve then the ability to wrinkle
the brow is preserved; in Bell's palsy the eye cannot be closed, when asked to close the
eyes and show the teeth, the eyeball rotates upwards and outwards - Bell's phenomenon.

Bell's palsy affects taste to the anterior 2/3 of the affected side of the tongue. The muscles of
the tongue are supplied by the hypoglossal nerve so weakness of the tongue does not occur
in Bell's palsy.

225. 53-year-old woman is started on a capecitabine based regime for the treatment of
metastatic carcinoma.

Which of the following is true of capecitabine?

(Please select 1 option)

Capecitabine is more effective than IV agents

Diarrhoea is rarely seen with therapy

It is a way to deliver 5 fluorouracil orally This is the correct answer

It is a way to deliver cisplatin orally Incorrect answer selected

It is not effective in the treatment of coloncarcinoma

The answer isthat it is a way to deliver 5 fluorouracil orally.

Capecitabine is a prodrug, which is metabolised to produce 5-fluorouracil, a


chemotherapeutic agent used intravenously in the treatment of cancer. Damage to rapidly
dividing cells in the GI tract leads to symptoms of diarrhoea and vomiting as a result of
capecitabine treatment. Studies of efficacy suggest no major differences in clinical remission
when capecitabine is compared to 5-fluorouracil given IV

226. 58-year-old man who has a history of hypertension and type 2 diabetes presents to
the Emergency department complaining of central chest pain which is going down his left
arm.

His medication includes ramipril, metformin, atorvastatin and gliclazide.

On examination his BP is 129/72 mmHg, and his pulse is 81. He has bibasal crackles on
auscultation of his chest.

Investigations reveal:

Haemoglobin 138 g/L (130-180)

White cell count 8.9 ×109/L (4-11)

Platelet 197 ×109/L (150-400)

Sodium 141 mmol/L (135-146)

Potassium 4.1 mmol/L (3.5-5)

Creatinine 123 µmol/L (79-118)

Glucose 12.3 mmol/L (<7.0)

ECG Anterolateral ST depression

He is given sublingual GTN.

Which of the following is the next most appropriate therapy?

(Please select 1 option)

Aspirin 300 mg and low molecular weight heparin

Aspirin 300 mg, clopidogrel 300 mg and low molecular weight heparin Correct

Beta blockade

Clopidogrel 300 mg
IV GTN infusion

The answer is aspirin, clopidogrel and low molecular weight heparin.

This patient is high risk given his history of type 2 diabetes mellitus, and as such should be
loaded with both aspirin and clopidogrel.

Further chest pain, or failure of his ECG signs to resolve may drive further intervention
including progression to angiography.

If this patient does not progress to angiogram then screening for ischaemia should be
considered prior to discharge.

56-year-old man complains of low mood and sleep disturbance for the last two months. In
the last two weeks he reports difficulty remembering the names of familiar objects and
difficulty writing.

Myoclonus in the upper limbs is noted on examination.

What is the single most relevant diagnosis?

(Please select 1 option)

Creutzfeldt-Jakob disease Correct

Huntington's disease

Hypothyroidism

Lewy body dementia

Wernicke's encephalopathy

Creutzfeldt-Jakob disease is characterised by a rapidly progressive dementia, myoclonus


and distinctive electroencephalographic and neuropathologic findings. The infectious agent
causing CJD is unique in being a conformationally abnormal prion protein, that is, it contains
no genetic material.

The dementia can be accompanied by signs of involvement of any part of the central
nervous system, but myoclonus is particularly common. Although typically occurring
sporadically in middle-aged adults, a family history may be present in 8-10%.

More recently, variant CJD in young adults has been linked with exposure to beef infected
with the bovine spongiform encephalopathy agent. This 'new variant' form often presents
with an extended neuropsychiatric prodrome with mood disturbance or other psychiatric
symptomatology.

227. 24-year-old man on treatment for epilepsy is admitted to hospital with severe, central
abdominal pain. Investigations reveal that serum amylase is markedly elevated.

Which drug is most likely responsible for this presentation?

(Please select 1 option)

Carbamazepine

Ethosuximide

Lamotrigine

Phenytoin

Sodium valproate Correct

This patient is suffering from acute drug-induced pancreatitis.

Side effects of valproate include

weight gain
nausea
vomiting
hair loss
easy bruising
tremor
hepatic failure
pancreatitis.

228. 20-year-old male with chronic epilepsy develops profound hyponatraemia.

Which drug is most likely responsible for this presentation?

(Please select 1 option)

Carbamazepine Correct

Ethosuximide

Sodium valproate

Tiagabine

Vigabatrin
The major systemic side effects of carbamazepine are

nausea
vomiting
diarrhoea
hyponatraemia
rash
pruritus
fluid retention.

229. Treatment of mild to moderate acne vulgaris is best started with topical therapy
such as benzoyl peroxide. Benzoyl peroxide works as a peeling agent. It increases skin
turnover, clearing pores and reducing the bacterial count.

This therapy is usually effective and would be the most appropriate choice for the 15-
year-old.

230. 55-year-old man presents with mild headache. He has changed his spectacles three
times over the past year.

There is mild cupping present in the disc and sickle-shaped scotoma present in both eyes.

What is the most appropriate treatment?

(Please select 1 option)

B complex vitamins

Corticosteroids

Pan-retinal photo coagulation

Peripheral iridectomy Incorrect answer selected

Pilocarpine eyedrops This is the correct answer

In chronic simple glaucoma a sickle-shaped scotoma called Seidel's sign may be seen as a
significant field defect. Pilocarpine, a drug that promotes pupillary constriction, miotic, is a
treatment for glaucoma.

Pilocarpine 1-4% is a direct cholinergic agonist which will cause miosis and accommodative
spasm. It is used to reverse the action of mydriatics but its greatest therapeutic use is in
lowering ocular pressure in both acute and chronic glaucoma by acting through the ciliary
muscle to increase aqueous outflow.

Pilocarpine may have local side effects which may be more severe in the initial two to four
weeks of treatment. These side effects include:

conjuctival vascular congestion


ciliary spasm
supraorbital headache
lacrimation
induced myopia
retinal detachment, and
reduced visual acuity in low light.

Pilocarpine may sometimes cause systemic parasympathetic symptoms, for example:

salivation
sweating
hypertension
tachycardia
diarrhoea and vomiting
bronchiolar spasm, and
pulmonary oedema.

231. What is the single most likely anaesthetic or analgesic cause of reduced variability
on a cardiotocography?
(Please select 1 option)

Epidural anaesthetic

General anaesthetic Incorrect answer selected

Intramuscular pethidine This is the correct answer

Pudendal block

Spinal anaesthetic

Pethidine, other opiates and some antihypertensives (alpha methyldopa and


labetalol) reduce cardiotocography (CTG) variability.
232. 12-year-old boy hit his head on a pole on the school playground. His friends noted
that he lost consciousness for a few seconds.

The school nurse sent him to the Emergency department as he had a small nasal bleed
that stopped spontaneously. In the intervening two hours, he has noticed a watery
discharge from his nose.

On examination there is considerable bruising on his nose and forehead, he was


complaining of a headache.

What is the most appropriate diagnosis?

(Please select 1 option)

Allergic sinusitis
Basilar skull fracture Correct

Ethmoid bone fracture

Frontal bone fracture

Orbital bone fracture

The child has sustained an head injury and subsequently describes a CSF rhinorrhoea
(indicating a cerebro-spinal fluid leak).

CSF rhinorrheoa can occur in skull or nasal bone fractures. Given the symptoms of loss
of consciousness and headache, this child is more at risk of having suffered a skull
fracture requiring emergency CT head investigation and specialist neuro-surgical
management.

An ethmoid bone fracture may also present this way. A skull x ray would help to
determine an air-fluid level and indeed allow some visualisation of the nasal bones,
though in children the nasal bones do not visualise well due to lack of fusion.

Either way this child would need assessment in the nearest Emergency department
and the school would be expected to follow a 'head injury' protocol.

233. 42-year-old man presents with episodic epistaxis. He gives a history of excessive
crusting, occasional epistaxis and has also noticed a whistling sound on talking and deep
inspiration. He is employed in a chrome plating factory.

On examination, he has gross septal deviation to the left and crusting over his septum.

What is the most likely cause of epistaxis in this patient?

(Please select 1 option)

Coagulopathy possibly induced by exposure to chrome Incorrect answer selected

Nasopharyngeal angiofibroma

Sarcoidosis

Septal perforation This is the correct answer

Wegener's disease

Chromium is an irritant and may cause destructive nasal septum lesions with chronic
exposure. This would explain the crusting and deviation. Chrome is not recognised to cause
coagulopathy on exposure.

Epistaxis, or nasal bleed, is a very common condition. Causes for epistaxis can be classified
into:
Traumatic - includes fractures of the nasal bones, maxilla, and floor of the cranial cavity
or soft tissue injury around the nasal cavity
Neoplastic - includes both benign (angiofibroma of nasal cavity) and malignant (maxillary
or other paranasal sinus carcinoma, Kaposi sarcoma)
Infective - rarely causes epistaxis, however severely congested conchae could bleed with
very trivial trauma or conditions like orf
Systemic - a variety of systemic illnesses could give rise to nasal bleed; the common
ones for a spontaneous epistaxis are hypertension, bleeding diathesis including
haematological malignancies
Miscellaneous - includes medications such as anticoagulants.

Nose picking is the most common cause of epistaxis in children.

Nasal polyps are a common cause of epistaxis but these are generally produced by mucosal
hypertrophy rather than true hyperplasia and so are not strictly neoplastic.

234. 60-year-old with a fluent dysphasia is unable to comprehend instructions.

What is the most likely site of arterial occlusion?

(Please select 1 option)

Basilar artery

Inferior division of middle cerebral artery (dominant hemisphere) Correct

Posterior inferior cerebellar artery

Superior division of middle cerebral artery (dominant hemisphere)

Thalamogeniculate artery

Wernicke's aphasia results from occlusion of the inferior division of the middle cerebral
artery which supplies the temporal cortex (posterior superior temporal gyrus, also known as
'Wernicke's area').

It is a fluent aphasia, where the comprehension is markedly impaired.

Broca's aphasia is classed an a non-fluent expressive aphasia, secondary to lesions in


Brodmann's area in the frontal lobe.

235. An active 25-year-old female has previously been warfarinised for three consecutive
episodes of deep vein thrombosis (DVT). She is presently off warfarin and is now
presenting with right calf pain again.

She has had no recent long haul flights or journeys. Other than the history of DVT she has
no significant past medical history and is not taking any medication.

What is the most likely underlying cause for her presentation?

(Please select 1 option)


Acquired thrombophilia Correct

HIV

Oral contraceptive pill

Pregnancy

Sickle cell disease

Patients with recurrent DVT may have an acquired thrombophila, for example
antiphospholipid syndrome, which may also be the cause for recurrent miscarriages in
women.

In this patient she is not noted to be pregnant or taking any medications.

The oral contraceptive pill is also recognised as a risk factor for developing venous
thromboembolism, the progesterone only pill is slightly lower risk in this respect.

236. Which of the following options about nasal polyps is correct?

(Please select 1 option)

Can cause anosmia Correct

Have a pinkish red appearance

Rarely recur after polypectomy

Rarely respond to steroids topically

They usually originate in the maxillary sinuses

The vast majority of nasal polyps arise in the ethmoid sinuses. Much more rarely
antrochoanal polyps arise from the maxillary sinuses and are associated with chronic
infection.

The cause of polyps is mainly allergy but there may be an infective component.

They nearly always regress with topical steroids, but many will require surgical removal and
most recur.

They have a yellowish grey appearance. Pink or red polyps should be regarded as
suspicious.

Anosmia is frequent due to blockage, and smell is usually restored after treatment but not
always.
Key Learning Points
Pharmacology

Since March 2011 tranexamic acid has been available for purchase from
pharmacies for the management of heavy menstrual bleeding

Ophthalmology

Pilocarpine promotes pupillary constriction and is a treatment for


glaucoma.

Anaesthetics & ITU

Many drugs cannot be used in pregnancy and delivery; it is important to be


aware of the more common side effects.

ENT

Exposure to chrome inhalation may result in destructive nasal lesions


including septal perforation

237. Which of the following is least true about bupropion (Zyban)?

(Please select 1 option)

It increases dopamine levels in the nucleus accumbens

It is an amphetamine derivative

It is an inhibitor of the cytochrome P450 enzyme system Incorrect answer selected

It is associated with an idiosyncratic risk of seizure This is the correct answer

It is contraindicated in anorexia nervosa

Bupropion (amfebutamone) (Zyban) is an amphetamine derivative almost identical in


structure to the withdrawn slimming drug, diethylpropion.

It is thought to act via increasing dopamine levels in the nucleus accumbens in a similar way
to nicotine in causing addiction. There is evidence from a meta-analysis of the two published
trials that it improves 12 month sustained abstinence rates and reduces the severity of
withdrawl symptoms.

There are currently no published trials of bupropion with minimal behavioural support.
Bupropion is associated with a dose-related risk of seizure of about 1:1000 and is
contraindicated in patients with a previous or current seizure disorder.

Common side effects are CNS reactions (insomnia, dizziness, tremor, anxiety, depression)
and hypersensitivity (urticaria, rash).

Bupropion inhibits metabolism of cytochrome P450 2D6.

It follows that caution is required when co-administering with, for example, antidepressants
(for example, paroxetine), antipsychotics (for example, risperidone), beta-blockers and class
1C anti-arrhythmics (for example, propafenone, flecainide).

238. 60-year-old woman presents with acute attacks of painful swelling of the left side of
her face and throat. She also complains of episodic abdominal pain.

On examination she has splenomegaly.

What is the most likely diagnosis?

(Please select 1 option)

Acquired C1 esterase deficiency Correct

CLL

Factor H deficiency

Inherited angioedema

Sarcoid

This patient has acquired C1 esterase deficiency; she is too old to present with inherited
angioedema and it does not explain the splenomegaly, which is probably due to a B cell
lymphoma.

C1 esterase inhibitor is a member of the serpin family of protease. This protein is produced
by hepatocytes and inhibits the catalytic subunits of the first component of the classical
complement pathway (C1r and C1s).

C1 esterase inhibitor also inhibits the function of kallikrein, plasmin, and coagulation factors
XIa and XIIa.

In C1 esterase deficiency, activated C1 and plasmin generate activated C2 kinin. Activated


C2 kinin results in the typical angioedema observed in patients with C1 esterase inhibitor
deficiency.

The inherited form of C1 esterase deficiency usually presents in the first or second decade.
The acquired form affects adult or elderly patients.

Acquired C1 esterase deficiency presents with recurrent, localised, subcutaneous swelling


which appears spontaneously and resolves within hours to days. The angioedema is not
associated with itching and is often painful. Most patients have associated low-grade
lymphoproliferative disease.

C1q levels are normal in the inherited form of C1 esterase inhibitor deficiency, they are
decreased in the acquired form. This is because of the large numbers of immune complexes
associated with the lymphoproliferative neoplastic process. These immune complexes
consume both C1q molecules and C1 esterase inhibitor, resulting in quantitative and
functional deficiency of both proteins.

239. A 54-year-old female school teacher is referred to the clinic with tiredness, lethargy
and an abnormal thyroid function test. The repeat thyroid function test is similar to that
obtained by the GP.

There is a low thyroid-stimulating hormone and elevated serum thyroxine.

Which of the following conditions is consistent with these results?

(Please select 1 option)

Graves Correct

Iodine deficiency

Secondary (pituitary) hyperthyroidism

Secondary (pituitary) hypothyroidism

Tertiary (hypothalamic) hyperthyroidism

Secondary hyperthyroidism results in increased secretion of thyroid stimulating hormone


(TSH) by the pituitary, therefore there is an increase in both TSH and serum thyroxine.

Tertiary hyperthyroidism is due to increased levels of thyroid releasing hormone (TRH),


therefore there is an increase in both TSH and serum thyroxine.

Iodine deficiency results in decreased thyroid hormone, therefore decreased thyroxine and
due to the feedback there would be an increase in TSH.

Secondary hypothyroidism is due to decreased TSH secretion hence there is decreased


plasma T4.

240. Which of the following is not a recognised cause of a dominant R in lead V1 on the
ECG?

(Please select 1 option)

Dextrocardia
Duchenne muscular dystrophy Incorrect answer selected

Ebstein's anomaly

Myotonic dystrophy This is the correct answer

Primary pulmonary hypertension

A dominant R wave in V1 has few causes

Right ventricular hypertrophy (RVH) as in Eisenmenger's syndrome


Primary pulmonary hypertension (HT)
Right bundle branch block (RBBB) as in Ebstein's anomaly
Wolff-Parkinson-White syndrome type A
Dextrocardia
True posterior myocardial infarction (MI)
Duchenne muscular dystrophy.

241. A 16-year-old A-level student develops deterioration in vision in her left eye over
three days. She complains of discomfort in the eye and thinks that difficulty with
perception of colour was the first problem that she noticed, during a trip to an art gallery.

On examination, visual acuity on the left is down to light perception. The pupil appears
dilated and does not constrict to light, although does when a torch is shone in the right eye.

What is the most likely diagnosis?

(Please select 1 option)

Amaurosis fugax

Anterior ischaemic optic neuropathy

Central retinal artery occlusion

CMV retinitis

Optic neuritis Correct

The causes of acute unilateral loss of vision include:

Open angle glaucoma - other eye is often affected


Giant cell arteritis - associated with headache, jaw claudication, scalp tenderness. High
ESR. Usually affects both eyes
Retinal artery occlusion - acute, with maximal deficit at the onset and variable resolution,
and a spot at the macula
Amaurosis fugax - transient blindness lasting minutes to hours. Associated with vascular
risk factors as usually embolic. May be described as a "curtain coming down"
Anterior optic neuropathy - painless, may be central field defect with colour disturbance a
prominent feature. Arteriosclerosis, hypertension and diabetes are risk factors
Optic neuritis - painful, colour vision affected. May not result in complete visual loss. May
be the first manifestation of MS, therefore requires appropriate investigation
Other causes - cytomegalovirus (CMV), toxoplasmosis in association with
immunodeficiency.

242. 51-year-old painter and decorator is referred to the clinic with "tremors".

Which of the following features would make a diagnosis of parkinsonism more likely
than essential tremor?

(Please select 1 option)

A response of the tremor to benzodiazepines

First degree relatives being affected with similar problems

Tremor restricted to the arms

Tremor that involves his voice

Unilaterality Correct

Essential tremor is usually generalised and symmetrical. It often affects the head
and voice.
First degree relatives would more commonly be affected if the diagnosis was
essential tremor.
Parkinson's tends to develop later in life. The neurological signs of rigidity and
bradykinesia are seen in Parkinson's, usually no neurological signs are seen in
essential tremor.
Parkinson's will respond to L-dopa, anticholinergics and dopamine agonists.
Essential tremor is often responsive to alcohol, benzodiazepines and beta-blockers.
Parkinson's sufferers often demonstrate micrographia, while essential tremor often
involves tremulous writing.
243. A 3-year-old child is investigated for recurrent chest infections with green sputum
production.

Her chest x ray reveals ring shadows and bronchial wall thickening of both bases.

What is the most likely diagnosis?

(Please select 1 option)

Aspiration pneumonia

Bronchiectasis Correct
Foreign body inspissation

Hypersensitivity pneumonitis

Tuberculosis

This patient has a history of recurrent chest infections with x ray appearances
suggesting bronchiectasis.
The most likely explanation for these features would be cystic fibrosis.

244. 6-month-old baby presents with vomiting, blood-stained stools and irritability.

On examination he has a tense abdomen and draws his knees up with palpation.

What is the most appropriate action you should take for this baby?

(Please select 1 option)

Abdominal x ray

Check serum amylase

Give antibiotics

Refer to paediatric surgeons Correct

Suppositories to relieve constipation

This child has features to suggest intussusception.

The most appropriate course of action is to refer immediately to a paediatric surgical unit.

Approaches to relieve the intussusception will initially entail attempts at air reduction and, if
this fails, surgery.

Risk factors for intussusception include viral infection and intestinal lymphadenopathy.

245. A 3-year-old boy attends a children's party and eats some peanuts. Almost
immediately he spits them out, and runs to his mother saying his mouth hurts. She notes
a rapidly developing raised itchy rash over his face, and that his eyes, lips and tongue
are swelling. He begins to have marked difficulty in breathing with audible stridor. An
ambulance is called.

On the arrival of the paramedics he has collapsed and appears unconscious. He has shallow
breathing at 10/min, HR 160/min (thready pulse), and is responding only to pain.
What is the most likely diagnosis?

(Please select 1 option)

Anaphylaxis Correct

Croup

Foreign body aspiration

Peritonsillar abscess

Retropharyngeal abscess

The history is of anaphylactic shock in response to peanuts.

After attention to airway, breathing and circulation management he requires IM adrenaline,


then IV steroids and antihistamines and nebulised salbutamol.

Fortunately anaphylactic shock is rare.

246. A 4-year-old girl develops a fever 14 days after returning from a family visit to India.
At the height of fever she feels shivery and cold. She has previously been well.

Full term normal delivery with no neonatal complications. Immunisations up to date. There is
no FH/SH of note.

On examination she has a fever to 39.8°C, Respiratory rate 20/min and pulse 100/min.
Chest and ENT examinations are normal. She is shivering violently. She has 3 cm spleen,
which is smooth and non-tender.

What is the most likely diagnosis?

(Please select 1 option)

HIV infection

Infectious mononucleosis

Kala-azar

Malaria Correct

Schistosomiasis

The history of fever, rigors and splenomegaly in a child returning from an endemic area
suggests malaria.

Thick and thin blood smears should be taken.


Malarial resistance is an increasing problem, and the latest information should be obtained
about the area of disease acquisition to ensure proper treatment.

247. A 14-year-old girl presents with muscle weakness, lassitude, anorexia and weight
loss over a six week period.

Suddenly she collapses and becomes cyanosed and clammy. She has previously been well.
Full term normal delivery with no neonatal complications. Immunisations up to date. Mother
has hypothroidism.

On examination she is apyrexial, with shallow breathing and thready pulse at 140/min. BP is
80/60 mmHg and oxygen saturation 85%. She has pigmentation along lines of pressure. She
responds to pain.

What is the most likely diagnosis?

(Please select 1 option)

Adrenal insufficiency Correct

Diabetes mellitus

Guillain-Barré syndrome

Muscular dystrophy

Myasthenia gravis

The history is of generalised unwellness for several weeks plus abnormal pigmentation,
followed by sudden collapse with shock and low blood pressure. The picture suggests acute
or chronic adrenal failure.

Addison's disease can occur as part of a polyendocrinopathy. Type 1 is associated with


cutaneous candidiasis and ectodermal dysplasia. Type 2 is associated with thyroid disease
and diabetes.

Treatment is vigorous salt replacement (N. saline) and hydrocortisone after baseline
endocrine tests are done (ACTH, cortisol, renin, aldosterone, 17-0H progesterone, adrenal
androgens).

248. A 63-year-old male presents to his general practitioner for his diabetic annual review.
He has generally been well.

He has a history of type 2 diabetes and hypertension for which he receives pioglitazone,
metformin, bendroflumethiazide and lisinopril. He continues to smoke despite advice.
On examination he has a blood pressure of 136/86 mmHg, has evidence of a peripheral
neuropathy in his feet but all pulses are palpable. He has a few scattered microaneurysms in
both fundi. Urinalysis reveals +++ blood and +protein.

An ultrasound of his renal system is requested and the US appearances of his bladder are
shown.

What is the most likely diagnosis?

(Please select 1 option)

Bladder diverticulum

Bladder tumour Correct

Candida infection

Prostate tumour

Urinary calculus

The haematuria and the appearances of an echogenic mass within the lower right region of
this USS suggest a bladder tumour, probably bladder cancer.

It is important to appreciate that proteinuria is typical of diabetic nephropathy and blood


should not feature.

Smoking is again a risk factor for bladder cancer as is occupational exposure to aniline dyes.

10491

Time taken: 02:54:51


17175
249. A 24-year-old man is evaluated for facial pain and fever lasting four days. Symptoms
began with an upper respiratory tract infection accompanied by purulent nasal discharge;
he then began to have pain over the right cheek and developed a fever of 38.4°C.

On examination he has a purulent nasal discharge and tenderness over the right maxilla. CT
shows right maxillary sinusitis.

Which of the following is the most effective antibiotic treatment for this patient?

(Please select 1 option)

Amoxicillin This is the correct answer

Cefuroxime

Ciprofloxacin

Co-amoxiclav Incorrect answer selected

Flucloxacillin

Acute bacterial sinusitis usually occurs following an upper respiratory infection that results in
obstruction of the osteomeatal complex, impaired mucociliary clearance and overproduction
of secretions.

Sinusitis can be treated with antibiotics, decongestants, steroid drops or sprays, mucolytics,
antihistamines, and surgery (lavage).

Studies have shown that 70% of cases of community-acquired acute sinusitis in adults and
children are caused by Streptococcus pneumoniae and Haemophilus influenzae.

Most guidance favours amoxicillin as first line treatment. There seems to be little evidence
base for this, however.

BMJ's clinical evidence website found three RCTs which showed no difference between
amoxicillin and placebo (in patients without bacteriological or radiological evidence of
sinusitis). However, there were no RCTs examining the effects of co-trimoxazole,
cephalosporins, azithromycin and erythromycin.

Second line therapies include ciprofloxacin and co-amoxiclav. First and second generation
cephalosporins are not generally favoured.
250. Which of the following is most characteristic of a patient who has vitamin C
deficiency?

(Please select 1 option)

Cheilosis and beefy red tongue

Diarrhoea and delusions

Ocular muscle palsy and dementia

Paraesthesia and ataxia

Perifollicular haemorrhages and hyperkeratosis Correct

Vitamin C occurs primarily in fruits and vegetables.

Scurvy is caused by a deficiency of ascorbic acid. Symptoms of scurvy include inflamed and
bleeding gums, petechiae and impaired wound healing.

The most common cutaneous findings are:

follicular hyperkeratosis
perifollicular haemorrhages
ecchymoses
xerosis
leg oedema
poor wound healing, and
bent or coiled body hairs.

Cheilosis and red tongue would be more in keeping with vitamin B12 deficiency (as are ataxia
and paraesthesia) or iron deficiency.

Diarrhoea and delusions suggest vitamin B deficiency (pellagra).

Ocular muscle palsy and dementia suggest thiamine deficiency or Wernicke's


encephalopathy.

Vitamin C deficiency is not uncommon in the elderly population and awareness of the signs
and symptoms is important if the diagnosis is to be made. The diagnosis can be confirmed
with measurement of vitamin C concentrations in the white cell.
251. A 15-year-old girl attends with her mother as an emergency to the Emergency
department.

Her mother provides a history of 30 minutes of deteriorating breathlessness and facial


puffiness. She has otherwise been well except for eczema and she takes the oral
contraceptive. She is in obvious distress, her breathing is laboured and noisy.

What is the single most likely cause of breathlessness?

(Please select 1 option)

Acute epiglottitis

Angio-oedema Correct

Asthma

Pneumothorax

Pulmonary embolism

The history of noisy breathing is suggestive of stridor, which can be triggered by an allergic
reaction in an otherwise well adolescent, and the history of atopy is supportive of a diagnosis
of angio-oedema. Similarly, the rather abrupt history is again suggestive.

The main differential diagnosis is asthma where one would expect expiratory wheeze;
however the silent chest is an ominous feature in acute severe asthma.

252. A 70-year-old lady presents with a painful swelling of the left breast.

Her GP has treated the lesion, which has been present for four weeks with three courses of
antibiotics but the erythema and tenderness have not improved. On examination there is
erythema and oedema of the breast and a tender mass is palpable together with lymph
nodes in the axilla.

What is the next step in management?

(Please select 1 option)

Aspiration of pus for microbiology assessment

Broad spectrum antibiotics

Incision and drainage

Referral to non-urgent breast clinic


Urgent mammogram Correct

The diagnosis of a breast abscess in a 70-year-old must be treated with suspicion as


abscesses are uncommon in this age group.

This is particularly the case if there is a suggestion of a mass, and lymphadenopathy is


present in addition to the erythema and oedema usually associated with an abscess.

In such cases an inflammatory breast cancer must be suspected. This requires a


mammogram or ultrasound as well as a tissue biopsy to confirm the diagnosis prior to
deciding upon treatment options.

253. A 73-year-old lady presents with abdominal discomfort and distension. She claims
not to have opened her bowels in the last week or passed flatus for two days.

On examination, she appears slow and has a temperature of 35.5°C. Her pulse is
56/minute. Her abdomen is not tender.

Abdominal x ray shows dilated loops of small and large bowel.

What is the most likely cause of her presentation?

(Please select 1 option)

Colitis

Colonic cancer

Diverticulitis Incorrect answer selected

Pseudo-obstruction This is the correct answer

Small bowel obstruction

This woman appears to have pseudo-obstruction related to hypothyroidism as suggested


by the slowness and hypothermia.

Other causes of pseudo-obstruction include:

Hypokalaemia
Diabetes
Uraemia, and
Hypocalcaemia, etc.

254. A 25-year-old woman is admitted on the medical intake.

She is 10 weeks post partum and has been generally unwell for two weeks with
malaise sweats and anxiety.
On examination she is haemodynamically stable, and clinically euthyroid.

TFTs show the following:

Free T4 33 pmol/L (9-23)

Free T3 8 nmol/L (3.5-6)

TSH <0.02 mU/L (0.5-5)

What is the appropriate management?

(Please select 1 option)

Carbimazole 40 mg/day

Lugol's iodine

Propranolol 20 mg tds Correct

Propylthiouracil 50 mg/tds

Radioactive iodine therapy

The diagnosis here is likely to be post partum thyroiditis which tends to occur within the
three months of delivery followed by a hypothyroid phase at three to six months,
followed by spontaneous recovery in one third of cases. In the remaining two-thirds, a
single-phase pattern or the reverse occurs.

Management is centred on symptomatic treatment using beta-blockers for relief of


tremor or anxiety, and observation for the development of persistent hypo- or
hyperthyroidism.

Graves' disease is a less likely diagnosis based on the proximity to delivery and the
absence of any other signs to suggest Graves' ophthalmopathy, goitre and bruit.

Hashitoxicosis is a possibility but is less likely than Graves'.

255. Which of the following features gives the worst prognosis post-myocardial infarction?

(Please select 1 option)


Anterior wall MI complicated with continuing left ventricular failure and an exercise tolerance of 3 METS
on a predischarge exercise test Correct

Anterior Wall MI with continuing frequent ventricular ectopy

Anterior Wall MI with ejection fraction on ECHO of 25%

Inferior Wall MI complicated with complete heart block that required temporary pacing support till day 3

Inferior Wall MI complicated with right ventricular MI and right ventricular failure

Poor effort tolerance is the single most important feature that identifies poor prognosis
following myocardial infarction. Thus the role of a pre-discharge exercise test in
prognostication and risk-stratification.

Other features with strong adverse prognostic significance are resuscitation from a
secondary (after the first 24 hours) ventricular tachyarrhythmia and poor left ventricle (LV)
function on echocardiography.

Frequent ventricular activity does not carry prognostic significance by itself nor does the
occurrence of complete heart block in the setting of inferior wall MI.

256. Which of the following investigations would contribute the most to acute risk
stratification in a patient with unstable angina?

(Please select 1 option)

Coronary angiography

Echocardiography

Electrocardiography

Myocardial perfusion scanning

Troponin testing Correct

The adverse prognostic factors in unstable angina are

Labile ECG changes


Persistent angina in spite of maximal medical therapy
Clinical evidence of heart failure.

However, the most important adverse prognostic factor is troponin elevation. These are the
patients who should be referred for urgent coronary revascularisation.

257. A 49-year-old post-woman with a long-standing diagnosis of acute intermittent


porphyria (AIP) is referred by her general practitioner for advice on drug therapy.

She is a longstanding poorly-controlled hypertensive. The GP has provided two


electrocardiograms from consecutive weeks both of which demonstrate atrial fibrillation.
She has not had an acute exacerbation of AIP for over seven years.

Which of the following drugs would be unsuitable for use in her treatment?

(Please select 1 option)

Aspirin

Atenolol

Bumetanide

Digoxin Incorrect answer selected

Methyldopa This is the correct answer

AIP is often associated with hypertension.

Centrally acting drugs, such as the following, are contraindicated:

methyldopa
clonidine
angiotensin-converting enzyme (ACE) inhibitors
calcium channel blockers, and
furosemide.

Among the diuretics, the following have been used safely:

amiloride
bumetanide
acetazolamide
cyclopenthiazide, and
triamterene.

Digoxin, beta-blockers, heparin, and warfarin are also thought to be safe.

258. 17-year-old female presents with a severe headache of sudden onset associated
with nausea and photophobia.

On examination there was evidence of meningism. She had a previous history of migraine
without aura and was taking the oral contraceptive pill.

Which of the following is the most appropriate initial investigation?

(Please select 1 option)

Cervical spine x ray

CT scan of head Correct


Lumbar puncture

MRI scan of head

Skull x ray

This presentation suggests the diagnosis of subarachnoid haemorrhage.

The first line investigation is a head CT scan looking for evidence of haemorrhage.

If this is not demonstrated it should be followed by a lumbar puncture as subarachnoid blood


may not be evident on up to 10% of cases on CT scan.

259. A 25-year-old hairdresser is admitted to hospital with a first episode of headaches


and dislike of bright light of two days duration.

She also gives a history of a sore throat two days before this admission. On clinical
examination Kernig's sign is positive.

Which investigation would most probably establish the diagnosis?

(Please select 1 option)

Blood cultures

Chest x ray

CT brain

Lumbar puncture Correct

MRI brain

In suspected meningitis, lumbar puncture is the most definitive investigation.

Where raised intracranial pressure is suspected, a CT brain scan should be obtained prior to
performing a lumbar puncture.

In an emergency, antibiotic therapy is indicated for any patient with suspected bacterial
meningitis.

In this life-threatening illness, treatment should not be delayed to obtain a CT or a lumbar


puncture. Lumbar puncture results will not be significantly compromised if obtained
immediately after antibiotic administration, and even if cultures are negative, bacterial
antigens will remain positive.

260. An 18-year-old rugby player is brought to the emergency department with a painful
right shoulder sustained when he fell awkwardly during a tackle.
On examination there is fullness in the deltopectoral groove and lowering of the anterior
axillary fold. The acromion process appears to be prominent. His arm is slightly abducted
and externally rotated.

What is the most likely diagnosis?

(Please select 1 option)

Acromioclavicular joint subluxation

Anterior dislocation of the shoulder Correct

Fracture of the acromion process

Fracture of the greater tuberosity of the humerus

Posterior dislocation of the shoulder

Anterior (subcoracoid) dislocation is the commonest type of shoulder dislocation (this is in


contrast to the hip, where posterior dislocation is the most common). The usual mechanism
of injury is a fall onto the outstretched arm when the arm is abducted and externally rotated.
It can also result from various sporting injuries, commonly basketball and rugby.

Pain is severe and the patient is unwilling to attempt movements of the shoulder.

A swelling may be noticed in the deltopectoral groove (displaced head) with an undue
prominence of the acromion process. The arm is held in slight abduction and external
rotation. There may be flattening and loss of contour of the shoulder just below the acromion
process and lowering of the anterior axillary fold.

If the axillary nerve is damaged patients may present with loss of sensation over the upper,
outer aspect of the arm (regimental badge area).

Posterior dislocation of the shoulder, although uncommon, may occur as a result of direct
blow to the shoulder joint causing the humeral head to be displaced from the glenoid cavity.

It may result from violent trauma such as an electric shock or an epileptic convulsion. The
arm is usually held (or fixed) in internal rotation (note: external rotation in anterior
dislocation), which cannot be rotated outwards even as far as the neutral position.

The normal shoulder contour is lost and the anterior aspect of the shoulder appears flat (in
contrast to fullness in anterior dislocation).

261. A patient on a medical ward received a transfusion 48 hours ago for symptomatic
anaemia on a background of chronic renal disease and obstructive airways disease.

He gives a history of previous transfusions in the last year.

The patient has now dropped his Hb by 20 g/L compared to his pretransfusion level and
reports a dark coloured urine. The LDH and bilirubin are elevated.

Which of the following is this most likely to represent?


(Please select 1 option)

Acute haemolytic transfusion reaction Incorrect answer selected

Acute hepatitis as an infective complication

Delayed haemolytic transfusion reaction This is the correct answer

Non-haemolytic febrile transfusion reaction

Transfusion related graft versus host disease

This case is an example of delayed haemolytic transfusion reaction which occurs 24 hours
after the transfusion.

This happens in a patient who has been previously immunised by transfusions or pregnancy.
The antibodies are not detectable initially but become obvious as a secondary immune
response to the antigen exposure during the transfusion occurs.

The following should be carried out:

A haemoglobin level
Blood film
LDH
Direct antiglobulin test
Renal profile
Serum bilirubin
Haptoglobin
Urinalysis for haemoglobinuria.

The group and antibody screen should be repeated.

A transfusion-associated graft versus host disease and an acute hepatitis are unlikely given
the time frame - both would be expected to occur in a week or two.

This is also not an acute haemolysis which would be expected to occur during the
transfusion.

Given the rise in bilirubin and LDH this is a haemolytic reaction.

262. A 34-year-old man with normal baseline cardiac and respiratory function starts on the
ABVD (Adriamycin, bleomycin, vinblastine and dacarbazine) chemotherapy regimen for
his stage IIB Hodgkin's lymphoma.

He tolerated the first three cycles of the chemotherapy well.

After completion of the fourth cycle he presents with exertional dyspnoea and a dry cough.
He is afebrile, a chest x ray and ECG are normal.

Which of the following is the most likely diagnostic possibility?


(Please select 1 option)

Adriamycin related cardiomyopathy

Bleomycin related pulmonary fibrosis This is the correct answer

Hyperemesis and reflex cough related to dacarbazine

Pneumocystis jirovecii pneumonia Incorrect answer selected

Vinblastine related neurotoxicity

Bleomycin related pulmonary fibrosis is a major toxicity of the widely used ABVD regimen for
treatment of cardiac toxicity. A high resolution CT scan and pulmonary function tests are
required to diagnose this condition.

Although Adriamycin can cause cardiotoxicity this is unusual at the doses used in this
regimen and one would expect abnormalities on the ECG.

He is afebrile so pneumocytis is less likely though needs to be considered in the differential.

There is no history of hyperemesis given and a vinblastine neuropathy is unlikely to present


as acute shortness of breath.

263. Which of the following is the blood product with the highest risk of transmission of a
bacterial infection related to transfusion?

(Please select 1 option)

Cryoprecipitate

Factor VIII concentrates

Fresh frozen plasma

Packed red cells Incorrect answer selected

Platelets This is the correct answer

Since platelets are stored at room temperature (22°C), the risk of bacterial contamination is
highest in this blood product.

In contrast packed red cells are stored at an average of 4°C while fresh frozen plasma as
well as cryoprecipitate are stored at−20°C.

Factor VIII concentrates are heat inactivated freeze dried products with a minimal risk of
bacterial contamination.
264. A 52-year-old housewife presents to the hypertension clinic with hypertension that
has proved difficult to control.

Her general practitioner has started ramipril 1.25 mg, amlodipine 5 mg, bisoprolol 2.5 mg
and bendroflumethiazide 2.5 mg. Her systolic blood pressure is 152 mmHg, and it has
consistently ranged 150 - 160 mmHg.

The urine dipstick is normal. Renal function and electrolytes are normal. Her ECG showed
borderline left axis deviation but is otherwise normal in rhythm and complexes, and a
subsequent echocardiogram demonstrates good ejection fraction with no significant systolic
or diastolic impairment.

What is the best treatment option for this patient?

(Please select 1 option)

Add spironolactone 2.5 mg

Add isosorbide mononitrate 25 mg BD

Increase bendroflumethiazide to 5 mg

Increase ramipril to 2.5 mg Correct

Venesection

Use of multiple anti-hypertensives at low doses is preferable to having fewer tablets at


higher doses, in view of the synergistic effectiveness of targeting several underpinning
mechanisms of hypertension.

The ramipril dose can be increased, as the renal function is normal and 1.25 mg is a modest
dose.

Repeat renal function is advised following any increase in ACE inhibition.

Spironolactone is indicated in heart failure, at the higher dose of 25 mg.

Other indications include portal hypertension-induced abdominal ascites, nephrotic


syndrome and hyperaldosteronism syndrome. Renal function is normal, making the
possibility of hyperaldosteronism (which features a hypokalaemic alkalosis) remote.

Nitrates are indicated in angina, as an adjuvant in heart failure, but not solely for
hypertension in view of tolerance to their antihypertensive action.

The echocardiogram excludes systolic and diastolic heart failure, and thus the indication for
spironolactone or nitrates.

Bendroflumethiazide above the standard dose of 2.5 mg OD has no beneficial effects in


blood pressure profile and runs an increased risk of side effects (exacerbating gout,
diabetes, lupus and renal impairment).

Therapeutic venesection has no role in the management of chronic hypertension.


Other antihypertensive options include adding another diuretic (metolazone or bumetanide
or two potent loop diuretics), alpha blocker (doxazosin) or beta blockers.

265. A 69-year-old male smoker complains of chest pain associated with shortness of
breath 72 hours post left hemicolectomy.

On examination he has full air entry in his chest. Full blood count, U&E and troponin have
been sent.

His observations reveal:

Pulse 110 bpm, regular

Blood pressure 100/75 mmHg

Respiratory rate 32

Temperature 36.5°C

pO2 8.1 kPa

pCO2 3.2 kPa

pH 7.5

What is the most likely diagnosis?

(Please select 1 option)

Chest infection

Myocardial infarction

Pneumothorax

Postoperative anaemia

Pulmonary embolism Correct

With the information currently available you have to treat as a PE, because he is hypoxic
despite his tachypnoea with low pCO2 and is apyrexial.

Treatment with supplemental oxygen and heparin should begin whilst waiting for FBC, U&E
and troponin to becomeavailable.

A chest x ray and ECG should be performed, and if PE remains the most likely diagnosis a
CT pulmonary angiogram/VQ scan should be performed
266. A 42-year-old pregnant lady early in the second trimester attends clinic for the results
of her amniocentesis.

The results show a low serum alpha-fetoprotein (AFP), low unconjugated serum oestriol and
elevated serum human chorionic gonadotrophin (hCG) level.

What is the most likely diagnosis?

(Please select 1 option)

Cri du chat syndrome

Down's syndrome Correct

Klinefelter syndrome

Tay-Sachs disease

Turner syndrome

These findings suggest that the fetus has Down's syndrome associated with trisomy 21. It is
strongly associated with maternal age.

Causes of an isolated high AFP include:

fetal abdominal wall defects


multiple pregnancy, and
bleeding in pregnancy.

267. A 78-year-old man presents with a six month history of forgetfulness. His wife reports
he occasionally gets lost when out walking around their neighbourhood, has been
misplacing items in the home and on a number of occasions left the front door ajar when
he had gone out for his walk.

Neurological examination is otherwise unremarkable.

What is the most likely diagnosis?

(Please select 1 option)

Alzheimer's disease Correct

Creutzfeldt-Jakob disease
Hypothyroidism

Post-ictal confusion

Pseudodementia

Dementia is a term that describes a progressive and pervasive loss of a number of different
cognitive capabilities. Alzheimer's dementia patients show deficits of:

visual-spatial skill
memory and cognitive capabilities (for example, problem solving, word finding and
speech)
navigation
arithmetic
writing
reading.

Alzheimer's disease is caused by a progressive neuronal damage, accumulation β-amyloid


peptide, senile plaques and neurofibrillary tangles.

268. A 40-year-old woman with acute schizophrenia who recently commenced treatment
develops stiffness of limbs, fever and increased confusion.

With which antipsychotic would you associate these side effects?

(Please select 1 option)

Chlorpromazine Correct

Fluoxetine

Olanzapine

Quetiapine

Sertindole

Chlorpromazine and other neuroleptic antipsychotic drugs have antidopaminergic effects.

They cause hyperprolactinaemia with accompanying hypogonadism, as well as neuroleptic


malignant syndrome (hyperthermia, muscular rigidity and altered consciousness).

269. A 25-year-old female presents with postnatal depression and refuses treatment.

What form of consent should be obtained in order to treat this patient?

(Please select 1 option)


Consent from carer

Consent from court of law

No consent required This is the correct answer

Verbal consent required Incorrect answer selected

Written consent required

In actuality no consent is required in this case as the patient is suffering from a mental health
issue which may cloud her judgement and so she can be sectioned. Also, it is important to
act in her best interests as there is a baby's health at stake.

However, best practice guidelines would recommend that one obtains approval for one's
actions from the next of kin if possible. Again, this is not required by law, as they cannot
actually consent on behalf of the patient.

A good knowledge of these consent guidelines is essential today.

270. A 70-year-old man complains of choking, nasal regurgitation and coughing during
meals. His wife has noticed a change in his voice and he has a spastic gait.

What is the most likely diagnosis?

(Please select 1 option)

Carcinoma of the oesophagus Incorrect answer selected

Herpes simplex oesophagitis

Motor neurone disease This is the correct answer

Pharyngeal pouch

Systemic sclerosis

This man is demonstrating signs of bulbar palsy; 25% of motor neurone disease (MND)
presents with bulbar palsy.

Dysphagia with MND is characteristically demonstrates regurgitation of liquids through the


nose.

271. A 45-year-old lady presents with a two week history of dull retrosternal chest pain.

Clinical examination is unremarkable as is the ECG. A CXR demonstrates an air fluid level
behind the heart.

Which hernia best fits this clinical case?


(Please select 1 option)

Epigastric

Hiatal Correct

Paraumbilical

Spigelian

Umbilical

Hiatal hernias may be either sliding or paraoesophageal. In the former, the whole of the
stomach moves proximally whereas in the latter, a portion of the stomach, rolls up alongside
the oesophagus.

The symptoms of retrostenal chest pain in sliding hernias may be treated by proton pump
inhibitors but if the pain is poorly controlled and persistent there may be a role for surgery.

For paraoesophageal hernias, early repair should be considered as there are risks of
strangulation of the stomach.

A spigelian hernia is quite rare, it is placed anatomically due to an area of weakness just
lateral to the posterior sheath of the rectus muscle. Preperitoneal fat emerges through the
defect in the spigelian fascia, so that the peritoneum comes through the fascia.

272. A 61-year-old man with a BMI of 22 kg/m2 and a 13 year history of type 2 diabetes
complains of weight loss, lethargy and polydypsia.

Present medication is metformin 1g bd, gliclazide 160 mg twice daily, and acarbose. HbA1c is
91 mmol/mol.

What is the most appropriate therapy?

(Please select 1 option)

Basal bolus - 4 times daily insulin regime

Glibenclamide

Orlistat - Xenical

Pioglitazone

Twice daily insulin Correct

This patient is experiencing features of insulin deficiency and should be commenced on


insulin.

A twice daily regime is usually used when converting patients to insulin therapy.
273. An 87-year-old woman is admitted with a dense stroke. She has a history of
Alzheimer's disease and has been a nursing home resident for the last three years.

She is deteriorating rapidly and the family would like her to be resuscitated if she has a
cardiac arrest.

What is the most appropriate action in this case?

(Please select 1 option)

Respect patient's wishes

Respect wishes of next of kin Incorrect answer selected

Treat under common law

Withhold resuscitation in the event of cardiac arrest This is the correct answer

Withhold treatment

'Do Not Attempt Resuscitation' (DNAR) orders are appropriate in cases where successful
resuscitation is unlikely. Generally the views of the patient, family and close friends should
be taken into consideration but the ultimate responsibility rests with the medical team and
most senior doctor involved with the patient's care.

DNAR orders are quite specific that there should be no medical intervention in the event of a
cardiac arrest. However this does not automatically indicate that any other treatment should
be witheld as a result of this decision.

274. A 35-year-old man attends the Emergency department after sustaining minor injuries
following a scuffle at a nightclub.

He is drunk and aggressive and has assaulted a nurse.

What is the most appropriate action to take in this case?

(Please select 1 option)

Call police Correct

Implement Mental Health Act

Implement Public Health Act

Seek court order

Seek psychiatric review

This man is clearly a threat to the safety of the staff. The police should be called to settle the
situation. The staff may then assess the patient safely.

It is likely that he is under the influence of alcohol but it is important to rule out other causes.
275. A 70-year-old woman presents with a three month history of vaginal bleeding. Pipelle
endometrial sampling and curettage reveal adenocarcinoma.

What is the most appropriate treatment for this patient?

(Please select 1 option)

Colposcopy

Subtotal hysterectomy

Suction curettage

Total abdominal hysterectomy and bilateral salpingo-oophorectomy Correct

Uterine artery embolisation

Total abdominal hysterectomy and bilateral salpingo-oophorectomy is the definitive form of


treatment in cases of uterine/endometrial malignancy.

Some recognised risk factors for endometrial carcinoma include:

increasing age
oestrogen therapy
late menopause
nulliparity
polycystic ovary syndrome (chronic anovulation)
obesity
diabetes mellitus, and
family history of endometrial, ovarian, breast, or colon cancer.

Endometrial sampling is gold standard for evaluation of endometrial neoplasia, though


routine sampling is not advised.

276. A 46-year-old obese female complains of recurrent pain under her right lower ribs
after eating fatty foods.

What would be the investigation of choice?

(Please select 1 option)

Abdominal x ray

Blood glucose

ERCP

Laparoscopy
Ultrasound abdomen Correct

This woman most likely has biliary colic and the investigation of choice is ultrasound of the
abdomen. This may show stones in the gall bladder, a dilated common bile duct or dilated
intrahepatic ducts (suggesting a blockage distally).

A thickened gall bladder with or without surrounding free fluid, would suggest cholecystitis.

The five F's associated with gall stones include:

Forty years of age


Fatty disposition
Female gender
Fair skin tone, and
Fertile.

This alludes to the common patient presentation of an overweight, middle-aged, white


women with a history of several pregnancies.

As politically incorrect as it sounds, a Western diet has been implicated with production of
gall stones. Higher oestrogen levels may also contribute as incidence of gallstones
decreases significantly in the post menopausal population.

277. A 72-year-old lady with end stage lung cancer is having difficulty breathing due to
secretions.

What would be the most appropriate palliative treatment?

(Please select 1 option)

Codeine

Hyoscine butylbromide Correct

Morphine

Prochlorperazine

Tricyclic antidepressants

Hyoscine may be used to help dry secretions in lung cancer as it acts as an antimuscarinic.

Hyoscine butylbromide causes less sedation than hyoscine hydrobromide. Both are given by
subcutaneous injection or infusions.

278. A 30-year-old female presents for follow up after a cervical smear was done. The
smear was histologically identified as CIN2.

What is the most appropriate management for this patient?


(Please select 1 option)

Cone biopsy

No action required

Refer for colposcopy Correct

Repeat cervical smear in six months

Treat with oral metronidazole

Screening for cervical carcinoma may prevent the development of disease. According to
NICE guidance on Cervical cancer - cervical screening (TA69), the screening interval using
liquid based cytology (rather than PAP) is three yearly for women less than 50, and five
yearly for women above 50. If a smear shows no endocervical cells then this should be
repeated.

Reasons for this include:

inadequate preparation or sampling, and


sampling at the wrong time of the menstrual cycle.

For severe dyskariosis, or carcinoma in situ, the patient requires treatment and hence
referral for colposcopy.

Clue cells are vaginal epithelial cells that appear fuzzy without distinct edges under a
microscope. When bacterial vaginosis is present, more than 20% of the sample vaginal
epithelial cells are clue cells.

Bacterial vaginosis is appropriately treated with metronidazole therapy. Although


trichomoniasis is treated with metronidazole, the patient needs appropriate investigation for
other sexually transmitted diseases.

279. A 72-year-old woman has a right-sided headache aggravated by brushing her hair.
She says she has been generally unwell for a few months with aching muscles.

Erythrocyte sedimentation rate (ESR) is elevated.

What is the most appropriate investigation?

(Please select 1 option)

Computed tomography scan


Electromyography

Muscle biopsy

Skull x ray

Temporal artery biopsy Correct

This patient has temporal arteritis along with features of polmyalgia rheumatica. ESR is
invariably elevated. The temporal artery is usually hardened and palpable, as well as being
characteristically tender to touch (or comb the hair).

If there is associated visual disturbance urgent referral to an ophthalmologist is required.

Treatment is with high dose corticosteroids. Remember to prescribe proton pump inhibitor
and bone protection.

A temporal artery biopsy should be arranged for the patient as soon as possible to help
confirm the diagnosis. A positive biopsy has a 100% sensitivity, however due to skip lesions
there is a high proportion of false negatives.

Follow up should be in the rheumatology clinic.

280. This 58-year-old male attends the clinic complaining of headaches and lethargy.

Examination reveals a blood pressure of 210/110 mmHg.

What does the fundal image show?

(Please select 1 option)

Diabetic retinopathy

Optic atrophy

Papilloedema Correct

Retinal artery occlusion


Retinal vein thrombosis

There is blurring of the disc margin which appears rather raised, signifying papilloedema.
This patient therefore has accelerated/malignant hypertension requiring immediate
treatment.

Other features of hypertensive retinopathy include:

Silver wiring of the retinal arteries


AV nipping
Haemorrhages
Hard exudates, and
Cotton wool spots.

281.

What rhythm is shown in this electrcardiogram?

(Please select 1 option)

Atrial fibrillation

Second degree heart block (Mobitz type 2)

Sinus arrhythmia

Sinus bradycardia

Third degree heart block (complete heart block) Correct

This is complete heart block.

There is no relationship between P waves and the QRS.

The ventricular rate is slow and represents a ventricular escape rhythm.

282. Psychiatric illness rather than an organic brain disorder is suggested by which of the
following?

(Please select 1 option)

A family history of major psychiatric illness Correct


Clouding of consciousness

Impaired short term memory

No previous history of psychiatric illness

Onset for the first time at the age of 55 years

This question tries to establish typical features of psychiatric disease such as


depression/schizophrenia rather than organic brain disease, for example, dementia.

Loss of short term memory and an older age are more typical of organic brain disease.

However a family history is especially associated with depressive illness and schizophrenia.

283. REVERSE SPLITTING OF S2

In this condition, P2 occurs before A2.

On expiration, A2 is delayed such that it occurs after P2. Inspiration causes P2 to be delayed and
the split is diminished.

Possible causes of a delayed A2 include:

 electrical delay - left bundle branch block


 systolic hypertension
 severe aortic stenosis or HOCM
 patent ductus arteriosus
 left heart failure

In patients with aortic stenosis, A2 is often very soft or even absent, in such circumstances there
appears to be a single component to the second heart sound.

Note that an early P2 may occur in type B Wolff-Parkinson- White syndrome (1).

Reference:

 Braunwald E. Heart Disease - A Textbook of Cardiovascular Medicine,WB Saunders.

284. A 59-year-old truck driver asks for your advice in diabetes clinic. He is keen to make
some lifestyle interventions in order to improve his lifespan.

He has a BMI of 32 kg/m2, smokes 20 cigarettes a day, suffers with angina, and has a
fasting BM of 9. He takes no medication.

Which single intervention will be most suitable?

(Please select 1 option)


Aspirin This is the correct answer

Cerivastatin

Insulin

Nicotine replacement therapy (NRT) Incorrect answer selected

Nutritionist support

This is a question based on current clinical trial evidence in diabetes.

The strongest evidence is in favour of aspirin (20% reduction in cardiac events over ten
years in this patient group). The Royal College is hardly going to advocate a nutritionist as a
means of improving on their wonderful treatment and besides, there are no mortality data.

Insulin is not indicated until diet and exercise and oral hypoglycaemic agents have failed.

NRT has a 15-20% success rate, only double that of willpower. There is no direct evidence
to show improvement in longevity, although it is reasonable to assume that it will help.

Cerivastatin has been removed worldwide following an increased risk of myositis.

We do not know this man's cholesterol level.

285. You are called to the death of a 90-year-old male on the care of the elderly unit.

He has a long history of breathlessness associated with heart failure and the nurses state
that he suddenly collapsed and died. He was not resuscitated.

Which of the following is an accepted cause of death that you may place on a death
certificate?

(Please select 1 option)

Cardiac arrest Incorrect answer selected

Heart failure

Ischaemic heart disease This is the correct answer

Old age

Respiratory arrest
Cardiac arrest, syncope, apnoea, respiratory arrest, heart, liver or kidney failure are all
modes, rather than underlying causes of death, and thus unacceptable for a death
certificate.

Generally, a cause of death is the pathological condition responsible for death, for example,
ischaemic heart disease or myocardial infarction.

'Old age' should be avoided on death certificates.

286. A 78-year-old male is brought to the Emergency department and has a witnessed
seizure in the resuscitation room.

His blood glucose is recorded as 1.0 mmol/L. He does not have diabetes, nor other
significant medical history.

He is given 50 ml of 50% dextrose and he slowly recovers over the next one hour. A serum
cortisol concentration later returns as 800 nmol/L (120-600).

Which of the following would be the most relevant investigation for this man?

(Please select 1 option)

Chest x ray

CT head scan

Electrocardiogram

Prolonged 72 hour fast Correct

Short Synacthen test

The historical and biochemical evidence here suggests a diagnosis of spontaneous


hypoglycaemia and the most likely cause would be an insulinoma. However, one would wish
to exclude possible drug administration and although not mentioned here, a sulphonylurea
screen should be undertaken.

He has presented with symptomatic hypoglycaemia, is not diabetic and therefore should not
have received insulin or a sulphonylurea.

There is nothing to suggest alcohol or drug misuse. Similarly, there is nothing to suggest
sepsis.

However, to prove a diagnosis of spontaneous hypoglycaemia a prolonged fast is required


and, should he develop hypoglycaemia, measurement of insulin and C peptide will be
needed to confirm the diagnosis.

The appropriate cortisol response during his hypoglycaemic episode (cortisol 800) excludes
hypoadrenalism.
287. A 19-year-old student complains of involuntary movements of her hands. You
diagnose chorea.

Which of the following investigations is not indicated?

(Please select 1 option)

Antinuclear antibodies

Antistreptolysin O titre

B12 levels This is the correct answer

Haemoglobin electrophoresis Incorrect answer selected

Serum caeruloplasmin levels

The movement disorder is chorea.

Sickle cell disease is associated with moyamoya which is a recognised cause.

Systemic lupus erythematosus, rheumatic fever, and Wilson's disease are all associated
with chorea; B12 deficiency is not.

288. Which of the following lipid abnormalities are most likely to be detected in a type 2
diabetic patient?

(Please select 1 option)

Elevated HDL concentrations

Elevated LDL concentrations Incorrect answer selected

Large buoyant LDL molecules

Reduced triglyceride concentrations

Small dense LDL molecules This is the correct answer

In type 2 diabetes increased cholesteryl ester transfer protein (CETP) activity results in the
transfer of triglycerides from very low-density lipoprotein (VLDL) to high-density lipoprotein
(HDL) and low-density lipoprotein (LDL).

This results in small dense LDL which is more atherogenic being able to be oxidised more
readily and penetrate endothelium and macrophages.

LDL is not typically elevated in type 2 diabetes, although there are qualitative changes as
indicated above. HDL is typically low in the type 2 diabetic patient.
289. A 23-year-old man is brought to the Emergency department with a gunshot injury to
his right upper thigh.

On examination, the wound lies about 4 cm below the inguinal ligament. The vascular status
of the limb is normal. Local neurological examination reveals numbness over the anterior
thigh and medial aspect of his leg. Although he is able to flex the hip, he is unable to extend
the knee on the affected side. The knee jerk is diminished but the ankle jerk is preserved.

Which of the following nerves is most likely to be affected in this patient?

(Please select 1 option)

Common peroneal nerve

Femoral nerve Correct

Lateral cutaneous nerve of thigh

Pudendal nerve

Saphenous nerve

The femoral nerve arises from the lumbar plexus (L2-4).

It exits the pelvis by passing beneath the medial inguinal ligament to enter the femoral
triangle, after penetrating the psoas muscle.

In the femoral triangle, it lies just lateral to the femoral artery and vein.

It may be injured by the following:

Gunshot wounds
Direct penetrating wounds
Traction during surgery
Catheterisation of the femoral artery
Massive haematoma within the thigh
Nerve injury secondary to femoral nerve block
Psoas abscess
Fractured pelvis
Dislocation of the hip

Apart from trauma, it may be affected in patients with diabetes mellitus (diabetic neuropathy)
and lumbar spondylosis.

The femoral nerve innervates the iliopsoas, which helps in flexion of the hip, and the
quadriceps, which help in extension of the knee.

The motor branch to the iliopsoas originates in the pelvis proximal to the inguinal ligament
and injury at or above this level leads to loss of hip flexion.
The sensory branch of the femoral nerve, the saphenous nerve, innervates the skin over the
medial aspect of the thigh and the anterior and medial aspects of the calf. Hence femoral
nerve injury results in numbness over the medial aspect of the thigh and the antero-medial
aspect of the leg.

Motor loss includes weakness of the quadriceps muscle and decreased patellar reflex (knee
jerk) (the ankle jerk is preserved since it is innervated by the tibial nerve [S1-S2]). In
longstanding, sub-acute injuries, the patient finds that the knee gives way on walking and
has difficulty climbing stairs.

290. Which statement regarding the syndrome of inappropriate ADH secretion (SIADH) is
true?

(Please select 1 option)

Oedema may occur due to fluid overload

Plasma sodium is usually normal Incorrect answer selected

Plasma urate levels are elevated

Serum globulins are high

Urine osmolality is greater than 500 mOsmol/kg This is the correct answer

SIADH produces inappropriate water retention despite low plasma osmolality.

Therefore, serum levels of Na, K, urea, creatinine and urate are all lowered. However,
plasma protein levels are normal, so dependent oedema is rare.

Urine is inappropriately concentrated.

Causes include

Tumours secreting ADH (small cell lung cancer, prostate, pancreas, thymus, lymphoma)
Drugs
Central nervous system (CNS) injury including Guillain-Barré syndrome
Lung infections and
Acute intermittent porphyria (AIP).

291. You are called to resuscitation in the emergency department to see a man with chest
pains.

The ECG reveals a broad complex tachycardia.

Which of the following suggests a diagnosis of supraventricular tachycardia (SVT) as


opposed to ventricular tachycardia (VT)?

(Please select 1 option)


Cannon a waves

Capture beats

Monophasic QRS

QRS duration >140msec

Termination with adenosine Correct

Points in favor of VT include

Any evidence of atrioventricular (AV) dissociation (for example, cannon a waves on


jugular venous pressure [JVP], varying intensity of S1)
Fusion and/or capture beats
Left axis deviation (between 90 and 180 degrees)
QRS duration greater than 140 msec
Specific QRS contours (for example, monophasic QRS in V1).

Points in favor of SVT

Long-short cycle sequence


Slowing or termination by increasing vagal tone or with adenosine (although there are
some rare VT that do terminate also)
Trileafed QRS in V1
RP interval less than 100 msec.

(In patients with a history of ischaemic heart disease, treat as VT!).

292. A 28-year-old male attends the Emergency department following an alleged assault.

He reports sustaining a blow to his nose that caused a small nosebleed and considerable
swelling and deformity. He complains of difficulty breathing through his left nostril.

Your examination confirms that there is no active bleeding from the nose. There is some
deviation of the nasal bones to the left and no septal haematoma.

Which of the following is the most appropriate cause of action?

(Please select 1 option)

Arrange an ENT follow up appointment for within the next one week This is the correct answer

Pack the patient's nose

Reassure the patient that his symptoms will resolve as the swelling subsides and needs no further
action
Refer the patient for emergency admission

Request x rays of the nasal bones Incorrect answer selected

Isolated nasal injuries are a common presentation to the Emergency department. Patients
rarely require emergency admission.

There are three major exceptions to this where patients will require emergency admission.
These are patients with:

a septal haematoma
a compound nasal fracture, or
associated epistaxis.

Nasal bone x rays are complex and unnecessary. Diagnosis of a nasal fracture is entirely
clinical.

Uncomplicated patients are best reviewed after five days in the ENT clinic, when associated
swelling has subsided and it can be determined whether manipulation of the fracture is
appropriate.

Traumatic epistaxis can be serious and may require packing if there is active bleeding.

293. 40-year-old man presents with a thyroid nodule. Five years ago he underwent
adrenal surgery to remove a phaeochromocytoma. His mother has had parathyroid
surgery.

Which tumour marker would most likely be elevated?

(Please select 1 option)

Alpha feto-protein (AFP)

Alpha subunit human chorionic gonadotrophin

Carcinoembryogenic antigen (CEA)

Serum calcitonin Correct

Serum thyroglobulin

Medullary thyroid cancer (MTC) needs to be excluded.

MTC is inherited in an autosomal dominant fashion, in about 20% of cases.

Serum calcitonin which is secreted by the parafollicular cells of the thyroid may be elevated
in MTC and is a useful tumour marker.

294. A 68-year-old man attends the clinic with a swelling closely related to his right iliac
fossa terminal ileostomy. He had previously undergone an emergency total colectomy for
toxic megacolon.
On examination there is a soft reducible swelling with a positive cough impulse.

Which hernia best fits this clinical case?

(Please select 1 option)

Epigastric

Hiatal

Incisional

Parastomal Correct

Spigelian

Parastomal hernias are probably under recorded and occur as a segment of bowel emerges
alongside the stoma.

For ileostomies, this often causes severe pain whilst for colostomies the symptoms are more
commonly related to poorly fitting appliances.

The diagnosis can be confirmed with CT and the stomal hernia repaired.

Next question

295. What is the most likely anaesthetic or analgesic cause of sudden maternal
hypotension?

(Please select 1 option)

Epidural anaesthetic This is the correct answer

Intramuscular pethidine

Nitrous oxide

Pudendal block

Spinal anaesthetic Incorrect answer selected

In spinal anaesthesia the needle (25G or smaller pencil point, Whitacre or Sprotte) parts
dura rather than tears it. In an epidural the needle is supposed to inject around the dura but
may inadvertently penetrate it.

Maternal hypotension is more likely to be due to dural penetration during an epidural


anaesthetic as this is the generally intended procedure.

Postdural puncture headache appears to be higher in association with spinal (3%) than
epidural (1%).
296. A 30-year-old lady presents with lower abdominal pain and has an intrauterine
device.

What is the most likely causative organism?

(Please select 1 option)

Actinomyces israelii This is the correct answer

Chlamydia trachomatis Incorrect answer selected

Gardnerella vaginalis

Mycobacterium tuberculosis

Neisseria gonorrhoea

Actinomyces israelii is a Gram positive rod which is recognised to colonise intrauterine


contraceptive devices (IUCDs).

A. israelii is normally colonised in the vagina, colon, and mouth. Infection is established first
by a breach of the mucosal barrier during various procedures for example dental or
gastrointestinal, or pathologies such as aspiration or diverticulitis.

Actinomyces tends to cause a sinus tract which leads to a fibrotic lesion, described to be
'wooden' in nature which is characteristic of infection with Actinomyces.

297. A 30-year-old lady presents with lower abdominal pain and has an intrauterine
device.

What is the most likely causative organism?

(Please select 1 option)

Actinomyces israelii This is the correct answer

Chlamydia trachomatis Incorrect answer selected

Gardnerella vaginalis

Mycobacterium tuberculosis

Neisseria gonorrhoea

Actinomyces israelii is a Gram positive rod which is recognised to colonise intrauterine


contraceptive devices (IUCDs).

A. israelii is normally colonised in the vagina, colon, and mouth. Infection is established first
by a breach of the mucosal barrier during various procedures for example dental or
gastrointestinal, or pathologies such as aspiration or diverticulitis.

Actinomyces tends to cause a sinus tract which leads to a fibrotic lesion, described to be
'wooden' in nature which is characteristic of infection with Actinomyces.
298. A 30-year-old nulliparous woman complains of difficulty conceiving. She and her
husband have been trying for a baby for over a year.

She has regular periods which are heavy, lasting for eight days. Her husband has seen a
urologist who has not found anything abnormal.

Her ovulation test is normal. Ultrasound reveals a 5 cm diameter submucous fibroid.

What is the most appropriate treatment in this case?

(Please select 1 option)

Colposcopy

Myomectomy Correct

Oral misoprostol

Suction curettage

Uterine artery embolisation

The submucous fibroid may be impairing this woman's fertility and surgical excision is worth
considering. The clue to the presence of a fibroid in this question is the history of long and
heavy menses.

Generally couples are advised to try and conceive naturally for a year, if conception fails
after this period, fertility testing is warranted.

Uterine artery embolisation as a treatment is generally contraindicated when fertility is


desired although there have been reports of pregnancy following this procedure

299. A 53-year-old anaemic man presents in the outpatients with alternating diarrhoea and
constipation. He has lost 4 kg in weight over the past two months, and his stool guaiac
test is positive.

What is the most appropriate investigation after clinical examination and digital rectal
examination?

(Please select 1 option)

Barium enema

Jejunal biopsy

Oesophagogastroduodenoscopy

Sigmoidoscopy Correct

Thyroid function test


The stool guaiac test detects the presence of occult blood in the stool (bowel movement).

The investigation of choice after clinical examination and digital rectal examination
(dependent on resources) is rigid sigmoidoscopy or, ideally, flexible sigmoidoscopy.

The majority of large bowel tumours are in the left colon and rectum. These symptoms
favour a left-sided lesion, so a colonoscopy or CT enema might not be needed.

If a tumour were found, then a staging CT would be needed.

300. A 56-year-old man with metastatic bowel carcinoma is being managed palliatively.
He takes paracetamol and tramadol but is not getting satisfactory pain relief.

What is the most appropriate treatment?

(Please select 1 option)

Codeine

Ibuprofen

Intravenous morphine

Oramorph Correct

Tricyclic antidepressants

Analgesia should follow the World Health Organization (WHO) cancer pain ladder for adults.

Although it has been proven that bone pain usually can be settled with a combination of non-
steroidal anti-inflammatory drugs (NSAIDs) and weak opioids, the treatment for moderate to
severe pain (step 3 on the pain ladder) is morphine/oxycodone.

When prescribing pain killers you should always try to follow the analgesic ladder; if a drug is
not providing satisfactory pain relief, a stronger drug should be used instead.

Morphine and other opioid drugs cause constipation. They should always be prescribed with
laxatives as a preventative measure.

301. Which of the following is characterised by postcricoid web arising, and is associated
with iron deficiency anaemia?

(Please select 1 option)


Achalasia

Barrett's oesophagus

Hiatus hernia

Plummer-Vinson syndrome Correct

Scleroderma

Plummer-Vinson syndrome (also called the Patterson-Kelly-Brown syndrome), is a relatively


uncommon condition, which has a higher incidence in post-menopausal women.

The presenting symptoms are those of dysphagia secondary to a postcricoid (oesophageal)


web and iron deficiency anaemia. Treatment is with iron supplementation and mechanical
widening of the web, to aid swallowing.

Follow up in these patients is important as there is a higher incidence of squamous cell


carcinoma in the pharynx and oesophagus.

302.

From the given list of options select the diagnosis for the above figure, which illustrates the
appearance of the axilla of a patient with type 2 diabetes:

(Please select 1 option)


Acanthosis nigricans Correct

Cellulitis

Chronic fungal infection

Necrobiosis

Pyoderma gangrenosum

Acanthosis nigricans is a hyperpigmented overgrowth of the epidermis, usually in the flexural


areas of the groin, axilla and neck.

It is associated with insulin resistance, a possible mechanism being the effect of raised
insulin concentrations acting on IGF-1 receptors in skin promoting growth

303. An otherwise healthy 10-month-old baby is brought to clinic by her concerned


parents who state that two days ago she had come into close contact with another child
who has measles.

Which of the following is the most appropriate approach?

(Please select 1 option)


Nothing need be done now but she should receive her MMR vaccination at the usual time of
approximately 12 months Incorrect answer selected

She should have her measles serology checked now

She should receive measles immunoglobulin now

She should receive the MMR vaccination now This is the correct answer

She should receive the MMR vaccine now together with measles immunoglobulin

The child should receive the measles, mumps, rubella (MMR) vaccination now but the
parents should be informed that she may well develop measles.

It is best to reassure the parents that if she develops measles it is most likely to be self
limiting and is treated appropriately.

There is no point in assessing serology or giving immunoglobulin.

Single vaccines are not available on the NHS and the child is not immune to mumps or
rubella.

304. Which one of the following conditions would nothave to be notified to the consultant
responsible for communicable disease control?
(Please select 1 option)

Campylobacter food poisoning

E. coli H0157 food poisoning Incorrect answer selected

Pneumococcal meningitis

Pneumococcal pneumonia This is the correct answer

Vivax malaria

There are many consultants responsible for communicable disease control.

Examples of notifiable diseases are:

Measles, mumps, rubella (MMR)


Diphtheria, tetanus, and pertussis (DTP)
Food poisoning
Anthrax, and
Malaria.

It is a statutory obligation to notify Public Health of these diseases if identified.

305. Which of the following statements regarding thrombocytosis is correct?

(Please select 1 option)

Erythropoietin is the key hormone in the regulation of megakaryocyte differentiation

May occur as a response to exercise Correct

Occurs exclusively in essential thrombocythaemia

Secondary thrombocytosis is an indication for hydroxycarbamide therapy

The most common cause is essential thrombocythaemia

The most common cause of thrombocytosis is a reactive thrombocytosis.

Thrombocythaemia may occur in any of the myeloproliferative disorders, particularly


polycythaemia rubra vera (PRV).

Thrombopoietin is the key hormone in the regulation of megakaryocyte differentiation.

Secondary thrombocytosis does not place the patient at risk for haemostatic nor
cardiovascular events.
306. A 3-week-old girl on the neonatal unit is noted to have a fever and decreased
spontaneous movement. She was born at 26/40 gestation weighing 1.02 kg.

After a stormy course she has returned from the neonatal intensive care for convalescence.
She is unimmunised. There is no FH/SH of note.

Fever of 37.6°C

Respiratory rate 30/min

Heart rate 100/min

She has multiple venepuncture sites, and has redness over a site on the dorsum of the right
foot and over the shin, which is swollen. She cries when this area is touched. She lies still in
her cot and she appears well perfused.

What is the most likely diagnosis?

(Please select 1 option)

Bacteraemia

Cellulitis

Osteomyelitis Correct

Rickets of prematurity

Septic arthritis

The history is of low grade fever with pseudoparalysis of the right leg and tibial inflammation.

This suggests an osteomyelitis in the right tibia complicating a previous infected drip site in
the foot. The infected drip site and the degree of extension of redness suggests this is more
serious than simple cellulitis.

USS, x ray, bone scan and blood cultures should be performed and IV antibiotics given to
cover Staph. aureus, Group BStreptococcus and Gram negative organisms.

307. A 5-month-old boy presents with florid red rash over his scalp, nappy area and trunk.
Despite this he seems well in himself and feeding well. He was born at term weighing 3.1
kg and there were no neonatal problems. He is fully immunised and there is no FH/SH of
note.
On examination he is apyrexial and well. The rash is florid red and confluent over his nappy
area. He has a crusty confluent covering over his scalp, extending onto his forehead.
Smaller 0.5-1 cm greasy lesions are present over his trunk.

What is the most likely diagnosis?

(Please select 1 option)

Acrodermatitis enteropathica

Ammoniacal dermatitis

Histiocytosis

Infantile eczema

Seborrhoeic dermatitis Correct

The picture is of extensive greasy scaly rash especially over the head (cradle cap) and
nappy area without systemic upset.

This is highly characteristic of seborrhoeic dermatitis.

Selenium shampoo and topical steroids usually result in rapid resolution.

308. In which one of the following diseases does the patient have to be isolated?

(Please select 1 option)

Glomerulonephritis (GN) - post streptococcal

Henoch-Schönlein purpura

Herpetic gingivostomatitis

Measles Correct

Rheumatic fever

Children with chicken pox and measles should be isolated, as there is a high risk of cross
infection.

Post streptococcal GN and rheumatic fever are mediated by immune complexes some time
after the initial infection.

Herpetic gingivostomatitis is spread when infected secretions are transmitted directly onto
stratified squamous epithelium of an uninfected individual. There is no aerosol spread.
309. A 22-year-old golf course groundsman develops sudden onset of breathlessness and
right-sided chest pain whilst maintaining the 18th hole. He is rushed to hospital, but his
condition worsens when being examined by a junior doctor.

She reported finding a trachea that was deviated to the left, and hearing very distant breath
sounds over his right lung. Following the intervention of a senior doctor, his condition
improves rapidly.

What is the most probable diagnosis?

(Please select 1 option)

Myocardial infarction

Pericarditis

Pneumonia

Pulmonary embolism

Tension pneumothorax Correct

Sudden onset of chest pain and respiratory distress in a previously fit young man should
alert one to the diagnosis of pneumothorax.

Tales of patients who developed a clinically obvious tension pneumothorax in hospital are
told frequently, so it is best to be able to recognise one if it occurs while you are actually
examining the patient!

The condition is treated by needle thoracocentesis in the second intercostal space. There is
no role for obtaining chest radiography prior to intervention.

310. An 18-year-old male is rushed into the Emergency department by a group of friends
who leave before they can be interviewed by medical staff.

He is semiconscious, has a respiratory rate of 8/min, a blood pressure of 120/70 mmHg and
a pulse of 60 bpm. He is noted to have needle track marks on his arms, and his pupils are
small.

What single initial treatment would you administer?

(Please select 1 option)

Adrenaline

Aspirin

Atropine
Dextrose

Naloxone Correct

These features of unconsciousness, respiratory depression plus small pupils suggest opiate
toxicity. This commonly occurs as the result of the abuse of street drugs such as heroin.

It should be relieved promptly with naloxone which is an opiate receptor antagonist.

Naloxone has a shorter half life than many opiates, so more than one dose may be
necessary.

311. A 19-year-old male attends the Emergency department complaining of a sudden left-
sided chest pain followed by dyspnoea. These symptoms developed whilst playing
football.

Which of the following investigations is most likely to provide a definitive diagnosis?

(Please select 1 option)

Blood gases

Chest x ray Correct

Echocardiogram

Electrocardiogram (ECG)

Pulmonary function tests

The history is suggestive of a pneumothorax, and of the available investigations a chest x


ray is the only test which will provide a definitive diagnosis.

ECG would be unlikely to reveal any abnormality and blood gases would be most likely to
reveal a highish O2 with lowish CO2 in the absence of any appreciable respiratory
compromise.

312. A 36-year-old Afro-Caribbean woman presents having suffered her fourth


miscarriage. She has a history of venous thrombosis.

She is positive for the lupus anticoagulant.

What is the likely diagnosis?

(Please select 1 option)

Antiphospholipid syndrome Correct

Bacterial vaginosis
Poorly controlled diabetes mellitus

Systemic lupus erythematosus

Uterine abnormality

Antiphospholipid syndrome is a cause of recurrent miscarriage.

It may present as:

Arterial or venous thrombosis


Livedo reticularis rash
Stroke
Adrenal haemorrhage
Migraine
Myelitis
Myocardial infarction, or
Multi-infarct dementia.

Anticardiolipin antibodies may be found.

Venous thrombi occur more often if lupus anticoagulant is positive and arterial thrombi if IgG
or IgM antiphospholipid antibody are positive. Long term warfarin is indicated.

Originally it was thought that up to 30% of SLE sufferers had antiphospholipid syndrome, but
it is now thought that primary antiphospholipid syndrome is a separate entity consisting of
tendency to thrombosis, positive antiphospholipid antibodies but the absence of clinical
features of SLE.

313. A 70-year-old male with a history of claudication at 150 yards is seen with a recent
deterioration in his claudication distance over the past month and is now complaining of
rest pain.

His past history includes hypertension, a previous myocardial infarct and


hypercholesterolaemia. He continues to smoke 20/day despite advice to refrain.

Review of his case notes reveals that when last reviewed in clinic six months previously the
femoral pulse was intact and distal pulses were present but weaker than the left leg. The
ankle brachial pressure index (ABPI) had been 0.85 and so no further imaging was
undertaken. On examination, the femoral pulse is now weak, the distal pulses are absent
and the ABPI is 0.4.

How should he be managed next?

(Please select 1 option)


Embolectomy

Exercise programme

Immediate vascular reconstruction

Reassurance and simple analgesia

Urgent vascular imaging Correct

This patient has suffered an acute deterioration on the background of chronic limb ischaemia
and his ABPI confirms that he has moved from intermittent claudication to critical ischaemia.

This is not uncommon amongst cardiopaths with claudication.

It is important to image the vessels either with duplex or an arteriogram dependent on local
expertise and then to perform urgent rather than emergency surgery.

314. These are the ophthalmoscopic appearances of a 74-year-old female who presents
with a long history of deterioration in her vision.

Her vision is 6/36 in both eyes and uncorrected with a pinhole.


What is the most likely diagnosis?

(Please select 1 option)

Glaucoma This is the correct answer

Macular degeneration Incorrect answer selected

Optic atrophy

Papilloedema

Retinitis pigmentosa

This fundus shows cupping of the optic disc which is typical of glaucoma.

Almost invariably there is an increase in pressure which is sufficient to cause damage to the
optic nerve head and causes changes in the visual field.

The mean value for intraocular pressure is 15-16 mmHg with a standard deviation of +/- 2.5.

The upper limit of normal is considered to be 21 mmHg.

315. A 54-year-old male who is a HGV driver and has a 10 year history of type 2 diabetes
is seen on annual review.

His glycaemic control is poor with a HBA1c of 91 mmol/mol (20-42) on maximal oral
hypoglycaemic therapy. You suggest switching to insulin but he refuses to do this as he
would lose his HGV licence. He also refuses to inform the DVLA himself.

What is the most appropriate action in this case?

(Please select 1 option)

Continue to review patient in clinic and accept that he continues to drive This is the correct answer

Discharge him from clinic as there is nothing more that you can do

Inform the DVLA even if the patient withholds his consent Incorrect answer selected

Inform his employer that he must stop driving and suggest administrative work

Tell his next of kin that they should inform the DVLA that he is no longer fit to drive

In this particular case, the patient has poor glycaemic control, but otherwise has no features
whatsoever that preclude him from driving such as retinopathy, neuropathy or
hypoglycaemic episodes. You cannot therefore force this patient to switch to insulin and
neither can you stop him driving.

He will continue to need a regular medical every three years for his continued HGV licence.
316. A 41-year-old female is brought into the Emergency department after taking an
uncertain quantity of paracetamol two hours previously and trying to hang herself.

She becomes agitated and insists that she wants to go home immediately. You judge that
she is at high risk of suicide.

Which of the following is the most appropriate course of action for this patient?

(Please select 1 option)

Ask her to sign a 'discharge against medical advice' form and let her go

Call the duty psychiatrist, but let the patient go if she insists and the duty psychiatrist does not arrive in
time to see her

Call the hospital security services, restrain her and sedate her Incorrect answer selected

Detain her under section 5(2) of the Mental Health Act

Detain the patient under common law, seeking the help of hospital security services This is the correct
answer

This patient has taken an unknown quantity of paracetamol tablets, and continues to have
suicidal ideation. In situations such as these you need to assess whether the patient has
capacity.

Taking a paracetamol overdose in itself does not mean the patient has no capacity, but often
capacity is lacking. The chance of suffering severe consequences if a paracetamol overdose
goes untreated is high. Often, therefore these patients do not possess the level of capacity
required for a decision of this importance. Family should be involved if possible.

In cases such as this the patient, if found to lack capacity, should be held under common law
until the appropriate time to take a blood sample (four hours after the overdose).

Psychiatric team advice can be sought, but they need the patient to be 'medically fit' to be
able to do a full assessment. In general this means they must have completed all treatment
necessary (for example,. NAC infusion) prior to mental health assessment.

Allowing self-discharge in this situation would be inappropriate.

Sedation is inappropriate in this situation.

Section 5(2) of the Mental Health Act is used for patients who are already admitted to the
hospital who have a mental illness, to allow compulsory detention for up to 72 hours. Formal
assessment under the Mental Health Act should be undertaken as soon as possible. It is not
used in the Emergency department.
317. A middle-aged woman complains of weight loss.

On examination she is tremulous, has palmar erythema and is tachycardic. She has some
eyelid swelling and asymmetric nodular areas over the pretibial area of her legs.

What single investigation will be most likely to provide the diagnosis?

(Please select 1 option)

Auto antibody profile

Scintigraphic imaging

Serum biochemistry

Skin biopsy from one of the nodular areas

Thyroid function studies Correct

The patient is likely to have Graves' disease (hyperthyroidism and diffuse goitre,
ophthalmopathy and dermopathy).

This is associated with pretibial myxoedema (bilateral, asymmetrical firm non pitting nodules
and plaques on the anterior aspect of the shin); in severe cases this can produce signifiant
deformity.

Graves' may also be associated with acropachy (proliferation of the periosteum and
clubbing). Other signs include Plummer's nails (onycholysis).

Eye signs include

Proptosis
Lid retraction
Periorbital swelling
Exophthalmic ophthalmoplegia
Strabismus
Diplopia.

318. Which of the following management options is not appropriate with a history of acute
intermittent porphyria?

(Please select 1 option)

Avoid alcohol

Carbohydrate loading during acute episodes

Discontinues barbiturates
Low dose chlorpromazine for pain

Oral contraceptive to regulate menses Correct

Barbiturates, sulfonamides, griseofulvin, chloroquine and certain steroids may precipate or


aggravate the attacks of acute porphyria, hence avoid specific treatment as symptom
control.

319. Which of the following is not a feature of the Holmes-Adie syndrome?

(Please select 1 option)

Both pupils may be affected

Global arreflexia is a recognised association Incorrect answer selected

Ptosis is rare

The affected pupil is always larger than the unaffected This is the correct answer

There is denervation supersensitivity of the pupil

Holmes-Adie syndrome consists of impaired pupillary constriction to light associated with


loss of deep tendon reflexes.

It is caused by idiopathic degeneration of parasympathetic neurones in the ciliary ganglion


and therefore exhibits denervation supersensitivity; the affected pupil will constrict on
exposure to low concentration of pilocarpine (0.125%) whereas the unaffected will not.

With time, the affected pupil may become small; the other pupil may also become affected.

320. An 85-year-old woman with diabetes mellitus presented with sudden onset of wild
flinging movements of the left arm which disappeared during sleep.

What is the most likely explanation?

(Please select 1 option)

Contralateral subthalamic nucleus infarction This is the correct answer

Focal motor seizures

Hypoglycaemia

Ipsilateral caudate nucleus infarction Incorrect answer selected

Ipsilateral cerebellar infarction


This is hemiballismus, and in a patient with diabetes is likely to be due to a vascular event in
the contralateral subthalamic nucleus.

Hemiballismus is usually characterised by involuntary flinging motions of the extremeties,


which are often violent. It is continuous and random, and can involve proximal, distal or facial
muscles. It is always unilateral, but it is common for arms and legs to move together. The
movements worsens with activity, and decrease with relaxation. It results from a decrease in
activity of the subthalamic nucleus of the basal ganglia, which results in decreased
suppression of involunary movements.

In addition to strokes there are a number of other causes of hemiballismus. These include
traumatic brain activity, amyotrophic lateral sclerosis, hypergylcaemia, malignancy, vascular
malformations, tuberculomas and demyelinating plaques.

Treatment should initially start with identifying and treating the cause. When pharmacological
treatment is necessary this is usually initially with an antidopaminergic such as haloperidol or
chlorpromazine. Topiramate can be used, as can intrathecal baclofen, botulinum toxin and
tetrabenazine. Functional neurosurgery can be used for cases which have failed to respond
to other treatment.

321. A 70-year-old woman presented with metastatic breast carcinoma and painful
metastases in the ribs and spine.

She had been taking MST continus(long acting morphine tablets)120 mg twice daily in
addition to paracetamol 1 g six hourly. She was also prescribed morphine solution for
breakthrough pain.

Which one of the following doses of morphine solution would be most appropriate?

(Please select 1 option)

10 mg Incorrect answer selected

20 mg

40 mg This is the correct answer

80 mg

120 mg

As she is receiving 120 mg every 12 hours, this is equivelent to 40 mg every four hours.

The breakthrough dose for oral morphine solution is calculated as the equivalent of a four
hourly dose.

322. A mild lymphocytosis of 15 × 109/l with a few smear cells is reported on a full blood
count result in a 70-year-old asymptomatic man attending clinic for an annual review.
Which of the following would be the most essential investigation to establish a diagnosis of
chronic lymphocytic leukaemia (CLL)?

(Please select 1 option)

CT scan of chest abdomen and pelvis

Lactic dehydrogenase (LDH) levels

Peripheral blood flow cytometry This is the correct answer

Presence of palpable cervical lymphadenopathy Incorrect answer selected

Presence of smear cells on the blood film

Flow cytometry showing a specific pattern of monoclonal B cell proliferation (CD19/5


coexpressing, CD23 positive, light chain restricted B cell population) is diagnostic of CLL.

CT scan and LDH are investigations needed to complete staging but not essential for
diagnosis.

Smear cells are reported in other lymphoproliferative as well as benign lymphocytosis and
merely indicate fragile lymphocytes which are artefactually smeared on the glass slide.

Cervical lymphadenopathy may be seen in CLL but can also be seen in any other cause of
lymphadenopathy (for example, viral infections, adenopathy secondary to local dental
infection).

323. A regular donor reports yellow discoloration of his eyes and fevers five days after a
blood donation.

What would be the next most appropriate course of action for the blood bank medical
officer?

(Please select 1 option)

Platelets are safe to be released in this situation

Recall blood products from this donor and arrange for retesting of this donor Correct

Release all the blood products from this donor if initial testing is negative

Selected blood products such as red cell packs may be released as these have a small volume of
plasma

The donor needs to be struck off the donor register

Depending on the complications developed by the donor an assessment must be made on


how to manage the donor as well as the blood products from the donation.
In this case, the blood products must be recalled till further testing and clarification of the
donor illness.

Release of one or any of the blood products must be prevented.

The donor need not be struck off the register at this stage till further testing results are
available.

324. An 8-year-old boy who lives in the UK and has no history of travel abroad, presents
with a one week history of abdominal pain and jaundice.

Examination reveals that he is jaundiced, has a temperature of 38.5°C and abdominal


tenderness with hepatomegaly.

What is the most probable diagnosis?

(Please select 1 option)

Biliary atresia

Galactosaemia

Hepatic neoplasia

Hepatitis A Correct

Malaria

This boy clearly has an infection causing jaundice, as suggested by the fever and abdominal
tenderness/hepatomegaly.

The most likely cause in this case is therefore hepatitis A, an RNA virus. Hepatitis A is
transmitted faeco-orally and it may cause jaundice, fever and painful hepatomegaly.

325. An 80-year-old male receiving treatment for prostate carcinoma complains of pelvic
pain and is noted to have pelvic metastases on radiological investigation.

What is the most appropriate treatment?

(Please select 1 option)

Escalate opioid therapy

Non-steroidal anti-inflammatory drugs

Palliative radiotherapy Correct

Pamidronate IV
Start syringe driver with diamorphine

The patient with prostate carcinoma may well respond to palliative radiotherapy which
should be considered in this case. Radiotherapy is likely to shrink the cancer and so help
with pain.

Analgesics are appropriate for symptom control thereafter.

326. A 49-year-old female is admitted with 40% burns sustained in a house fire.

What is the most appropriate immediate treatment?

(Please select 1 option)

Anaesthetise and intubate

Deroof blisters

Escharotomy

Intravenous fluids Correct

Irrigate with cold water

The woman has 40% burns and in general burns greater than 15% require intravenous
fluids. Burns patients are prone to infection and dehydration.

Burns management is dictated by the extent and degree of burns. The extent of burns in an
adult is calculated by the 'rule of nines'. The degree of the burn is defined by its depth: partial
thickness or full thickness.

Inhalation injury greatly increases the mortality of burn patients. Indicators of such injury are:

burns sustained in a closed space


facial or oropharyngeal burns
singed nasal hair, and
carbonaceous sputum.

Such patients may benefit from early prophylactic intubation and ventilation.

327. A 70-year-old man presents with poor stream and nocturia.

On examination, he has a lemon tinge to his skin, ascites, a palpable bladder and enlarged
prostate gland. His blood pressure is 170/95 mmHg.

What is the single most likely diagnosis?

(Please select 1 option)


Acute liver failure

Carcinoma of the bladder

Carcinoma of the prostate Incorrect answer selected

Chronic kidney disease This is the correct answer

Nephrotic syndrome

This man has an enlarged prostate resulting in an obstructive uropathy.

Chronic renal failure is the substantial, irreversible, and usually longstanding loss of renal
function. Obstructive uropathy due to prostatic hypertrophy is a recognised cause.

Symptoms include:

fatigue
weakness
dyspnoea
pleuritic pain
ankle swelling
restless legs
anorexia
vomiting
pruritus
decreased concentration
bone pain, and
impotence.

Pallor, yellow skin pigmentation, brown nails, purpura, bruising, excoriation, peripheral
odema and high blood pressure are some of the recognised signs.

328. Regarding dysphagia, which disease is characterised by displacement of part of the


stomach into the thorax?

(Please select 1 option)

Achalasia

Barrett's oesophagus

Hiatus hernia Correct

Plummer-Vinson syndrome

Scleroderma
The proximal stomach herniates through the diaphragm into the thorax. Eighty percent are
the sliding variety where the gastro-oesophageal junction slides up into the chest.

The other variety making 20% is rolling hiatus hernia, where the gastro-oesophageal junction
remains in the abdomen but a bulge of stomach herniates up into the chest alongside the
oesophagus.

329.

From the given list of options select the most appropriate diagnosis to account for this
appearance on the legs of a 50-year-old patient with glycosuria.

(Please select 1 option)

Chronic fungal infection

Erythema marginatum

Erythema nodosum

Granuloma annulare Incorrect answer selected

Necrobiosis This is the correct answer

Necrobiosis is strongly associated with diabetes.

The shin is the usual site.

Ulceration occurs in around 25% of lesions and the lesions are either partially or completely
anaesthetic.

330. Which one of the following require urgent referral for upper endoscopy?

(Please select 1 option)


A 35-year-old male who has a history of waterbrash and dyspepsia which has responded to a course of
ranitidine but since stopping has recurred
A 45-year-old male with a one month history of persistent dyspepsia

A 56-year-old male with a one month history of dyspepsia and a pulsatile central abdominal mass

A 62-year-old male with a three month history of unexplained weight loss, tenesmus and a right
abdominal mass

A 73-year-old male with a three month history of dyspepsia which has failed to respond to a course of
proton pump inhibitors Correct

Criteria for referral for urgent endoscopy include

Dysphagia (at any age)


Dyspepsia at any age combined with any one of weight loss, anaemia or vomiting
Dyspepsia in a patient aged 55 or above with onset of dyspepsia within one year and
persistent symptoms
Dyspepsia with one of Barrett's oesophagus, family history of upper gastrointestinal (GI)
carcinoma, pernicious anaemia or upper GI surgery more than 20 years ago
Jaundice
Abdominal mass.

With regard to the presented cases, the 56-year-old man has dyspepsia with what seems to
be an aortic aneurysm. This requires an ultrasound and vascular opinion.

In the case of unexplained weight loss, tenesmus and upper right mass the problem is likely
to be a colonic carcinoma.

331. What does the inferior mesenteric artery supply?

(Please select 1 option)

From the duodenum to the first two thirds of the transverse colon

From the oesophagus to the duodenum, also the liver and stomach

From the splenic flexure to the first third of the rectum Correct

The greater curvature of the stomach

The last two thirds of the rectum

The coeliac axis supplies the liver and stomach and from the oesophagus to the first half of
the duodenum.

The second half of the duodenum to the first two thirds of the transverse colon is supplied by
the superior mesenteric artery.

The inferior mesenteric supplies the last third of the transverse colon (approximately from
the splenic flexure) to the first third of the rectum.
The last two thirds of the rectum are supplied by the middle rectal artery.

The greater curvature of the stomach is supplied by branches of the splenic artery, which
itself comes from the coeliac axis.

332. A 6-month-old infant is admitted with vomiting.

His parents report that he 'possets' frequently, and may bring up the whole bottle up to 'two
hours' after a feed. His full blood count reveals a microcytic anaemia.

What is the most likely diagnosis?

(Please select 1 option)

Alpha1-antitrypsin deficiency

Cystic fibrosis

Duodenal atresia

Hiatus hernia Correct

Pyloric stenosis

This child's frequent vomiting and microcytic anaemia suggest a hiatus hernia with reflux.

Cystic fibrosis would be associated with chest infections and iron deficiency but not the
vomiting.

333. A 65-year-old man presents as an emergency with tearing back pain believed by the
general practitioner to be renal colic. This is his first episode of such pain.

Past medical history includes hypertension and hypercholesterolaemia and he is a current


smoker of 30 cigarettes a day. Dipstick testing of his urine is normal.

On assessment in the emergency unit he has a pulse of 82 beats per minute and a blood
pressure of 138/90 mmHg and, as there is no house officer available, he is provided with
analgesia and is sent to the urology ward for clerking. Before he is assessed he becomes
acutely unwell with a blood pressure of 106/68 mmHg and a pulse of 120 beats per minute.

What is the likely diagnosis?

(Please select 1 option)

Acute pancreatitis

Perforated duodenal ulcer


Pyelonephritis

Renal colic

Ruptured aortic aneurysm Correct

This is a classic scenario and emphasises the importance of obtaining an accurate history.
The presence of an aneurysm must be excluded in any patient who presents with back pain.

In this case, there is a strong cardiovascular history which, along with male gender, is typical
of that of a patient with an aortic aneurysm. Furthermore, the nature of the pain and absence
of haematuria tends to count against renal colic.

The other classic mimic for aortic aneurysm is lower abdominal pain radiating to the testicle.

334. A 20-year-old man was admitted as an emergency with severe pain in the right iliac
fossa and inability to weight bear on the right leg. He had seen his GP a month
previously with right iliac fossa pain and had been told it was probably a viral illness.

His past history was unremarkable. On examination his temperature was 38.2°C. There was
marked tenderness and the sensation of a deep mass in the right iliac fossa. Straight leg
lifting was limited and rotational movements of the hip were also painful.

Investigations reveal:

White cell count 17.2 ×109/L (4-11)

C reactive protein 154 mg/L (<10)

CT scan shows:
What is the diagnosis?

(Please select 1 option)

Acute appendicitis

Carcinoma caecum

Crohn's disease

Polycystic kidney disease

Psoas abscess Correct

The CT scan clearly demonstrates a large right-sided psoas abscess.

This accounts for the pain and neurological symptoms experienced by the patient as the
nerve plexus lies on the surface of psoas. It may well be that the abscess originated
following an episode of appendicitis a month previously at his initial presentation.

Psoas abscesses can occur in Crohn's disease, in TB and following perforation of caecal
tumours but the scenario presented is not in keeping with these diagnoses.

They sometimes track beneath the inguinal ligament and present as groin swellings.
335. A 14-year-old boy is admitted to the Emergency Department, hypotensive after an
injury sustained whilst playing rugby. The boy was tackled from the side and experienced
severe pain over the left lower rib cage.

On examination his pulse is 140 bpm and blood pressure 80/40 mmHg. There was bruising
over the left flank and tenderness on palpation.

What is the likely diagnosis?

(Please select 1 option)

Aortic dissection

Liver trauma

Musculoskeletal injury only

Renal haematoma

Splenic rupture Correct

High impact injuries to the left flank may cause damage the spleen or kidney as well as the
ribs and soft tissue. If there are fractured ribs and the patient is hypotensive suspect rupture
of the spleen.

If the patient responds to fluid resuscitation, a CT scan be arranged to confirm the diagnosis.
The injury can be graded and together with physical parameters a decision can be made to
conservatively manage or operate. Observation should be undertaken in an HDU setting
initially so that any deterioration in haemodynamic parameters can be acetd upon. CT is also
useful in trauma to exclude other injuries, such as hollow visceral injuries that would
mandate a laparotomy.

Renal trauma rarely renders the patient hypotensive unless other organs are also injured,
and more force would be required for aortic dissection.

336. A 3-year-old girl is brought in by ambulance from a house fire. She has extensive
areas of peeling skin over the trunk and face, with blackening around the mouth and
nostrils.

She had a full term normal delivery with no neonatal problems. Her immunisations are up to
date. There is no family or social history of note.
On examination the temperature is 36.7°C, respiratory rate 25/min, pulse 130/min. Capillary
refill time of three seconds. 40% of second and third degree burns over the trunk and face.

What is the most important part of management?

(Please select 1 option)

Analgesia

Bag and mask ventilation

Face mask oxygen

Intubation Correct

IV fluid bolus

The picture is one of severe burns and smoke inhalation.

Shock can occur in the first few hours from loss of large amounts of plasma from denuded
skin.

The airway should be secured by immediate intubation before it becomes too oedematous
and occludes completely. One hundred percent O2 should be given, as pulmonary oedema
can be anticipated.

Two large-bore cannulae should be inserted and 20 ml/kg of N. saline given.

Maintenance and continuing losses should be calculated from the time of the fire.

337. A 2-year-old male child is brought to the Emergency department by his mother.

He has been unable to bear weight since a fall from a chair this morning. He is tender over
the mid-tibia but there is no obvious limb deformity.

What is the x ray most likely to reveal?

(Please select 1 option)

Angulated fracture

Compound fracture

Epiphyseal fracture

No fracture likely

Spiral fracture Correct

This child has most likely a spiral fracture as he is tender in the mid tibia, and there is no
obvious deformity.
The injury is commonly caused by a twisting movement along the long axis of the bone
which would have been caused by the fall.

338. A 3-year-old boy is brought to the Emergency department by his father.

The boy is complaining of a painful elbow after being lifted up by his outstretched arms
earlier that day. He is refusing to use his arm.

What is the most likely diagnosis?

(Please select 1 option)

Fracture of the clavicle

Fracture of the lateral epicondyle

Fracture of the medial epicondyle

Fracture of the shaft of the radius/ulnar

Subluxation of the radial head Correct

Subluxation of the radial head or "pulled elbow" is a common injury in young children. It
results from a direct pull on the arm that causes the radial head to be pulled from its
ligament.

The child usually complains of a painful elbow and there is limited supination and extension.
It is common for children to refuse to use their arm.

If the history is characteristic the diagnosis can be made clinically without the need for x ray.

Treatment includes analgesia and manipulation by supination of the elbow whilst in 90° of
flexion.

339. In a patient with prognathism, mal-occlusion of teeth, greasy skin, increasing hat
size, which of the following is not associated with the underlying disease?

(Please select 1 option)

Elevated blood glucose

Increased risk of bowel cancer

Renal calculi if the condition is untreated

Suppression of GH in the GTT Correct

Treatment with cabergoline


Acromegaly is associated with renal stones.

Biochemical features include impaired GT, increased PO4, TG, Ca in urine, prolactin;
decreased T4, low TBG, hypopituitarism.

The aim of treatment is for symptomatic control GH<5mU.

Transphenoidal hypophysectomy is the commonest surgical approach.

Drugs used for treatment include bromocriptine, cabergoline and octreotide.

340. A 42-year-old woman with a long history of disabling rheumatoid arthritis presents to
her GP with severe bruising of her upper and lower limbs.

She is noted to have central adiposity, and her blood pressure is 160/90 mmHg. Her platelet
count and clotting screen are normal.

Which one of her medications is the most likely cause for her symptoms?

(Please select 1 option)

Anti-TNF therapy

Celecoxib

Diclofenac

Gold

Prednisolone Correct

This patient has signs and symptoms consistent with steroid-induced (iatrogenic) Cushing's
disease.

This would cause thin skin and capillary fragility that might result in bruising, often from trivial
injuries.

341. A 70-year-old man complains of progressive weakness over the past two months.

He has had to stop work as a train driver and has noticed that he is dropping objects from
his hands. On direct questioning, he reports some difficulty with swallowing, and feels that
he has lost some weight recently.

There is no relevant past medical or family history, and he takes paracetamol for intermittent
back pain that he had for a number of years. He smokes 10 cigarettes a day, and drinks
lager on occasions.
On examination, he looks well, although rather morose. There is some erythema over the
backs of his hands. Cranial nerve and cognitive examination is entirely normal. In the upper
limbs there is reduced power of the finger flexors bilaterally with some of wrist flexion and
extension. Reflexes are normal, sensory examination is normal. In the lower limbs, there is
some weakness of hip flexion, but otherwise no neurological deficit is found. Sphincter
function is normal. Systems examination is normal.

Nerve conduction studies are performed and reported as normal in the upper and lower
limbs. EMG examination, however, is reported as showing a "myopathic" pattern.

Which test would provide a diagnosis?

(Please select 1 option)

Anti-Jo1 antibodies

Chest x ray

ESR

MRI scanning of the brain and cord

Muscle biopsy Correct

This pattern is typical of inclusion body myositis (IBM), in the age, sex and muscle groups
affected.

This does also, sometimes, impact on swallowing.

The EMG shows that a myopathic process is going on.

In terms of the differential, polymyositis and dermatomyositis would be considered, although


the process is painless with no typical heliotrope rash.

Serum creatine kinase (CK) tends to be higher in polymyositis than IBM.

342. A 1-year-old child has recurrent episodes of diarrhoea and bronchopneumonia.

His full blood count shows a normal haemoglobin concentration, normal white cell count but
low lymphocyte count. His total immunoglobulin concentration is also low.

What is the likely diagnosis?

(Please select 1 option)

AIDS

Alpha-1 antitrypsin deficiency


Cystic fibrosis

Hypogammaglobulinaemia

Severe combined immunodeficiency syndrome Correct

The combination of low lymphocyte count and low immunoglobulin concentration suggests
SCID.

The defining feature of SCID, commonly known as "bubble boy" disease, is a defect in the
specialised white blood cells (B- and T-lymphocytes) that defend us from infection by
viruses, bacteria and fungi.

Without a functional immune system, SCID patients are susceptible to recurrent infections
such as pneumonia, meningitis and chicken pox, and can die before the first year of life.

Though invasive, new treatments such as bone marrow and stem-cell transplantation save
as many as 80% of SCID patients.

It appears to be X linked in as many as 50% of cases.

343. A 6-year-old girl presents with cervical lymphadenopathy of three months standing,
which has increased progressively in size. She has been off colour with a chronic cough
and hoarse voice.

She had a full term normal delivery with no neonatal problems. Her immunisations are up to
date. There is no family or social history of note.

On examination the temperature is 36.8°C, respiratory rate 20/min and pulse 85/min. She
looks unwell and thin and is in the third centile for weight, 25% for height. Massive rubbery
enlargement of right cervical glands, which are non-tender. No overlying erythema.
Otherwise well.

What is the most likely diagnosis?

(Please select 1 option)

HIV

Infectious mononucleosis

Kawasaki disease

Lymphoma This is the correct answer

Mycobacterial adenitis Incorrect answer selected


The progressive enlargement of nodes over several months without associated inflammation
suggests malignant infiltration. The abnormal voice suggests involvement of the recurrent
laryngeal nerve.

Assessment will require chest x ray and magnetic resonance imaging (MRI) or computed
tomography (CT) of neck and thorax, and lymph node biopsy.

Prognosis will depend on staging and histology (including molecular tests).

344. A 7-year-old girl with known epilepsy presents with a history of prolonged convulsion.
She has had a cold for the past 2 days with mild fever. This evening she began fitting 35
minutes ago and this has continued unabated.

She was a full term normal delivery with no neonatal complications. Immunisations up to
date. There is a family history of epilepsy in a first cousin.

On examination she has a temperature of 38.6°C, irregular breathing, and pulse of 120/min.
Oxygen saturations are 87% despite facemask Oxygen and oropharyngeal airway. Her right
ear drum is red and bulging.

What is the first-line treatment of choice?

(Please select 1 option)

Diazepam IV

Diazepam PR Incorrect answer selected

Lorazepam IV This is the correct answer

Paraldehyde PR

Phenytoin IV

The history is of prolonged continuous generalised seizure in a known epileptic. This fulfils
the definition of Status Epilepticus, which is >30 min continuous convulsion or sequence of
convulsions without return to consciousness between them.

A blood glucose should always be checked (hypoglycaemia is easily treated, and prolonged
hypoglycaemia may damage the brain).

IV lorazepam is the treatment of choice and should be given if IV access is available,


repeated after 10 mins if there is no response. Intrabuccal midazolam or PR diazepam may
be used if no IV access can be obtained. Phenytoin and thiopentone infusion are second
and third line therapies.
345. A 3-year-old boy presents with an itchy rash.

This has been present on his wrists and hands for three weeks and disturbs his sleep. He
was born at 37+2/40 weighing 3.56 kg and there were no neonatal problems. He is fully
immunised and on no medication. His 5-year-old sister has a similar rash.

On examination his temperature is 36.4°C, RR 20/min and HR 90/min. He has red raised
papules around his wrist and small linear skin breaks in the finger webs. Otherwise there are
no abnormalities to find.

What is the most likely diagnosis?

(Please select 1 option)

Ammoniacal dermatitis

Atopic dermatitis

Impetigo

Napkin candidiasis

Scabies Correct

The history of itchy papular rash worst at night, affected sibling, and finding of burrows all
point to scabies as the diagnosis.

As the female mites burrow and bury their eggs and faeces, toxic and antigenic substances
cause the condition.

346. A 23-year-old man is about to undergo appendicectomy.

The consultant rings the house officer and asks her to prescribe prophylactic antibiotics.

What is the most appropriate prophylactic antibiotic for this patient?

(Please select 1 option)

Cefuroxime This is the correct answer

Flucloxacillin

Imipenem

Metronidazole Incorrect answer selected

Penicillin

Prophylactic antibiotics are used for gut surgery as wound infections occur in up to 60%.
These rates are substantially reduced by using prophylactic antibiotics.

The choice of cefuroxime will kill anaerobes, enterococci and coliforms which are prevalent
in the gut.

347. A 50-year-old company executive presents with fever, dry cough, chest pain and
diarrhoea after returning from his company's July convention in a Mediterranean resort.

Despite the paucity of respiratory signs on clinical examination, his chest x ray confirms a
bilateral diffuse pneumonia.

Which is the most likely causative organism of his illness?

(Please select 1 option)

Chlamydia psittaci

Klebsiella pneumoniae

Legionella pneumophila Correct

Staphylococcus aureus

Streptococcus pneumoniae

Legionnaires' disease is a rare but serious acute respiratory disease caused by the
bacteriumLegionella pneumophila.

Infection happens when people inhale aerosols that contain theLegionella bacterium.
Infection often occurs around hot water systems, air conditioning cooling towers and
whirlpool spas.

The early symptoms are flu-like and include muscle aches, tiredness, headaches, dry cough
and fever. Diarrhoea and confusion may also develop.

The symptoms frequently lead to pneumonia, and the disease is fatal in 10-15% of otherwise
healthy individuals.

The urinary antigen test for Legionella species is the most useful test being rapidly
available and accurate (70% specificity and 100% sensitivity).

It is associated with a 5-15% mortality associated with respiratory and renal failure.

Treatment is with macrolides and/or ofloxacins.


348. A 65-year-old male with a history of alcohol abuse and treated Addison's disease
presents with marked weakness. He is known to be poorly compliant with therapy.

His ECG is shown.

Which of the following is the most likely metabolic cause of these ECG changes?

(Please select 1 option)

Hypercalcaemia

Hyperkalaemia Correct

Hypokalaemia

Hyponatraemia

Uraemia

The ECG reveals tented T waves, widening of the QRS complexes and P wave
disappearance - features of hyperkalaemia.

Addison's disease is a cause of hyperkalaemia as hypocortisolism reduces renal potassium


excretion due to reduced activation of the sodium/potassium pump in the distal convoluted
tubule.

The biochemical picture is therefore one of:

Hyponatraemia
Elevated urea
Hypoglycaemia, and
Hyperkalaemia.

Hyperkalaemia initially causes membrane excitability due to partial membrane depolarisation


then, as levels of serum potassium increase, it may cause muscle weakness, arrhythmias,
and eventually cardiac arrest.

As the hyperkalaemia progresses, the ECG changes deteriorate: peaked T waves,


decreased P waves and QRS widening.

349. A 5-year-old girl with an ejection systolic murmur radiating between the scapulae and
radio femoral delay has an unusual facial appearance.

Which one of the following is a possible diagnosis?

(Please select 1 option)

Down syndrome

Edwards' syndrome Incorrect answer selected

Noonan's syndrome

Turner's syndrome This is the correct answer

Williams' syndrome

Turner's syndrome is associated with aortic coarctation.

Noonan's syndrome has a phenotype similar to Turner's, but is AD, thus meaning equal sex
distribution. In addition, Noonan's syndrome is associated with pulmonary stenosis and
hypertrophic obstructive cardiomyopathy (HOCM), not coarctation.

Williams' syndrome has associated supravalvular aortic stenosis.

Facial features of Turner's syndrome include webbed neck, high arched palate and low
posterior hair line.

350. A 45-year-old woman is referred by the GP because of altered bowel habit for 4
months. She is opening her bowels 4 times a day and has associated weight loss.

On examination, she is pale, she has axillary pigmentation there is pitting oedema to the
upper calf. Her pulse was 90 and blood pressure 95/60. Her heart sounds were normal. She
has bilateral pleural effusions. Her abdomen is distended and she has shifting dullness,
there is a 2 cm firm liver edge palpable.
Investigations reveal the following:

Hb 92 g/L (115-165)

MCV 70 fL (80-96)

WCC 4.5 ×109 (4-11)

Platelets 340 ×109 (150-400)

ESR 60 mm/hour (0-20)

PT 12.3 secs (11.5-15.5)

Glucose 7.1 mmol/L (3.0-6.0)

Na 139 mmol/L (137-144)

K 3.2 mmol/L (3.5-4.9)

Urea 3.4 mmol/L (2.5-7.5)

Creatinine 88 µmol/L (60-110)

Calcium 2.6 mmol/L (2.2-2.6)

Phosphate 1.3 mmol/L (0.8-1.4)

Bilirubin 44 µmol/L (1-22)

Alkaline Phosphatase 132 U/L (45-105)

ALT 28 U/L (5-35)

AST 33 U/L (1-31)

Albumin 21 g/L (37-49)

CRP 19 mg/L (<10)

Urine dip shows protein +++

What is the most important investigation for patient management?


(Please select 1 option)

Colonoscopy Correct

ASO

Viral serology

CT thorax

Ultrasound of the abdomen

The patient has Nephrotic Syndrome. Nephrotic syndrome is characterised by:

Heavy proteinuria >5 g daily in adults; >40 mg/h/m2 in children


Hypoalbuminaemia (<30 g/L)
Oedema
Hyperlipidaemia

Causes of Nephrotic Syndrome:

Minimal change GN-most common cause in children


Membranous glomerulonephritis (membranous nephropathy)- most common cause of
primary nephrotic syndrome in adults (40% of cases)
Focal Segmental Glomerulosclerosis
Membranoproliferative glomerulonephritis (mesangiocapillary proliferative
glomerulonephritis)-10% of all cases of nephrotic syndrome.
Other proliferative glomerulonephritides- nephrotic syndrome may be observed in a
proportion of individuals with post-streptococcal glomerulonephritis, IgA nephropathy,
and Henoch-Schönlein purpura.
Nephrotic syndrome associated with systemic diseases, e.g. Diabetic glomerulosclerosis,
Amyloidosis, and Lupus nephritis.

Causes of Membranous GN:

Neoplasia
Penicillamine, captopril, mercury, gold
Hep B, syphilis, malaria
RA, SLE, Sarcoid.

C3 nephritic factor is an autoantibody specific for alternative pathway C3 convertase


(C3bBb) This antibody stabilises C3bBb and causes consumption of C3. It is found in
mesangiocapillary GN type II and partial lipodystrophy.

In this patient colonoscopy is essential to exclude a colonic neoplasm in view of previous


weight loss and change of bowel habits.
351. A 48-year-old attends the acute medical unit with a sore throat, six weeks after
starting a new medication.

An FBC is performed which shows:

Hb 107 g/L (115-165)

WCC 1.4 ×109/L (4-11)

Platelets 151 ×109/L (150-400)

Neutrophils 0.0 -

Lymphocytes 1.4 ×109/L (1.5-4)

Which of the following drugs is the likely cause?

(Please select 1 option)

Alendronate

Calcitonin

Carbimazole Correct

Chlorpropamide

Clomifene

The woman has agranulocytosis as a complication of carbimazole therapy.

Patients prescribed this drug are always warned to seek medical attention if they develop
symptoms or signs of infection, in particular a sore throat.

Carbimazole should be stopped promptly, and the neutropenia is reversible.

352. A 30-year-old man was found to have a BP 205/120 mmHg.

Blood tests revealed:

Na 145 mmol/L (137-144)

K 3.0 mmol/L (3.5-4.9)

He has a low plasma rennin and low plasma aldosterone.

Which of the following is a differential diagnosis?


(Please select 1 option)

21-hydroxylase deficiency

Bartter's syndrome

Gitelman's syndrome

Liddle's syndrome This is the correct answer

Phaeochromocytoma Incorrect answer selected

Causes of apparent mineralocorticoid excess include:

11-beta hydroxylase deficiency (CAH)


11-beta hydroxysteroid dehydrogenase-2 deficiency
Liquorice (glycyrrhetinic acid) addiction
Carbenoxolone, and
Liddle's syndrome (auto dominant condition with increased renal tubular resorption of Na
and in most cases K loss).

353. A 62-year-old male with a history of ischaemic heart disease is admitted with chest
pain of 13 hours duration and some ST depression inferolaterally.

Which of the following would be the most appropriate treatment option for this patient?

(Please select 1 option)

IV diamorphine 10 mg Incorrect answer selected

Oral aspirin 600 mg

Oral isosobide mononitrate 30 mg

Oxygen 100%

Subcutaneous fondaparinux 2.5 mg This is the correct answer

The treatment approach in this patient with acute coronary syndrome is:

Aspirin 300 mg
Try some glyceryl trinitrate (GTN).

If this fails then:

Isosorbitdinitrate (ISDN) intravenously


Diamorphine (or morphine) 2.5 mg
4 L oxygen, and
fondaparinux - a subcutaneous anticoagulant.
354. A 22-year-old female is admitted very distressed and short of breath.

Examination reveals a respiratory rate of 35/min, a pulse of 120 beats per minute, a blood
pressure 110/70 mmHg, oxygen saturations of 90% and a peak expiratory flow rate <50%
predicted.

The emergency medical services have administered salbutamol 5 mg (twice) and face
mask oxygen.

Which of the following is the most appropriate next action in this patient?

(Please select 1 option)

Arterial blood gas analysis

Intensive care referral

Oxygen 35% Incorrect answer selected

Prednisolone 40 mg

Salbutamol 5 mg and ipratropium bromide 0.5 mg This is the correct answer

This patient fits the criteria for acute severe asthma. In such cases β2-agonists should be
administered as soon as possible, preferably nebulised driven by oxygen. Repeat doses
should be given at 15-30 minute intervals, or continuous nebulisation can be used where
there is inadequate response to bolus therapy. Nebulised ipratropium bromide should be
added for patients with acute severe or life threatening asthma, or those with a poor initial
response. Its addition produces significantly greater bronchodilation than aβ2-agonist
alone.

Oxygen should be given to maintain saturations at 94-98%. Patients with saturations


less than 92% on air should have an ABG to exclude hypercapnia. However, starting
treatment should not be delayed to do the ABG. Initially high-flow oxygen is used, and then
weaned to maintain adequate saturations. Unless you suspect COPD there is not a need
to be cautious with oxygen therapy.

Steroids reduce mortality, relapses, subsequent hospital admission and requirement for
β2-agonists1. The earlier they are given in the attack, the better the outcome. A dose of
40-50 mg should therefore be given once oxygen and nebuliser therapy has been
established. This should be continued for five days, or until recovery, and can then be
stopped abruptly unless the patient has taken long term oral corticosteroids.

Failure to respond to the above treatment steps may warrant the use of intravenous
magnesium sulphate and aminophylline. However, these should not be used without
discussion with your senior colleagues.

Intensive care is indicated for patients with severe acute or life threatening asthma who are
failing to respond to therapy. Consider it in patients with deteriorating peak flow, persisting
or worsening hypoxia, hypercapnia, acidosis, exhaustion or altered conscious state. All
patients who are transferred to an intensive care unit should be accompanied by a doctor
who can intubate if necessary.

As an aside, chest radiographs are not indicated unless you suspect pneumothorax or
consolidation, or there is life-threatening asthma, a failure to respond to treatment or a
need for ventilation.

355. An otherwise fit 80-year-old lady presents with an eight hour history of an acutely
ischaemic right arm with severe pain in the hand and fingers. She is right handed. There
was no history of pain or restricted movement prior to this current episode.

She has a history of atrial fibrillation for which she has been prescribed digoxin and takes
appropriate medications for hypertension. On examination the hand is very cold and pale but
some movement and sensation are preserved. There is a good axillary artery pulse but distal
pulses are impalpable.

What is the correct management?

(Please select 1 option)

Arrange a duplex scan

Book an outpatient appointment for vascular surgery

Commence heparin

Embolectomy This is the correct answer

Thrombolysis Incorrect answer selected

This lady has an acute limb occlusion.

Upper limb ischaemia is most commonly due to emboli followed by trauma and
atherosclerotic occlusions are rare. Whilst the hand is currently viable it will not remain so for
long.

It is likely that the embolus will have lodged at the level of the brachial artery and so surgical
exploration and embolectomy using a Fogarty catheter is the correct treatment.

Intraoperative thrombolysis may help treat distal material not removed by the catheter.

356. A 24-year-old female who has previously suffered with severe depression presents
with secondary amenorrhoea.

She is found to have a prolactin of 645mU/L (normal 50-350).

Which of the drugs which she takes may cause this?


(Please select 1 option)

Becotide

Montelukast

Omeprazole

Risperidone This is the correct answer

Sertraline Incorrect answer selected

Antipsychotic medications, in paricular the typical antipsychotics and risperidone, are known
to elevate prolactin levels. This is due to their ability to block dopamine D2 receptors. By
doing so they block dopamine's action on the pitutary. This reduces inhibition of prolactin
secretion, thereby causing hyperprolactinaemia. Symptoms of this include amenorrhoea,
galactorrhoea, infertility, loss of libido and erectile dysfunction.

Sertraline is not thought to exert dopamine antagonist effects and thus does not result in
hyperprolactinaemia.

The other medications on her list have not been commonly associated with
hyperprolactinaemia.

357. A 30-year-old male presents to the emergency department complaining of facial pain.
He has had stabbing pains on the left side of his face which lasted for one second at a
time, he describes the pain as agony.

The symptoms could sometimes be triggered by washing or shaving.

What is the most likely diagnosis?

(Please select 1 option)

Classic migraine

Common migraine

Sinusitis

Post herpetic neuralgia

Trigeminal neuralgia Correct

The fact that these attacks last for such a short period of time would make the first four
stems unlikely.

Trigeminal neuralgia consists of severely painful attacks unilaterally in one of the divisions of
the trigeminal nerve. They may be triggered by a number of factors, but contact with the
affected side of the face is the most common one.
The pain of post herpetic neuralgia and sinusitis is more chronic than intermittent.

Migraine attacks typically last for hours. Classic refererence is to the presence of a visual
aura in this context.

358. A 50-year-old woman is admitted to the surgical ward for resection of a colorectal
carcinoma. She has type 2 diabetes and her current glycaemic control is good on 5 mg
glibenclamide daily.

Which of the following is the most appropriate pre-operative management plan for this
patient's glycaemic control?

(Please select 1 option)

Continue glibenclamide at current dose

Give one bolus injection of insulin plus potassium just before surgery

Increase the dose of glibenclamide to 15 mg

Stop glibenclamide before surgery

Stop glibenclamide on the morning of surgery and commence insulin by intravenous infusion Correct

The most appropriate answer is to stop glibenclamide on the morning of surgery, using IV
insulin and potassium.

This patient is undergoing major surgery, therefore glycaemic control needs to be optimal
during this period of extreme stress.

This is best achieved by stopping the glibenclamide on the morning of surgery and starting
the patient on IV insulin and IV dextrose with K.

At this time the rate of insulin infusion can then be adjusted according to frequent blood
glucose measurements.

359. Approximately 1% of pregnant women develop clinically important red cell antibodies,
the most common being rhesus antibodies.

The women negative for D antigen develop antibodies on exposure to D positive blood (such
as fetomaternal haemorrhage, abortions and transfusions). This increases the risk of
haemolytic disease of the newborn (HDN) in subsequent pregnancies.

From the following, choose the correct statement about rhesus antibodies in pregnancy.

(Please select 1 option)


D positive women are less likely than D negative women to form antibodies to other red cell antigens
(such as Kell and Duffy)

Following delivery, the degree of fetomaternal haemorrhage should be calculated on a blood sample
from a D negative mother Correct

Maternal antibody titres do not predict haemolytic disease of newborn

Pregnant women should be checked for antibodies at 28 weeks as fetomaternal haemorrhage occurs
only after the second trimester.

The fetal Rh type is not dependent on the paternal Rh grouping

Following delivery, the degree of fetomaternal haemorrhage (FMH) should be calculated on


a blood sample from a D negative mother to adjust the dose of anti-D in the D negative
mother delivering a D positive child.

D positive women and D negative women have the same chances of developing antibodies
to other red cell antigens.

All pregnant women should have a blood group and antibody screen in their first trimester or
at presentation, whichever is earlier.

The fetal Rh type depends on the paternal and maternal Rh typing.

Maternal antibody titres correlate with the degree of haemolytic disease of the newborn
(HDN).

360. A 42-year-old woman presents to the clinic with a chronic cough.

Which of the following features would increase the suspicion that she is suffering from
asthma?

(Please select 1 option)

Associated dizziness

Chronic cough without wheeze

Symptoms corresponding with a coryzal illness

Symptoms in response to exercise Correct

Voice disturbance

The answer is symptoms in response to exercise.


Guidelines from BTS/SIGN on the diagnosis and management of asthma suggest that the
presence of wheeze, breathlessness, chest tightness or cough, particularly if symptoms are
worse at night or in the early morning, after exercise, allergy exposure or cold air, are
suggestive of the diagnosis.

A family history of atopy or asthma, personal history of atopy, widespread wheeze, low
forced expiratory volume in one second (FEV1) or peak expiratory flow rate (PEFR) also
support the diagnosis.

Dizziness, light-headedness, voice disturbance and chronic cough without wheeze do not
support a diagnosis of asthma. In addition, there is no evidence that symptoms which
correspond with a cold (coryzal illness) suggest an underlying diagnosis of asthma.

361. A 40-year-old man is admitted to the Emergency department after being involved in a
house fire. He is extremely drowsy but you notice on examining him that he seems well
perfused, with his cheeks looking almost pink.

His BP is 100/60 mmHg and his pulse is 95 and regular. Blood gas analysis reveals a CO
level of 12% and a metabolic acidosis with a pH of 7.15.

Which of the following is the most appropriate next intervention?

(Please select 1 option)

100% oxygen by mask Correct

Hyperbaric oxygen

IV mannitol

IV sodium bicarbonate

Nebulised salbutamol

The answer is 100% oxygen by mask.

Hyperbaric oxygen is recommended by some countries including the United States, although
because of a lack of randomised control evidence it is not standard practice in the United
Kingdom as there appears to be no significant improvement in outcome versus high flow
oxygen alone.

Sodium bicarbonate is not indicated, and IV mannitol is used only if there is suspicion of
cerebral oedema. Key to prognosis is removal from the source of carbon monoxide as
quickly as possible, and instigation of high flow oxygen treatment.

At a level of 12%, long term psychological disturbance or memory loss is possible.


362. A 39-year-old woman with advanced cervical cancer with pelvic spread presents to
the Emergency department with nausea and vomiting for the past 24 hours.

She is unable to keep any fluids down and has not passed flatus or faeces for the past three
days. On examination she is pyrexial 38.4, has a BP of 95/65 mmHg, and a pulse of 92. Her
abdomen is distended, and there are no discernable bowel sounds. Abdominal x ray reveals
distended large bowel.

Which of the following procedures is the most appropriate with respect to relieving her
obstruction?

(Please select 1 option)

AP resection

Ileostomy

Percutaneous gastrostomy

Small bowel resection and reanastomosis

Transverse loop colostomy Correct

The answer is transverse loop colostomy.

We are told in the history that this patient has advanced cervical carcinoma. As such any
procedure should be minimal, allowing as rapid recovery as possible so that the impact on
patient morbidity is not excessive.

A transverse loop colostomy will allow resolution of bowel obstruction and allow her to
continue eating.

Percutanous gastrostomy, ileostomy and small bowel resection are all procedures which are
considered for small bowel obstruction.

363. A 27-year-old man presents to his GP with a six month history of feeling depressed.
He also states that recently he has experienced jerky movements flitting from one part of
the body to another.

His father experienced similar symptoms aged 30 and committed suicide.

Which is the most likely diagnosis?

(Please select 1 option)

Acute intermittent porphyria Incorrect answer selected

Epilepsy
Huntington's disease This is the correct answer

Systemic lupus erythematosus

Wilson's disease

This man has Huntington's disease. He is experiencing choreiform movements. It is


autosomal dominant and his father had the condition and committed suicide.

Huntington's is one of the genetic diseases which is associated with anticipation. This means
that the trinucleotide repeat increases in size, affecting subsequent generations earlier and
with increased severity.

364. A 41-year-old pregnant lady at 18 weeks gestation attends clinic for the results of her
amniocentesis.

The results show that the alpha-fetoprotein is greater than 2.5 multiples of the median and
the presence of acetylcholinesterase has been detected.

What is the most likely diagnosis?

(Please select 1 option)

Beta-thalassaemia major

Edwards' syndrome

Open neural tube defect Correct

Tay-Sachs disease

Turner syndrome

These findings are suggestive of an open neural tube defect such as spina bifida,
encephalocele or anencephaly.

Neural tube defects may be open or closed.

Folic acid consumed during the gestation period can help reduce the risk of neural tube
defects.

365. A 27-year-old girl presents with unilateral proptosis, weight loss, palpitations and
increased appetite.

What is the most likely diagnosis?

(Please select 1 option)


Acute iritis

Pituitary tumour

Retinitis pigmentosa

Temporal arteritis

Thyroid-associated ophthalmopathy Correct

This is thyroid-associated ophthalmopathy in a patient with Graves' disease.

Note that although classically bilateral, unilateral proptosis may occur

366. A 65-year-old woman presents with tiredness, headache, blurring of vision in the left
eye and is unable to comb her hair.

What is the most likely diagnosis?

(Please select 1 option)

Acute glaucoma

Amaurosis fugax

Cataracts

Temporal arteritis Correct

Viral conjunctivitis

Temporal arteritis is seen usually in those over 55 years. It is associated with polymyalgia
rheumatica.

Difficulty in combing her hair may be due to scalp tenderness or proximal muscle weakness.

Diagnosis is supported by a greatly raised erythrocyte sedimentation rate (ESR) and C


reactive protein (CRP) and temporal artery biopsy.

367. A 2-year-old boy is admitted to hospital with vomiting, non-bloody watery diarrhoea
and is dehydrated. It emerges other children from his play group have developed a
similar illness.

What is the single most probable causative agent?

(Please select 1 option)


Coxsackie

Cytomegalovirus

Mumps

Rotavirus Correct

Rubella

Rotavirus is the most common cause of severe viral gastroenteritis worldwide. Infection is
via the faeco-oral route and often occurs in children aged between 6 months to 6 years. This
RNA virus replicates in the intestinal mucosal cells, damages transport mechanisms leading
to salt and water depletion which results in diarrhorea and vomiting.

Diagnosis is made from clinical features and culture of virus from stools and also by
polymerase chain reaction techniques.

Treatment is mainly rehydration and correction of any electrolyte imbalance.

368. A 2-year-old boy is mildly unwell. His mother has noticed vesicles in his mouth,
palms and soles of his feet.

What is the single most probable causative agent?

(Please select 1 option)

Adenovirus

Coxsackie Correct

Cytomegalovirus

Measles

Rubella

Coxsackie A16 virus is the cause of hand, foot and mouth disease characterised by fever,
sore throat and ulcerating vesicles in palms, orophaynx and on soles.

Incubation period is five to seven days and these heal without crusting. Treatment is
supportive.

369. A 34-year-old man who is on antiepileptic drugs presents with visual field defects
three months after commencing a new anticonvulsant.

Which drug is most likely responsible for this presentation?


(Please select 1 option)

Carbamazepine

Ethosuximide Incorrect answer selected

Lamotrigine

Phenytoin

Vigabatrin This is the correct answer

As many as 30-50% of patients with long term exposure to vigabatrin have developed
irreversible concentric visual field loss of varying severity that is often asymptomatic.

Regular visual field testing is advised.

370. A 55-year-old male presents with redness of both cheeks. It is severe enough to
disrupt his confidence and daily life. He seeks medical advice and is diagnosed with
acne rosacea.

What is the most appropriate treatment?

(Please select 1 option)

No treatment

Oral retinoids

Oral tetracyclines This is the correct answer

Topical benzoyl peroxide

Topical retinoic acid Incorrect answer selected

In mild acne rosacea, topical treatment with metronidazole cream may be used.

In more severe cases the most appropriate therapy is a long course of oral tetracyclines.

Mimics of rosacea include topical steroid treatment on the face and SLE.

371. What type of amputation would be most appropriate in a patient with ischaemic
gangrene of the lower leg with ulceration over the medial malleolus and spreading
infection proximally?

(Please select 1 option)


Above knee amputation

Below knee amputation Correct

Hindquarter amputation

Supracondylar amputation

Tarsal amputation

The indications for amputation are ischaemia, tumours, pain and deformity. Eighty to 90% of
amputations are performed for ischaemic gangrene secondary to peripheral vascular
disease.

Diabetics have large and small vessel disease and a ray amputation of the toes is often the
first stage of a series of amputation levels. Where pain and deformity cannot be alleviated by
medical means, amputation is the final option.

Supracondylar amputations have the advantage of a longer stump which allows the patient
to turn in bed, however the stump is usually not long enough to fit an internal knee
mechanism prosthesis and thus is unpopular. Where the patient is already immobile, it is
may be a practical option.

372. A 90-year-old lady is immobile due to severe osteoarthritis in the hips and knees.
She has developed marked arterial ulceration in the right lower limb. She requires full
nursing care.

What type of amputation would be appropriate in this case?

(Please select 1 option)

Above knee amputation

Below knee amputation Incorrect answer selected

Hindquarter amputation

Supracondylar amputation This is the correct answer

Syme's amputation

The indications for amputation are ischaemia, tumours, pain and deformity. Eighty to 90% of
amputations are performed for ischaemic gangrene secondary to peripheral vascular
disease.

Supracondylar amputations have the advantage of a longer stump which allows the patient
to turn in bed, however the stump is usually not long enough to fit an internal knee
mechanism prosthesis and thus is unpopular. Where the patient is already immobile, it is
may be a practical option.
Supracondylar amputations are transfemoral above knee amputation just above the femoral
condyles.

373. A 45-year-old man has been found to have HIV. He is promiscuous and has a
number of casual sex partners. He has vowed not change his lifestyle.

What action should be taken in this case?

(Please select 1 option)

Call police

Give general advice Correct

Implement Mental Health Act

Implement Public Health Act

Treat under common law

HIV is not a notifiable disease. Making it so may deter people from coming forward for
treatment. All one can do in this scenario is to give advice for practising safe sex and to
inform their close contacts. He should be warned that his sexual contacts may bring a
court case against him for criminal assault for knowingly spreading HIV to them.

If the patient refuses to disclose information to a close partner who is at risk, it is possible for
the doctor to tell that individual but this may be seen as a breach of duty of confidence. In
this case, it is not possible as he does not have a close regular partner.

374. A 29-year-old man is involved in a car accident and and undergoes an emergency
splenectomy. Three years later the patient is admitted to hospital acutely unwell with a
cough.

He has a fever of 39°C and a white cell count of 19 ×109/L. Within 12 hours he develops
septicaemic shock.

What is most likely causative organism?

(Please select 1 option)

Bacteroides fragilis

Klebsiella

Pneumococcus Correct

Pseudomonas aeruginosa
Staphylococcus aureus

Post-splenectomy, patients are at increased risk of infections by an encapsulated organism.

This gentleman has respiratory symptoms and signs of septicaemia as evidenced by the
temperature of 39°C. The most likely organism is Pneumococcus.

Other encapsulated organisms include Neisseria andHaemophilus.

It is generally advised that asplenic patients take penicillin V orally life long as prophylaxis
against encapsulated bacteria.

375. A 35-year-old female patient is brought to the emergency department after a bicycle
accident where she injured her abdomen on the handlebars.

Observations reveal that she is pale and sweaty with a blood pressure of 88/58 mmHg and a
pulse of 110 bpm.

What is the most likely diagnosis?

(Please select 1 option)

Bowel perforation

Pancreatitis

Ruptured appendix

Ruptured ovary

Ruptured spleen Correct

This woman has had blunt abdominal trauma.

Features of shock following this trauma would suggest a ruptured spleen. This is a surgical
emergency, the patient needs resuscitation and stabalisation before going to theatre for
possible splenectomy.

376. A 72-year-old female presents with tiredness and weakness. On examination she is
pale and has haemoglobin of 72 g/L with an MCV of 68 fL.

What nail changes may be seen in association this patient's condition?


(Please select 1 option)

Beau's lines

Clubbing

Koilonychia Correct

Pitting

Splinter haemorrhages

Koilonychia, or spoon shaped nails are typical of iron deficiency anaemia (low MCV, low Hb).
It can also occur in association with trauma and the nail patella syndrome.

Beau's nails are ridges on the nail seen in chronic ill health, for example renal failure.

Pitting in the nail is seen in psoriasis.

Clubbing has four grades and can be seen in congenital, cardiac, gastrointestinal and
respiratory conditions.

Splinter haemorrhages are seen in infective endocarditis.

377. Which of the following drugs is an inhaled glucocorticoid used as maintenance


treatment in all but very mild asthmatic individuals?

(Please select 1 option)

Fluticasone propionate Correct

Oxitropium bromide

Sodium cromoglicate

Terbutaline

Zafirlukast

Fluticasone propionate is a corticosteroid and is a powerful anti-inflammatory agent used for


the above purpose. Similar drugs include beclometasone and budesonide.

The step wise management of asthma as per the British Thoracic Society (BTS) guidelines
is summarised below:
 Step 1 - short acting bronchodilator, for example, short acting beta agonist -
terbutaline, salbutamol
 Step 2 - inhaled steroid for example, fluticasone, budesonide
 Step 3 - add on therapy, trial of long acting beta agonist, for example, salmeterol and
increase dose of inhaled steroid
 Step 4 - trial of other medication add-ons, for example, leukotriene receptor
antagonist, theophyllines
 Step 5 - oral steroids.

378. What is the most appropriate palliative treatment for a 45-year-old man with superior
vena cava (SVC) obstruction from a bronchial carcinoma?

(Please select 1 option)

Diuretics

High dose steroids

Morphine

Nebulisers

Radiotherapy Correct

Superior vena caval obstruction is commonly due to small cell lung cancer, though it is
important to gain a diagnosis through tissue biopsy and histology. The typical presentation of
such a patient includes:

facial oedema
distension of the neck veins (fixed with inspiration and expiration)
plethoric appearance
central cyanosis, and
distension of chest wall veins.

The main symptom is that of dyspnoea. These symptoms usually progress over days to
weeks.

Ideally a biopsy for histology should be taken before treatment to make a tissue diagnosis.

In the past the patient would be treated with emergency radiotherapy, however it is now a
more calculated decision and in some cases endovenous stents may be applied prior to or
rather than radiotherapy.

However, of the options listed here radiotherapy remains the mainstay of treatment for a
patient presenting with SVC.
379. A 14-year-old boy presents with drowsiness and generalised headache. He is
recovering from a bilateral parotitis. His CT scan is normal.

What is the single most definitive investigation in this case?

(Please select 1 option)

Electroencephalogram

Fundoscopy

Lumbar puncture Correct

Magnetic resonance imaging

Mental state examination

This patient probably has viral meningitis probably caused by the mumps virus, which is a
cause of bilateral parotitis.

Microscopy and culture of the CSF would show a CSF lymphocytosis

380. This result is obtained on a 37-year-old insulin-dependent male with diabetes who
has suffered from diabetes for 15 years.

From the given list select the most appropriate response to explain this result.

(Please select 1 option)

A complication of diuretic therapy

Contraindication to ACE inhibitors

End stage renal disease

Microalbuminuria Correct

Proteinuria
The result shows a microalbuminuria screen with an albumin concentration of 93 mg/l.
Concentrations between 21-200 mg/l indicate microalbuminuria. Screening for
microalbuminuria varies from centre to centre but the most commonly used technique
involves analysis of three consecutive early morning samples, as an isolated sample is
insufficient to confirm microalbuminuria.

Using a 24 hour urine collection, microalbuminuria is defined as a urine albumin excretion of


30-300 mg per day. Microalbuminuria is a marker of early nephropathy.

The major importance of microalbuminuria is its association with increased vascular risk and
as a marker of generalised systemic vascular endothelial dysfunction. Patients with
microalbuminuria should thus have their vascular risk factor profile maximally treated.

381. This 52-year-old male presents to the practice with headaches.

He smokes 20 cigarettes per day and takes atenolol 25 mg/d for hypertension.

On examination his blood pressure is 165/95 mmHg and his pulse is 78 beats per minute.
His ECG is illustrated.

What does his ECG show?

(Please select 1 option)

Atrial ectopics

Bradycardia

Heart block
Left ventricular hypertrophy This is the correct answer

Myocardial infarction Incorrect answer selected

The ECG shows the patient to have a rate of 66 bpm with a normal axis. There is left
ventricular hypertrophy (LVH) as demonstrated by tall R waves (upstroke) in the chest
leads.There are many criteria for diagnosing LVH mostly based on the size of the QRS
deflections.LVH is a result of increased left ventricular pressures and signifies end-organ
damage.

It is an independent cardiovascular risk factor and causes include hypertension, coarctation


of the aorta and aortic stenosis.

Note also that ST depression can occur in the presence of LVH without any underlying
coronary artery disease - the so called "strain" pattern.

382. A 5-year-old child has a height which is recorded as being below the third centile.

Which of the following is the most likely aetiology of his short stature?

(Please select 1 option)

Familial short stature Correct

Klinefelter's syndrome

Maternal deprivation

New onset diabetes mellitus

Undiagnosed congenital heart disease

The commonest cause of short stature is familial short stature.

Maternal deprivation and chronic illness such as congenital heart disease can also cause
short stature, but are less frequent causes.

Klinefelter's syndrome is associated with tall stature.

Poorly controlled chronic diabetes leads to malnutrition, delayed growth and puberty.

383. A 17-year-old male who suffers from schizophrenia has had several admissions in
the last five years.

He has been poorly compliant with his medications, but he has excellent therapeutic
responses when he does take them regularly.

Which of the following is the next suitable management step required?


(Please select 1 option)

Admit him to a secure unit

Depot injections of antipsychotic medications Correct

Electroconvulsive therapy (ECT)

Prescribe sedatives

Vocational rehabilitation

The use of antipsychotic medications, also known as neuroleptic medication or major


tranquilisers, is the mainstay of treatment for schizophrenia.

These medications have repeatedly been shown to diminish the positive symptoms of
schizophrenia and prevent relapses. Depot preparations are occasionally required to
improve compliance.

ECT is also used for the treatment of schizophrenia, but its effect is short lived and less
effective when compared with antipsychotic medication.

384. Which of the following features is characteristic of early Alzheimer's disease?

(Please select 1 option)

Ataxic gait

Impaired short term memory Correct

Myoclonic jerks

Urinary incontinence

Visual hallucinations

Alzheimer's disease is characterised early in the disease by short term memory loss.

The other features listed here would suggest an alternative diagnosis such as:

Normal pressure hydrocephalus (gait ataxia and urinary incontinence)


Creutzfeldt-Jakob disease (myoclonic jerks)
Delirium or Lewy body dementia (visual hallucinations).
385. What is the function of cholecystokinin?

(Please select 1 option)

Stimulates gallbladder contraction Correct

Increases gastric emptying

Increases stomach acid production

Reduces stomach acid production

Suppresses appetite

Cholecystokinin neutralises the acidic, partially digested, food by inducing the gallbladder to
contract and the pancreas to release alkaline pancreatic fluid.

Gastrin increases stomach acid and increases the rate of gastric emptying.

Secretin reduces stomach acid production.

Leptin acts to suppress appetite.

386. A 13-year-old boy who has undergone a previous tonsillectomy comes to the surgery
complaining of increasing deafness.

He has had problems with snoring at night and listening to the teacher at school for the past
few months. According to his mother he has suffered significant nose bleeds and she has
noticed that he seems to have difficulty breathing when he snores at night.

A hearing test has revealed significant bilateral conductive deafness and he has very large
adenoids on examination, with bilateral glue ear.

Which of the following is the treatment of choice?

(Please select 1 option)

Adenoidectomy

Adenoidectomy and bilateral grommet insertion Correct

Bilateral grommet insertion

Bilateral grommet insertion and long acting antibiotic therapy

Long term oral antibiotic therapy

This patient has significant symptoms of obstruction related to large adenoids, and glue ear.
Adenoidectomy alone and bilateral grommet insertion alone have been shown to be less
effective than the combination of adenoidectomy and grommet insertion.

Long term antibiotic therapy is not effective in the presence of continued obstruction, and is
therefore inappropriate as an option here.

387. A 35-year-old female presents with menorrhagia that has not responded to treatment
with non-steroidal anti-inflammatory drugs.

One year ago she underwent sterilisation.

Which of the following would be the most appropriate treatment for her?

(Please select 1 option)

Depo-Provera

Dianette

Medroxyprogesterone acetate

Mefenamic acid

Tranexamic acid Correct

This woman does not need any contraceptive agent, and as NSAIDs have failed, tranexamic
acid may be effective.

There is evidence indicating that this agent is useful in reducing menstrual losses.

KEY LEARNING POINTS

Cardiology

nstemi management

Surgery

Surgical exploration and embolectomy using a Fogarty catheter is the correct


treatment of acute arterial limb ischaemia

Neurology

Huntington's disease is an autosomal dominant genetic disease.


Pharmacology

Vigabatrin requires regular visual field assessment due to risk of developing


visual field defects.

Dermatology

Long term treatment for severe rosacea is a prolonged course of oral


tetracyclines.

388. Which statement regarding fast acetylator status is most true?

(Please select 1 option)

80% of Caucasians are fast acetylators

Acute hepatocellular necrosis with isoniazid is more common in slow acetylators as compared to fast
acetylators

Fast acetylators are more likely to develop antinuclear antibodies on hydralazine

Has an autosomal dominant inheritance Correct

Risk of phenytoin toxicity in a patient concurrently prescribed isoniazid is greater

Approximately 40% of Caucasians are fast acetylators (88% would be true for the
Japanese).

In any population there is a bi-modal distribution of fast and slow acetylation. The difference
is in the amount (or activity) of the enzyme N-acetyltransferase.

Acute hepatocellular toxicity is more common with isoniazid in fast acetylators through
increased formation of an hepatotoxic metabolite. There is also a greater risk of failure of
isoniazid therapy due to inadequate plasma levels; equally higher doses of dapsone may be
needed to treat dermatitis herpetiformis and leprosy.

Other side effects are more common with slow acetylators due to higher plasma levels and
include

Drug-induced systemic lupus erythematosus (SLE) (hydralazine, isoniazid, procainamide,


sulphonamides),
Peripheral neuropathy (isoniazid: treated with addition of pyridoxine [vit. B6]])
Haemolysis (dapsone).
389. An 80-year-old man presents with a short history of increasing confusion.

Preceding this, he fell three weeks ago in the bathroom. In the afternoon he was examined
by his GP and he was alert with a normal physical examination. The patient has a history of
hypertension for which he takes bendroflumethiazide.

Three weeks later the patient was admitted because the dazed state had returned. He is
afebrile, has a pulse of 80 per minute regular and blood pressure of 152/86 mmHg. He has a
GCS of 15, but his response to questions is slightly slowed, he is disoriented in time and
there is some deficit in recent memory.

The patient moves slowly, but muscle strength is preserved. Neurologic examination shows
slight hyperactivity of the tendon reflexes on the right. Plantar responses are unclear
because of bilateral withdrawal.

Which of the following would be the most appropriate next investigation for this man?

(Please select 1 option)

24-hour ambulatory ECG

Computed tomograms of the head Correct

Dopplers of the carotid arteries

Electromyography and nerve conduction testing

Serum alcohol concentration

The history of previous fall and subsequent development of confusion and neurological
features points to a possible diagnosis of chronic subdural haematoma.

This is best investigated with CT, which is the investigation of choice. A skull x ray may
reveal a fracture.

390. A 72-year-old man with type 2 diabetes is admitted from the emergency department
following a collapse. His speech is mildly slurred.

He takes atenolol and ramipril for hypertension and smokes 10 cigarettes a day. On
examination there is mild weakness on the left with no sensory deficit.
What is the most likely diagnosis?

(Please select 1 option)

Left internal capsule haemorrhage

Left internal capsule infarct

Right internal capsule haemorrhage

Right internal capsule infarct Correct

Right pontine haemorrhage

This man has had an acute event that has produced a left-sided weakness without sensory
or speech disturbance; this implies a localised process.

In view of his hypertension, a small (lacunar) infarct is the most likely process, and the
compatible site would be the right internal capsule.

The corticostriatal tracts pass through the contralateral internal capsule, and this site,
therefore, tends to produce purely motor deficits. A bleed would be very unusual in
producing a deficit this focal.

Diffusion weighted MRI imaging is the most important diagnostic tool, acutely.

Management involves appropriate control of risk factors and aspirin.

391. A 23-year-old student has a night out to celebrate passing his exams. He falls asleep
at a friend's house, in the kitchen. When he awakes, he finds that lifting his left hand has
become difficult and he is unable to straighten his fingers.

Which other feature would be unlikely for his probable diagnosis:

(Please select 1 option)

Loss of sensation over the dorsum of the first interosseous space

Weakness of triceps

Extension of the fingers at the metacarpophalangeal joints with passive wrist extension This is the
correct answer

Weakness of brachioradialis Incorrect answer selected

Extension of the fingers at the interphalangeal joints with passive wrist extension

Lesions of the radial nerve may be caused by prolonged compression at the upper arm, in
trauma, or if the head is against the arm while asleep (Saturday night palsy).
Falling asleep with the arm over a chair or prolonged crutch use may damage the nerve in
the axilla.

The radial nerve (C6-8,T1) supplies the extensors of the wrist, elbow and fingers. Passive
extension at the wrist allows extension of the interphalangeal joints due to the action of the
small muscles of the hand, which are supplied by the ulnar nerve.

The only area of sensory distribution of the radial nerve is over the first dorsal interosseous
space.

392. Which of the following is not a feature of acromegaly?

(Please select 1 option)

Hypotension Correct

Glycosuria

Headache

Skin tags

Sweating

Features of disease activity are

Headache
Increased ring size
Ill fitting dentures
Sweating
Skin tags
Glycosuria
Hypertension
Further visual field loss.

Causes of death are

Cardiovascular
Tumour expansion-mass effect
Haemorrhage
Hypertension
Degenerative vascular disease.

393. Which of the following would suggest a diagnosis of subarachnoid haemorrhage


rather than bacterial meningitis in an 18-year-old female with a history of headache and
photophobia?
(Please select 1 option)

A blood neutrophil leucocytosis

A family history of polycystic renal disease Correct

A fluctuating conscious level

A history of diabetes mellitus

A history of opiate abuse

Fluctuating level of consiousness can occur in both meningitis and subarachnoid


haemorrhage (SAH).

Hypertension is a risk factor for SAH, but not diabetes.

Opiate abuse does not increase the risk for SAH.

Cerebral aneurysms are associated with polystic kidney disease. Bleeding can occur from
these and polycystic kindey disease may be linked with hypertension.

394. A 17-year-old boy has learning difficulties and is seen in the genetics clinic as his
maternal uncles also had learning difficulties.

Examination reveals that the patient has large ears and large testes.

What is the most likely genetic diagnosis?

(Please select 1 option)

47 XYY

Acromegaly

Fragile X syndrome Correct

Klinefelter’s syndrome

Mosaic Down’s syndrome

In addition to moderate to severe mental retardation, other characteristics of individuals with


Fragile X syndrome may include:

large ears
macroorchidism
prognathism
speech delays
prominent forehead
double-jointedness
autistic symptoms, and
occasional self-mutilation.

The face is typically long and narrow, with a high arched palate and large ears.

Otitis media, strabismus, and dental problems may be present. Other common
characteristics include:

hyperextensible joints
hypotonia, and
heart problems, including mitral valve prolapse.

In post pubertal males, abnormally large testes are a distinctive feature.

The following can occur In young children:

delayed motor development


hyperactivity
behavioural problems
toe walking, and
occasional seizures.

395. A 26-year-old man with long term bipolar disorder complains of lethargy, weight gain
and hypothermia.

With which antipsychotic would you associate these side effects?

(Please select 1 option)

Amitriptyline

Chlorpromazine Incorrect answer selected

Clozapine

Fluoxetine

Lithium This is the correct answer

Lithium is notorious for its side effects.

Among the side effects of lithium are a range of thyroid disorders (hypothyroidism,
hyperthyroidism and thyroiditis).
396. A 55-year-old lady presents to her general practitioner complaining of feeling weak.

She has difficulty combing her hair in the morning and also has difficulty climbing the stairs
at home. Over the last three months she has developed problems swallowing her food.

What is the most likely diagnosis?

(Please select 1 option)

Dermatomyositis Incorrect answer selected

Diffuse cutaneous scleroderma

Osteomalacia

Polymyositis This is the correct answer

Systemic lupus erythematosus

This lady has polymyositis. Muscle biopsy is the definitive test in establishing the diagnosis
and anti-Jo-1 antibodies may be present.

Both polymyositis and dermatomyositis are idiopathic inflammatory myopathies affecting the
skeletal muscles. They result in muscle weakness with widespread multi-system symptoms.
However, dermatomyositis can be distiguished by pathognomic clinical signs such as
Gottron's papules and heliotrope rash.

Osteomalacia may present with muscle weakness and difficulty in walking (typically a
'waddling gait' is described). However it should not cause weakness in the upper limbs to the
extent that the patient is unable to raise the arms.

397. Which of the following volatile agents is associated with hepatitis?

(Please select 1 option)

Desflurane

Enflurane

Halothane Correct

Isoflurane

Sevoflurane

Halothane hepatitis may result in damage ranging from minor derangement in liver function
tests to fulminant hepatic failure.

It has been described as the appearance of liver damage within 28 days of halothane
exposure when other known causes of liver disease have been excluded.
Seventy five per cent of patients with halothane hepatitis have antibodies reacting to
halothane-altered antigens.

Halothane should be avoided if:

A previous exposure has occurred within three months


There is a known adverse reaction to halothane
There is a family history of adverse reaction
Pre-existing liver disease.

KEY LEARNING POINTS

Pharmacology

Lithium has a large side effect profile of which a doctor needs to be aware in
order to recognise any untoward symptoms.

Rheumatology

Gottron's papules and a heliotrope rash are pathognomic for dermatomyositis.

ENT

Aural polyps can be indicators of underlying disease, including chronic otitis


media and cholesteatoma.

398. In Winter, a 2-month-old boy presents with a two day history of coryza and difficulty
in feeding. He develops cough and wheeze, and mother is concerned about his
breathing pattern.

He was born at 28 weeks gestation and required seven days of ventilation for surfactant
deficient lung disease. Mother is a heavy smoker.

On examination he has O2 saturations of 91% in air, with a heart rate of 150/minute and
respiratory rate of 60/minute with moderate recession.

What is the most likely diagnosis?

(Please select 1 option)

Aspiration pneumonia

Bronchiolitis Correct
Cystic fibrosis

Pneumococcal pneumonia

Virus-induced wheeze

This is a classical picture of bronchiolitis.

The causative organism is almost always respiratory syncytial virus (RSV).

The likelihood of admission is increased with prematurity and maternal smoking.

Other risk factors for severity are:

Congenital heart disease


Immunodeficiency, and
Gastro-oesophageal reflux.

399. A 57-year-old smoker gives a three month history of persistent hoarseness. On direct
questioning he admits to right-sided earache.

On examination he is hoarse and has mild stridor. Examination of his ears is normal.
Endoscopy of his upper airway shows an irregular mass in the larynx.

What is the most likely diagnosis?

(Please select 1 option)

Carcinoma of the larynx Correct

Laryngeal lymphoma

Laryngeal papillomatosis

Laryngeal tuberculosis

Thyroid carcinoma

This patient is a heavy smoker, his symptoms are of laryngeal pathology, and an irregular
mass is noted on nasal endoscopy.

These features point to a diagnosis of laryngeal carcinoma.


400. A 65-year-old male smoker with severe rheumatoid arthritis presents with a dry
cough and progressive dyspnoea over the past months.

On examination he is mildly cyanosed and has end inspiratory crepitations. A chest x ray
shows widespread reticulonodular changes.

What is the single most likely diagnosis?

(Please select 1 option)

Bronchial asthma

COPD

Pneumonia

Respiratory failure

Rheumatoid lung Correct

This patient has historical and examination features suggesting pulmonary fibrosis. This
suggests rheumatoid lung; it is more common in severe rheumatoid disease and in smokers.
The CXR changes reported are consistent with the diagnosis.

To make a diagnosis of respiratory failure one would need a blood gas result.

Bronchial asthma would be suggested in reversible airways obstruction and hence


fluctuation of symptoms and wheeze on auscultation.

The history is inconsistent with chronic obstructive pulmonary disease (COPD).

Pneumonia would be suggested by

Infective symptoms
Pyrexia
Consolidation on CXR.

401. Which of the following would be a definitely abnormal finding in a 75-year-old man?

(Please select 1 option)

Corneal arcus

Pingueculae

Posterior vitreous detachment Incorrect answer selected

Presbyopia

Xanthelasma This is the correct answer


Xanthelasma suggests hyperlipidaemia in any age group whereas the finding of
corneal arcus diminishes in significance after the age of 70.

Pingueculae are yellowish nodules on the nasal and temporal conjuctiva which are
common in advanced age and are of no consequence.

Posterior vitreous detachment occurs when the vitreous partially detaches from the
retina. It gives rise to the sensation of floaters or, if the vitreous tugs on the retina,
flashing lights. No treatment is required but it may predispose to retinal detachment.

Presbyopia is the name given to the gradual decline in accommodation with age owing
to the loss of elasticity in the lens.

402. A 72-year-old female is diagnosed with giant cell arteritis and is treated with
prednisolone 60 mg per day.

What is the most appropriate treatment for the prevention of steroid induced osteoporosis?

(Please select 1 option)

Alendronic acid This is the correct answer

Calcium Incorrect answer selected

Raloxifene

Tibolone

Vitamin D

Oral glucocorticoids are associated with significant increase in fracture risk, from doses as
low as 5mg daily. Loss of bone-mineral density is greatest in the first few months of
glucocorticoid therapy, but fracture risk declines rapidly after stopping. There is an increased
risk of fracture over and above the effect of low bone mineral density.

Patients older than 65 years are considered at high risk of osteoporotic fractureas are those
with a prior fragility fracture, and they should commence on bone-protective therapy at the
time of starting glucocorticoid therapy. Measurement of bone density is not required before
starting therapy. In patients younger than 65 years without risk factors, DEXA scan is
recommended for assessment of fracture risk.

General measures to reduce bone loss include use of the lowest dose of glucocorticoids
possible, and steroid-sparing agents. Dietary calcium should be increased and physical
activity, with smoking and alcohol minimised. Daily intake 1,500 mg of calcium and 800U of
vitamin D3 is recommended. Bone-protective therapy which can be used includes:

alendronate
alfacalcidol
calcitonin
calcitriol
cyclic etidronate, and
risedronate.

Bisphosphonates are generally considered first line. If unsuitable, then calcitriol or strontium
ranelate may be considered.

Raloxifene is an selective oestrogen receptor modulator (SERM) that has oestrogenic


actions and anti-oestrogenic actions on the uterus and breast. It can be used in the
prevention of postmenopausal osteoporosis where bisphosphonates are not suitable, but not
commonly in steroid-induced osteoporosis.

Tibolone is a form of hormone-replacement therapy which can be used in post-menopausal


women. However, it should not be considered first-line therapy, and is only used where other
therapies are contra-indicated, not tolerated, or there is a lack of response.

403. Which feature does not occur in syringomyelia?

(Please select 1 option)

Charcot joints

Chiari type I malformation

Dissociated sensory loss

Horner's syndrome

Proximal muscle weakness Correct

Syringomyelia is a chronic disorder characterised by the presence of long glial-lined cavities


situated in the central part of the cord, most commonly at cervical and upper thoracic level,
but sometimes extending into the medulla.

Recognised causes include

Chiari type I malformation


Central cord tumours
Basal arachnoiditis and
Trauma.

Clinical features are

1. Sensory: Decussating fibres (pain and temperature) travel close to the midline and are
therefore affected first producing an area of dissociated sensory loss in the territory of the
affected segments. As the syrinx extends upwards into the medulla the fifth nerve nucleus
becomes affected causing progression of the area of sensory impairment on to the face. As
the lesion enlarges, the spinothalamic tract and subsequently dorsal columns become
affected causing corresponding sensory deficits below the level of the lesion.
2. Motor: Destruction of anterior horn cells compressed by the syrinx results in wasting of the
muscles subserved by them; the small hand muscles are often affected first since the
lesion usually begins in the cervico-thoracic cord. Later the pyramidal tracts may be
compressed causing pyramidal signs below the level of the lesion. Sympathetic motor
fibres to the pupil travel close to the midline in the cervical cord and may be affected early,
producing uni- or bilateral Horner's syndrome.
3. Trophic: Sensory and sympathetic denervation result in abnormalities of sweating,
thickening of subcutaneous tissues, atrophy and decalcification of bones, and the
development of Charcot's joints.
4. Where the condition is caused by tumour or arachnoiditis, resection or decompression may
retard or abolish progression. Cases associated with Chiari I malformation may benefit from
decompression of the foramen magnum.

404. Which condition is associated with inflammatory bowel disease in over 70% of
cases?

(Please select 1 option)

Idiopathic autoimmune hepatitis

Primary biliary cirrhosis

Primary sclerosing cholangitis Correct

Steatohepatitis

Vanishing bile duct syndrome

Seventy five percent of primary sclerosing cholangitis is associated with inflammatory


bowel diease, usually ulcerative colitis.

It progresses even if the colitis is controlled.

Next question

405. A 25-year-old female presents with a three month history of amenorrhoea and
galactorrhoea.

Which of the following is most likely?

(Please select 1 option)

Adrenal failure
Hypothyroidism Incorrect answer selected

Liver failure

Pregnancy This is the correct answer

Renal failure

All of the above can cause secondary amenorrhoea and galactorrhoea, however the most
likely cause in this age group would be pregnancy.

406. Which of the following regarding parathyroid hormone is true?

(Please select 1 option)

Decreases the renal clearance of phosphate

Depresses the activity of the parathyroid glands

Is increased in the blood when the calcium levels rise

Is secreted by the anterior pituitary gland

Mobilises calcium from bone independently of its action on the kidney Correct

Parathormone (PTH) is an 84aa protein secreted by the parathyroid gland.

PTH levels rise as serum ionised calcium levels decrease.

It increases plasma calcium by

Increasing osteoclastic resorption of bone (rapid effect)


Increasing intestinal absorption of calcium (slow effect)
Increasing synthesis of 1, 25-(OH)2-D3
Increasing tubular resorption of calcium and
Increasing the excretion of phosphate.

It acts via specific receptors in the membrane that activate adenylate cyclase.

407. For which of the following patients would a gamma irradiated blood product be
recommended?

(Please select 1 option)

A 16-year-old thalassaemic receiving regular transfusions

A 19-year-old nulliparous female after a road traffic accident


A 37-year-old patient with Hodgkins' lymphoma receiving chemotherapy Correct

A 42-year-old lady receiving adjuvant hormonal therapy for breast cancer post radical mastectomy

Postoperatively for carcinoma of the colon in a 50-year-old male

The most common indications for irradiated blood products include

Those at risk of transfusion associated with graft versus host disease such as neonates
Those receiving purine analogues based chemotherapy
Hodgkin's lymphoma
Immunodeficiency states
Post bone marrow transplants

The other scenarios described here do not necessarily represent a immunosuppressed state
in the list of conditions eligible for a irradiated blood product.

Next question

408. A patient has the presence of lysine, arginine, ornithine and cystine in the urine. The
disorder is autosomal recessive.

The treatment includes urinary alkalinisation and penicillamine.

What type of renal calculus is present?

(Please select 1 option)

Calcium oxylate

Cystine Correct

Pure oxylate

Triple phosphate

Uric acid

In this condition there is a defect in the tubular reabsorption of cystine, orthinine, arginine
and lysine (COAL). Stone formation is far more common in those who are homozygotes for
the condition.

Most patients have no underlying abnormality to account for stone formation.

The abnormalities that cause stones to form involve:

Abnormal constituents in the urine


Stasis, or
Infection.

409. A 19-year-old patient attends her GP with a renal colic. She informs her GP that
she has an hereditary disease and that her mother had this problem.

What is the most likely type of renal calculus?

(Please select 1 option)

Calcium oxylate

Cystine Correct

Pure oxylate

Triple phosphate

Uric acid

Although an autosomal recessive condition, the heterozygotes may still have an


increased incidence of cystine stone formation. There are a number of phenotypes and
the expression of the cystinuria gene is very variable.

410. A 20-year-old woman presents to the Emergency department with severe right-sided
abdominal pain and left shoulder pain. She stopped taking the minipill two months ago
and has not menstruated since.

Her blood pressure is 90/50 mmHg and her pulse rate is 120/min.

What is the most appropriate treatment?

(Please select 1 option)

Colposcopy

Intravenous antibiotics

Laparoscopy

Suction curettage

Urgent resuscitation and laparotomy Correct

This woman is hypotensive and is likely to be bleeding from an ectopic pregnancy or a


ruptured ovarian cyst.

In either case urgent resuscitation and laparotomy is the most appropriate form of treatment.
In woman of child bearing age presenting with abdominal pain, it is compulsory to conduct a
beta HCG urine test to exclude pregnancy. Ectopic pregnancies are often missed on
ultrasound scan and so patients are not diagnosed, but a ruptured ectopic can be life
threatening for the woman.

411. A 33-year-old male is admitted to the Emergency department after a fire accident
with petrol in his garage.

He is conscious, breathless but vital signs are fine. However, he has singed his nostrils and
has pain in his throat.

What is the most appropriate immediate treatment?

(Please select 1 option)

Anaesthetise and intubate Correct

Escharotomy

Intravenous fluids

Intravenous opiates

Referral to the specialised burns unit

The issue here is the potential major airway damage. His symptoms and signs indicate
upper airway and probable lung injury. Although there is no immediate airway problem, this
patient may benefit from anaesthesia and intubation to protect his airway and reduce the risk
of pulmonary complications.

Burns management is dictated by the extent and degree of burns. The extent of burns in an
adult is calculated by the 'rule of nines'. The degree of the burn is defined its depth: partial
thickness or full thickness.

Inhalation injury greatly increases the mortality of burn patients. Indicators of such injury are:

burns sustained in a closed space


facial or oropharyngeal burns
singed nasal hair, and
carbonaceous sputum.

Such patients may benefit from early prophylactic intubation and ventilation.

412. Which of the following drugs is an anticholinergic bronchodilator, often used in


addition to adrenoceptor stimulants?

(Please select 1 option)


Aminophylline

Fluticasone propionate

Oxitropium bromide Correct

Sodium cromoglicate

Zafirlukast

Oxitropium bromide is an anticholinergic agent that causes bronchodilatation. Ipratropium


bromide is a similar drug.

Aminophylline is a nonselective adenosine receptor antagonist and phosphodiesterase


inhibitor and aids bronchodilation.

Fluticasone is a synthetic glucocorticoid which acts as an anti-inflammatory and helps in


maintenance of asthma.

Sodium cromoglicate prevents activation of inflammatory cells and is useful in mild asthma.
It is administered intranasally.

Zafirlukast is a leukotriene receptor antagonist which blocks the release of leukotriene and
so reduces bronchoconstriction.

413. A 35-year-old lady presents to her GP with a four month history of dry eyes and dry
mouth.

Select the most appropriate diagnosis for this patient.

(Please select 1 option)

Diffuse cutaneous scleroderma

Limited cutaneous scleroderma

Primary Sjögren's syndrome Correct

Secondary Sjögren's syndrome

Systemic lupus erythematosus

This lady has primary Sjögren's syndrome.

Anti-Ro and anti-La antibodies are present in 70% of patients with this condition and a
positive Schirmer test would confirm defective tear production.

Next question
414. A 70-year-old retired carpenter presents to his GP with a four month history of right-
sided nasal blockage. Recently he had a blood stained nasal discharge.

He also complains of epiphora and that his dentures are now ill-fitting.

What is the most likely diagnosis?

(Please select 1 option)

Acute ethmoiditis

Acute maxillary sinusitis

Maxillary adenocarcinoma Correct

Nasal polyps

Vasomotor rhinitis

This man has a maxillary adenocarcinoma caused by longstanding exposure to wood dusts.
Maxillary adenocarcinoma is an aggressive head and neck malignancy which can spread
into the palate and cause local pressure symptoms.

Spread into the palate it has caused a lump with ill-fitting dentures.

Blockage of the lacrimal duct has caused epiphora.

415. These thyroid function tests were obtained on a 72-year-old female who was
receiving treatment for heart failure associated with atrial fibrillation:

Free T4 32.9 pmol/L (9.8-23.1)

TSH <0.02 mU/L (0.35-5.50)

Free T3 11.1 pmol/L (3.5-6.5)

Over the last one month she had noticed agitation and weight loss.

Which of the above drugs may be responsible for the abnormalities of thyroid function?

(Please select 1 option)

Amiodarone Correct

Atenolol
Carbimazole

Digoxin

Verapamil

Amiodarone is an effective class III antiarrhythmic that is frequently used in the treatment of
atrial fibrillation. However, thyroid dysfunction including hypothyroidism and hyperthyroidism
are side effects.

Beta blockers and carbimazole have an antithyroid effect.

This patient's thyroid function tests with elevated T3 and T4 concentrations and suppressed
TSH could be related to an amiodarone-induced hyperthyroidism. If possible the drug should
be stopped and she may require antithyroid therapy.

416.

These are the urea and electrolytes of a 67-year-old male taken before and two weeks after
the introduction of an antihypertensive.

He presents with general tiredness. He is a smoker of 20 cigarettes per day and has cramp-
like calf pain on exertion. His blood pressure is 182/102 mmHg.

Which drug may account for his present symptoms and electrolyte abnormalities?

(Please select 1 option)

Angiotensin converting enzyme inhibitor Correct

Calcium antagonist

Loop diuretic
Spironolactone

Thiazide diuretic

This patient is hypertensive and has symptoms suggestive of intermittent claudication.

This together with the rapid deterioration in renal function following the introduction of an
antihypertensive suggests that the patient has renal artery stenosis and the offending drug is
an ACE inhibitor.

Adequate renal perfusion is facilitated through high angiotensin concentrations in renal


artery stenosis and the inhibition of ACE may result in renal ischaemia with deteriorating
renal function. Angiotensin 2 receptor antagonists may also cause this deterioration.

Withdrawal of the ACE inhibitor is required.

417. Which of the following is the most likely cause of aural polyps?

(Please select 1 option)

Acoustic trauma

Acute mastoiditis

Bell's palsy

Chronic otitis externa Incorrect answer selected

Chronic otitis media This is the correct answer

Ménière's disease

Mycoplasma infection

Sensorineural deafness

Thiazide diuretics

This question tests your knowledge of the causes of aural polyps and how often these
causative factors result in aural polyps.

The commonest causes of aural polyps are chronic otitis media, cholesteatoma and retained
tympanostomy tubes. Constant irritation due to chronic infection is often a cause.

According to one study, the causative factor occurs with the following frequency:

Chronic otitis media - 43%


Cholesteatoma - 29%
Retained tympanostomy tubes - 23%.
Points of note with regard to examination technique: Firstly, bear in mind that this question
asks for the causes, not the consequences, of aural polyps. Secondly, the question asks for
the most likely causes for aural polyps. Whilst chronic otitis externa and mycoplasma
infection can cause aural polyps, they are not amongst the most likely causes.

You might also like